Kaplan Micro Flashcards

1
Q
  1. An unvaccinated dental student has a needle-stick accident involving an empty syringe that
    had been previously used on a patient with a known hepatitis B infection. Which of the
    following is the most probable outcome for the medical worker?
    A. Acute hepatitis followed by recovery
    B. “Healthy” carrier
    C. Persistent infection followed by recovery
    D. Persistent infection progressing to chronic hepatitis
    E. Subclinical disease followed by recovery
A

The correct answer is E. Hepatitis B infection can produce a wide variety of clinical outcomes. The
most common outcome (60% to 65%), however, turns out to be subclinical disease followed by
complete recovery. The other choices listed show other possible outcomes, and their statistical
impact. Approximately 20% to 25% of infected persons develop acute hepatitis (choice A), which is
followed in 99% of these cases by recovery and in about 1% of cases by fulminant hepatitis.
Approximately 5% to 10% of cases become “healthy” carriers (choice B). Approximately 4% of cases
develop persistent infection, 67% to 90% of which then recover (choice C) and 10% to 33% of which
have chronic hepatitis (choice D).

How well did you know this?
1
Not at all
2
3
4
5
Perfectly
2
Q
  1. Biopsy of an ulcerated gastric lesion on a 60-year-old smoker demonstrates the presence of
    a gastric carcinoma. If the patient noted that he had severe “heart burn” for the past several
    years, with which of the following infectious agents has this type of lesion been most strongly
    associated?
    A. Epstein-Barr virus
    B. Helicobacter pylori
    C. Human papilloma virus
    D. Molluscum contagiosum virus
    E. Schistosoma haematobium
A

The correct answer is B. The patient has gastric carcinoma, which has been strongly linked, in at
least some studies, to prior gastric infection with Helicobacter pylori. H. pylori has also been
implicated in the etiologies of gastric peptic ulcer, chronic gastritis, and (questionably) gastric
lymphoma. It is believed that treatment of the H. pylori infection with a combination antibiotic
treatment regimen, such as amoxicillin + metronidazole + proton pump inhibitor + bismuth
subsalicylate will decrease the incidence of this type of carcinoma. Epstein-Barr virus (choice A) has
been linked to African Burkitt lymphoma and nasopharyngeal carcinoma. Human papilloma virus
(choice C) has been linked to a variety of warts, condyloma, and genital cancers. Molluscum
contagiosum virus (choice D) is a poxvirus that causes small tumor-like papules of the skin.
Schistosoma haematobium(choice E) has been linked to bladder cancer.

How well did you know this?
1
Not at all
2
3
4
5
Perfectly
3
Q
  1. The biological attribute of influenza A virus, which allows the sudden appearance of
    dramatically new genetic variants, is also present in a limited number of other viral families.
    Which of the following viruses also possesses this biological attribute?
    A. Coronavirus
    B. HIV
    C. Measles virus
    D. Rotavirus
    E. Rubella virus
A

The correct answer is D. Pandemics of influenza A can be caused by the ability of the virus to
undergo dramatic genetic changes of type by reassortment of its segmented RNA genome, a trait
called genetic shift. The only virus on the list that possesses a segmented genome is the rotavirus, in
the reovirus family, which possesses 10-11 segments in its genome. Coronavirus (choice A) is not
segmented and is a cause of the common cold. HIV (choice B) is not segmented and is known for its
genetic drift (minor mutational changes over time caused by an error-prone polymerase), not genetic
shift. Measles virus (choice C) is not segmented and is controlled largely by vaccination. The virus is
known as a paramyxovirus. Rubella virus (choice E) is not segmented. The togavirus causes
arthralgia (primarily in young women), fever, malaise, coryza, lymphadenopathy, and a fine
maculopapular rash.

How well did you know this?
1
Not at all
2
3
4
5
Perfectly
4
Q
  1. Three months after a needle-stick exposure to blood from a patient with hepatitis
    1
    B, a nurse is evaluated for infection with the virus. Laboratory results reveal: HBsAg absent anti-HBs
    antibody absent IgM anti-HBc present IgG anti-HBc absent HBeAg absent. On the basis of these results, which of the following most accurately describes the nurse’s
    hepatitis B status?
    A. She had been effectively vaccinated against hepatitis B bfore the needle-stick exposure occurred
    B. She has mounted an inappropriate antibody response to hepatitis B as a result of an
    immunocompromised state
    C. She is a carrier of hepatitis B
    D. She is actively infected with hepatitis B
    E. She was not infected with hepatitis B
A

The correct answer is choice D. The nurse’s elevated IgM anti-HBc indicates that she was infected
with hepatitis B. Formerly, HBsAg (surface antigen) and anti-HBsAg (antibody to surface antigen)
were used exclusively to determine this. Typically, HBsAg is positive for up to 6 months, and anti
HBsAg is positive for years after that. Unfortunately, this simple scheme has the disadvantage that
many patients have a 2-week to 4-month “window” period, when the surface antigen (HBsAg) and the
antibody (anti-HBs) are not detectable. Presumably, for a relatively brief period, HBsAg production
exactly matches antibody production, and the two coprecipitate such that neither free species is
present in adequate concentration to be detectable. This problem can be circumvented by concurrent
measurements of other antigens and antibodies, including HBeAg, anti HBe, and anti-HBc (HBcAg is
not reliable). During the window period, IgM anti-HBc may be the only marker of recent HBV infection,
as it is in this nurse.
If the nurse had been effectively vaccinated for hepatitis B (choice A), she would have had an
elevated anti-HBs antibody level and no HBsAg present in the serum. Anti-HBc antibody would have
been absent as well. The antibody response to hepatitis B infection was appropriate in this person,
which argues against immunocompromise (choice B). Carriers (choice C) have elevated anti-HBs
and may have persistently elevated HBsAg (in approximately 10% of cases). IgG anti-HBc (not IgM)
predominates in these chronic patients. Had she not been infected with hepatitis B at all (choice E),
IgM anti-HBc would be absent.

How well did you know this?
1
Not at all
2
3
4
5
Perfectly
5
Q
  1. An important factor in determining the likelihood of converting to HIV+ status after a
    needlestick injury is which of the following?
    A. Depth of the needlestick
    B. Volume of blood in the needle
    C. Type of needle
    D. Health status of source patient
    E. All of the above
A

The correct answer is E. If a healthcare worker is accidently stuck with an HIV-infected bloody
sharp, the worker may or may not contract HIV disease. Conversion following needlestick accidents is
low overall, sometimes estimated at 0.3% (3 per thousand). Several factors, however, have been
identified that make it MORE likely that the accident will transfer the virus to the worker. They include
a large volume of blood, a hollow-bore needle, visible blood on the needle, a deep (not superficial)
stick into deep skin or muscle, and a source patient in late stages of AIDS. Followup of needlestick
injuries is a rapidly evolving field, so be sure to use your most recent notes and test reviews to stay on
top of this subject.

How well did you know this?
1
Not at all
2
3
4
5
Perfectly
6
Q
  1. A 16-year-old boy with sickle cell disease is hospitalized for a severe infection. His
    symptoms include fever, chills, cough, and chest pain. Bacteria from the patient’s sputum
    yield optochin-sensitive organisms with a positive Quellung reaction. Which of the following is
    the most likely pathogen?
    A. Escherichia coli
    B. Haemophilus influenzae
    C. Klebsiella pneumoniae
    D. Neisseria gonorrhoeae
    E. Streptococcus pneumoniae
A

The correct answer is E. The combination of optochin sensitivity and positive Quellung reaction is
characteristic of a single organism, Streptococcus pneumoniae (diplococcus). The Quellung reaction
is a capsular swelling caused by contact with specific capsular antisera. The other encapsulated
organisms that have Quellung-positive reactions are Haemophilus influenzae (choice B), Neisseria
meningitidis, and Klebsiella pneumoniae (choice C). None of these organisms, however, are optochin
sensitive. The other choices, Escherichia coli (choice A) and Neisseria gonorrhoeae (choice D), are
not encapsulated.

How well did you know this?
1
Not at all
2
3
4
5
Perfectly
7
Q
  1. A 33-year-old woman has recently returned from overseas and presents with severe, acute,
    right upper quadrant abdominal pain, bloody diarrhea, and tenesmus. CT scan of the liver
    demonstrates lesions identified as abscesses. Which of the following organisms is the most
    likely cause of her illness?
    A. Ascaris lumbricoides
    B. Entamoeba histolytica
    C. Enterobius vermicularis
    D. Salmonella typhi
    E. Shigella species
A

The correct answer is B. The patient probably has hepatic amebiasis, a life-threatening complication
of intestinal infection with Entamoeba histolytica. E. histolytica is transmitted by way of the fecal-oral
route. Intestinal colonization (which may be asymptomatic) always precedes infection of the liver. The
abscesses generally contain necrotic debris, with amoebae located along the edges of the abscess.
Patients often present initially with frequent bloody, small-volume stools, often associated with fever,
abdominal cramps, tenesmus, and fecal urgency. When untreated, this condition can result in the
development of hepatic amebiasis. Ascaris lumbricoides (choice A) can cause intestinal obstruction,
but does not usually cause hepatic abscesses. Enterobius vermicularis (choice C) is the pinworm,
which inhabits the rectum. Bloody diarrhea can also be seen with some strains of Salmonella (choice
D) and Shigella (choice E), but these organisms do not usually cause hepatic abscesses. Bacterial
causes of hepatic abscesses include E. coli, Klebsiella, Streptococcus, Staphylococcus, Bacteroides,
and Pseudomonas.

How well did you know this?
1
Not at all
2
3
4
5
Perfectly
8
Q
  1. A 65-year-old man presents with fever, severe headache, and nuchal rigidity. Lumbar
    puncture reveals cloudy cerebrospinal fluid (CSF) with elevated neutophils, elevated protein,
    and decreased glucose. Which of the following is the most probable etiologic agent of this
    condition in a patient of this age?
    A. Arbovirus
    B. Herpesvirus
    C. Mycobacterium tuberculosis
    D. Streptococcus pneumoniae
    E. Neisseria meningitidis
A

The correct answer is D. The clinical manifestations (fever, headache, and nuchal rigidity), along
with the CSF findings (increased neutrophils, elevated protein, and reduced glucose), strongly
indicate acute pyogenic (bacterial) meningitis as the underlying condition. Of the microorganisms
listed, either Neisseria meningitidis or Streptococcus pneumoniae can cause this form of meningitis;
however, Streptococcus pneumoniae is by far the most frequent organism causing acute meningitis in
elderly patients. Arboviruses and herpesviruses (choices A and B) can cause an encephalitis
characterized by lymphocytic infiltration of the brain parenchyma and leptomeninges. In this case,
CSF findings would include an increased number of lymphocytes and a normal glucose concentration,
although the protein level in CSF would be increased. Mycobacterium tuberculosis(choice C) may
cause a chronic meningoencephalitis, with a prolonged clinical course. It is characterized
pathologically by a dense granulomatous infiltrate of the base of the brain. Associated CSF findings
include increased lymphocytes and normal or slightly decreased glucose.

How well did you know this?
1
Not at all
2
3
4
5
Perfectly
9
Q
  1. A child has a history of recurrent infections with organisms having polysaccharide antigens
    (i.e., Streptococcus pneumoniae and Haemophilus influenzae). This susceptibility can be
    explained by a deficiency of
    A. C3 nephritic factor
    B. C5
    C. IgG subclass 2
    D. secretory IgA
A

The correct answer is C. IgG is the predominant antibody in the secondary immune response. IgG
subclass 2 is directed against polysaccharide antigens and is involved in the host defense against
encapsulated bacteria. C3 nephritic factor (choice A) is an IgG autoantibody that binds to C3
convertase, making it resistant to inactivation. This leads to persistently low serum complement levels
and is associated with Type II membranoproliferative glomerulonephritis. C5 (choice B) is a
component of the complement system. C5a is an anaphylatoxin that effects vasodilatation in acute
inflammation. It is also chemotactic for neutrophils and monocytes and increases the expression of
adhesion molecules. A deficiency of C5a would affect the acute inflammatory response against any
microorganism or foreign substance. Secretory IgA (choice D) is the immunoglobulin associated with
mucous membranes. Selective IgA deficiency is the most common hereditary immunodeficiency. In
this disorder, there is failure of the B cell to switch the heavy chain class from IgM to IgA. Patients
have an increased incidence of sinopulmonary infections, diarrhea, allergies, and autoimmune
diseases.

How well did you know this?
1
Not at all
2
3
4
5
Perfectly
10
Q
  1. A sexually active 25-year-old man with epididymitis and orchitis demonstrates a prominent
    leukocytic infiltrate with numerous neutrophils on biopsy. Which of the following organisms is
    the most likely cause of this man’s infection?
    A. Escherichia coli
    B. Mycobacterium tuberculosis
    C. Neisseria gonorrhoeae
    D. Pseudomonas sp.
    E. Treponema pallidum
A

The correct answer is C. Acute epididymitis and orchitis with prominent neutrophils in a sexually
active male are most likely caused by infection with Neisseria gonorrhoeae or Chlamydia trachomatis.
N. gonorrhoeae can produce a nonspecific pattern of acute inflammation (nonspecific epididymitis and
orchitis) or can be sufficiently severe as to cause frank abscesses within the epididymis.
Escherichia coli(choice A) is an important cause of nonspecific epididymitis and orchitis in children
with congenital genitourinary abnormalities and in older men. Mycobacterium tuberculosis(choice B)
can cause tuberculosis of the epididymis and testes, characterized by granuloma formation.
Pseudomonas sp.(choice D) has been implicated as an important cause of nonspecific epididymitis
and orchitis in older men. Treponema pallidum(choice E), the causative agent of syphilis, can cause
testicular involvement with (in later stages) gumma formation, endarteritis, or a prominent plasma cell
infiltrate.

How well did you know this?
1
Not at all
2
3
4
5
Perfectly
11
Q
  1. A 5-year-old child develops a febrile disease with cough, a blotchy rash, and cervical and
    axillary lymphadenopathy. Also noted is an erythematous, maculopapular rash behind the ears
    and along the hairline, involving the neck and, to a lesser extent, the trunk. Examination of this
    patient’s oropharynx would likely reveal which of the following lesions?
    A. Adherent thin, whitish patch on gingiva
    B. Cold sores on the lips
    C. Curdy white material overlying an erythematous base on the oral mucosa
    D. Large shallow ulcers on the oral mucosa
    E. Multiple small white spots on the buccal mucosa
A

The correct answer is E. The disease described is measles (rubeola), which has the typical
presentation described in the question stem. Measles is caused by a Morbillivirus, an RNA virus
belonging to the Paramyxovirus family. Koplik’s spots, which are pathognomonic for measles, are
small, bluish-white spots on the buccal mucosa in the early stages of measles. These lesions appear
just before the onset of the characteristic rash which is a brick red irregular maculopapular rash that
onsets 3-4 days after the onset of the prodrome (which can also involve the extremities) and fade as
the rash develops. This is usually a prodrome of fever, coryza, cough, conjunctivitis, malaise, irritibility
and photophobia as well as koplik’s spots. Leukoplakia is a premalignant condition characterized by
adherent whitish patches on the gingiva (choice A) and other sites in the oral cavity. Histologically,
they are similar to hyperkeratoses. Cold sores of the lips (choice B) are due to infection with herpes
viruses. Candida infection (thrush) produces curdy white material loosely attached to an erythematous
base (choice C). When removed there is typically a painful lesion that may bleed. Aphthous ulcers
are large shallow ulcers of the oral mucosa (choice D), commonly known as canker sores. No
systemic involvement is seen.

How well did you know this?
1
Not at all
2
3
4
5
Perfectly
12
Q
  1. A 38-year-old woman vacationing in Connecticut is bitten by a tick and develops chronic
    arthritis of the knee and hip joints and paralysis of the left facial muscles. A physical
    examination during the early stages of the disorder would most likely have revealed
    A. aphthous ulcers in the mouth
    B. erythema chronicum migrans
    C. flaccid paralysis of limb flexors
    D. purpuric lesions in a bathing trunk distribution
    E. spastic paralysis of limb extensors
A

The correct answer is B. Lyme disease should be suspected in a patient who is bitten by a tick in the
northeastern United States. Lyme disease was named after a township in eastern Connecticut where
the disease was endemic. The disease is spread by way of a tick vector that transmits a spirochete
that causes a systemic illness. Erythema chronicum migrans is usually the first sign of the illness. This
is a large red patch on the buttocks or chest that slowly expands as the center blanches. Generally,
patients also have constitutional symptoms, such as fever and chills, during this phase. Stiff neck may
develop, along with other signs of meningeal irritation, because of an aseptic meningitis. Other
neurologic complications of Lyme disease include Bell palsy caused by involvement of branches of
the facial nerve. Arthritis is a prominent feature in approximately half the patients with Lyme disease.
It tends to appear several months after the infection but may persist for several years. The course of
the chronic arthritis shows exacerbations and remissions; the most commonly affected joints are the
knees and hips. Cardiac abnormalities in Lyme disease include pericarditis and heart block.
Skin manifestations do not include aphthous ulcers (choice A). Flaccid or spastic paralysis of limbs
(choices C and E) does not accompany Lyme disease; neurologic involvement is generally limited to
cranial nerves and meningitis. Purpura (choice D) is associated with vasculitis and does not occur in
Lyme disease.

How well did you know this?
1
Not at all
2
3
4
5
Perfectly
13
Q
  1. A 31-year-old HIV-positive man develops a severe pneumonia. Lower respiratory tract
    secretions stained with methenamine silver stain demonstrate cup-shaped cysts with sharply
    outlined walls. Which of the following organisms is the most likely pathogen in this case?
    A. Candida albicans
    B. Giardia lamblia
    C. Haemophilus influenzae
    D. Pneumocystis carinii
    E. Streptococcus pneumoniae
A

The correct answer is D. The organism described is Pneumocystis carinii, which is an opportunistic
parasite that seems to be more closely related to fungi than to protozoa. Its cyst form, when stained
with silver stains, has the distinctive appearance described in the question stem, and is typically found
in frothy material that occupies the lumen of alveoli. Pneumocystis pneumonia is a common infection
among AIDS patients, and is very uncommon in other clinical settings. Formerly, many AIDS patients
died with Pneumocystis pneumonia, but the combination of early drug treatment (with
trimethoprim/sulfamethoxazole or pentamidine) and prophylaxis (usually with
trimethoprim/sulfamethoxazole) has decreased the number of fatal infections. In severe cases,
Pneumocystis infection can sometimes be demonstrated in extrapulmonary sites. Candida
albicans(choice A) can infect the lung and stain with methenamine silver, but the respiratory tract
secretions would probably include fungal hyphae and yeast forms. Giardia lamblia (choice B) causes
diarrhea, rather than pneumonia. Haemophilus influenzae(choice C) and Streptococcus
pneumoniae(choice E) are bacteria that would not stain with silver stains and are not especially
common in HIV-positive patients.

How well did you know this?
1
Not at all
2
3
4
5
Perfectly
14
Q
14. A neonate is born in very poor condition, with a severe, generalized encephalitis. If fever is
noted as well as temporal lobe involvement, which of the following viruses is the most likely
pathogen in this setting?
A. Eastern equine encephalitis virus
B. Herpes simplex type II
C. Herpes zoster-varicella virus
D. Poliomyelitis virus
E. St. Louis encephalitis virus
A

The correct answer is B. Viral causes of neonatal encephalitis include three members of the herpes
family of viruses: herpes simplex I, herpes simplex II, and cytomegalovirus. All three types can have
devastating effects on the neonate, with extensive CNS damage leading to mental retardation,
seizures, and focal neurologic problems. Acyclovir may be of some help in modifying these infections,
but both treatment and prognosis remain very problematic. Herpes simplex encephalitis presents with
nonspecific symptoms: a flu-like prodome followed by headache, fever, behavioral and speech
disturbances. A distinguishable feature is a propensity to involve the temporal lobe with mass effect
on imaging studies and temporal lobe seizure foci on EEG’s. Eastern equine encephalitis virus
(choice A) and St. Louis encephalitis virus (choice E) are causes of epidemic encephalitis but are not
the most likely cause of neonatal encephalitis. Herpes zoster-varicella virus (choice C), unlike herpes
simplex, is not usually a cause of neonatal encephalitis. Poliomyelitis virus (choice D) is a
gastrointestinally transmitted virus that is not usually encountered in neonates (or anyone else in the
U.S. currently).

How well did you know this?
1
Not at all
2
3
4
5
Perfectly
15
Q
  1. A 23-year-old woman with a history of sickle cell disease presents with fever and severe
    bone pain localized to her left tibia. An x-ray film reveals a lytic lesion, and blood cultures
    reveal infection. A bone culture grows gram-negative rods. Which of the following best
    describes the infecting organism?
    A. It is comma-shaped and sensitive to acidic pH
    B. It is a facultative intracellular parasite
    C. It is motile and does not ferment lactose
    D. It is motile and oxidase positive
    E. It is a nonmotile facultative anaerobe
A

The correct answer is C. The presence of sickle cell disease in a question stem is usually a
significant clue. This question tests the fact that patients with sickle cell anemia are more susceptible
to osteomyelitis caused by Salmonella. The patient’s fever, bone pain, and x-ray results indicate
osteomyelitis. Note, however, that Staphylococcus aureus (gram-positive coccus) is the most
common cause of osteomyelitis in sicklers and nonsicklers. If it has not been ruled out on bone
culture, you should have looked for it in the answer choices. Notice that you were required to know
more than just the organism’s name; you needed to know its distinguishing features. Choice C
describes Salmonella (a gram-negative rod) accurately. Salmonella exists in more than 1,800
serotypes and is known to contaminate poultry.
A comma-shaped organism that is sensitive to acidic pH (choice A) is Vibrio cholerae, a gramnegative
rod that causes severe enterotoxin-induced diarrhea, with “rice water” stools and
dehydration. The toxin acts by stimulating adenylyl cyclase to overproduce cAMP in the brush border
of the small intestine. A facultative intracellular parasite (choice B) is Legionella, a catalase-positive
gram-negative rod. It contaminates air-conditioning cooling towers and causes Legionnaire disease (a
type of pneumonia). A motile and oxidase-positive organism (choice D) is Pseudomonas, a gramnegative
rod with pili that sometimes produces a polysaccharide slime layer. P. aeruginosa is the
prototype and commonly colonizes the lungs of patients with cystic fibrosis. It is associated with bluegreen
pus. A nonmotile, facultative anaerobe (choice E) is Shigella, a gram-negative rod that does
not produce H2S. All Shigella contain an endotoxic lipopolysaccharide. The organism causes bacillary
dysentery, with abdominal cramps, fever, and mucoid, bloody diarrhea.

How well did you know this?
1
Not at all
2
3
4
5
Perfectly
16
Q
  1. A patient suffers an infarct in the territory of the middle cerebral artery. Pathologic
    examination of the patient’s brain would likely show
    A. caseous necrosis
    B. coagulative necrosis
    C. enzymatic fat necrosis
    D. gangrenous necrosis
    E. liquefactive necrosis
A

The correct answer is E. Liquefactive necrosis occurs in brain or other neural tissues and in
pancreatic tissue. In this type of necrosis, the tissue appears liquefied under the microscope, without
preservation of cell outlines. Liquefactive necrosis can also be seen in some bacterial infections,
especially those caused by pyogenic Staphylococci, Streptococci, or certain coliform bacteria.
Caseous necrosis (choice A) is generally an indication of infection by Mycobacterium tuberculosis.
6
The term caseous refers to the appearance of the tissue, i.e., soft, white necrotic areas that have a
cheese-like appearance. Microscopically, the necrotic areas are lightly eosinophilic (stain light pink),
with little or no discernible cellular detail. The eosinophilia reflects staining of residual cellular proteins.
Coagulative necrosis (choice B) is a more common type of necrosis, characteristic of anoxic injury
and most infarcts. Cellular outlines are preserved, but proteins are denatured, and the cells stain in an
eosinophilic manner. Enzymatic fat necrosis (choice C) is seen primarily with pancreatic injury when
pancreatic lipases are released and digest fat to form free fatty acids. These fatty acids complex with
calcium, resulting in the production of calcium soaps (saponification) in the pancreatic tissue or in
extrapancreatic fatty tissues (eg, omentum). In gangrene (choice D), bacterial infection is
superimposed on a background of massive necrosis and putrefaction.

How well did you know this?
1
Not at all
2
3
4
5
Perfectly
17
Q
  1. A 4-year-old girl presents with a maculopapular rash on her hands and feet and painful
    ulcers distributed anteriorly on her lips, palate, tongue, and buccal mucosa. Systemic features
    and lymphadenopathy are absent. Which of the following viruses is most likely to have caused
    this disorder?
    A. Coronavirus
    B. Coxsackievirus type A16
    C. Herpes simplex virus type 1
    D. Parainfluenza type 3
    E. Rhinovirus
A

The correct answer is B. Hand-foot-and-mouth disease is characterized by the appearance of ulcers
in the mouth and a maculopapular or vesicular rash on the hands and feet. It is most frequently
caused by coxsackievirus type A16, although other coxsackieviruses have occasionally been
implicated. The disease usually affects young children. Systemic features and lymphadenopathy are
absent, and recovery is uneventful.
Coronavirus (choice A) is a cause of the common cold. Herpes simplex virus type 1 (choice C)
causes a variety of diseases, including gingivostomatitis, pharyngotonsillitis, herpes labialis, genital
herpes, keratoconjunctivitis, and encephalitis. Parainfluenza virus (choice D) is responsible for croup.
Croup, or acute laryngotracheobronchitis, is an acute febrile illness with stridor, hoarseness, and
cough. Rhinovirus (choice E) is a member of Picornaviridae. It is the most common cause of the
common cold.

How well did you know this?
1
Not at all
2
3
4
5
Perfectly
18
Q
  1. Evaluation of an adult third world immigrant to this country demonstrates chronic
    headaches, chronic mild nuchal rigidity, and chronic inflammatory infiltrate of the CSF with
    lymphocytes, plasma cells, macrophages, and fibroblasts. If an acid-fast organism is
    identified, which of the following is the most likely etiologic agent?
    A. Herpes virus
    B. Mumps virus
    C. Mycobacterium tuberculosis
    D. Neisseria meningitidis
    E. Streptococcus pneumoniae
A

The correct answer is C. Although this may seem to be a difficult clinical question, by simply
knowing that mycobacteria are acid-fast organisms, one can answer the question. Nuchal rigidity
suggests meningitis. It is convenient to classify meningitis based on the cerebrospinal fluid (CSF)
findings: (1) acute pyogenic meningitis if neutrophils are markedly increased, (2) acute lymphocytic
meningitis if lymphocytes (alone) are markedly increased, and (3) chronic meningitis if lymphocytes,
plasma cells, macrophages, and fibroblasts are increased. This patient has chronic meningitis. The
classic cause of chronic meningitis is tuberculosis, whose etiologic agent is Mycobacterium
tuberculosis. Other causes include other indolent meningeal infections such as syphilis, brucellosis,
and chronic fungal infections. The granulomas that are characteristic findings in other tissues may or
may not be present in the meningeal tissue, and are usually not recognizable in CSF. Tubercular
meningitis is now uncommon in this country. In immigrants from third world countries, a history of
pulmonary tuberculosis may be distant, undiagnosed, or deliberately concealed. Herpes virus (choice
A) and mumps virus (choice B) are causes of acute lymphocytic meningitis. These organisms are not
acid-fast. Neisseria meningitidis(choice D) is a gram-negative bacteria. Streptococcus
pneumoniae(choice E) is a gram-positive bacteria. Both are causes of acute pyogenic meningitis.
Neither is acid-fast.

How well did you know this?
1
Not at all
2
3
4
5
Perfectly
19
Q
  1. A 45-year-old white man with a history of alcohol abuse and periodontal disease has a
    spiking fever, chills, and lung consolidation. A chest x-ray shows a cavity in the right lower
    lobe that has an air/fluid level. Based on the clinical presentation, which of the following would
    be the most likely cause of the symptoms?
    A. Anaerobic bacteria
    B. Aspergillus fumigatus
    C. Entamoeba histolytica
    D. Staphylococcus aureus
    E. Streptococcus pyogenes
A

The correct answer is A. Anaerobic bacteria derived from the oral flora in the clinical setting of
periodontal disease are the most common isolates from lung abscesses. Single lung abscesses are
the most common pattern, with the superior segment of a lower lobe or the posterior segment of an
upper lobe being affected most often. Prominent members of the oral anaerobic flora include
fusobacteria, prevotella, and aerobic spirochetes. Aspergillus fumigatus(choice B) presents in the
lung as hemorrhagic infarctions, aspergillomas (fungus balls) in cavitary tuberculosis cavities, or as
allergic bronchopulmonary aspergillosis. Entamoeba histolytica(choice C) is associated with
pulmonary abscesses as an extension of a liver abscess across the diaphragm. Staphylococcus
aureus(choice D) usually presents as multiple lung lesions in noncontiguous sites because the
spread is embolic. The source of the infection is usually tricuspid endocarditis in intravenous drug
abusers. Streptococcus pyogenes(choice E) typically produces a bronchopneumonia pattern
following an upper respiratory infection.

How well did you know this?
1
Not at all
2
3
4
5
Perfectly
20
Q
  1. A young mother takes her infant to the pediatrician, who notices the infant’s teeth are
    yellow and brownish striped. The antibiotic this mother most likely took during pregnancy
    A. inhibits aminoacyl-tRNA binding
    B. inhibits peptidyl transferase
    C. interferes with cell wall synthesis
    D. is a large, cyclic lactone-ring structure
A

The correct answer is A. This question relates to an NDBE favorite side effect: the teeth mottling
that occurs when a child is exposed to tetracycline in utero. You should remember that tetracycline is
contraindicated in pregnancy and early childhood up to approximately age 8 years. Tetracycline is a
bacteriostatic drug that binds to the 30s subunit of ribosomes, preventing aminoacyl-tRNA from
binding with complementary mRNA. This inhibits peptide bond synthesis. Resistance is plasmidmediated.
Inhibition of peptidyl transferase (choice B) occurs with chloramphenicol, a broad-spectrum
bacteriostatic agent that binds to the 50s subunit of ribosomes. Resistance is plasmid-mediated. It has
high toxicity (gastrointestinal disturbances, aplastic anemia, and gray baby syndrome), so it is used
mainly in severe infections or as a topical agent. Interference with cell wall synthesis (choice C)
occurs with penicillins and cephalosporins, the beta-lactam antibiotics. Resistance to these drugs
appears in organisms that have developed beta-lactamases (penicillinases), enzymes that destroy the
beta-lactam ring of these medications. The wider spectrum ampicillin, amoxicillin, ticarcillin, and
carbenicillin are particularly penicillinase susceptible. Large, cyclic, lactone-ring structures (choice D)
describe the macrolides: erythromycin, azithromycin, and clarithromycin. They inhibit bacterial protein
synthesis by reacting with the 50s ribosomal subunit and preventing the release of the uncharged
tRNA. Resistance is plasmid-mediated. Common side effects include gastrointestinal irritation, skin
rashes, and eosinophilia. Erythromycin is a popular choice for patients with penicillin hypersensitivity.
It is a cytochrome p450 inhibitor and therefore must be used with caution in patients taking other
drugs.

How well did you know this?
1
Not at all
2
3
4
5
Perfectly
21
Q
  1. A 38-year-old woman complains of cold and painful fingertips, as well as difficulty
    swallowing and indigestion. Physical examination shows a thickened, shiny epidermis over
    the entire body, with restricted movement of the extremities, particularly the fingers, which
    appear claw-like. Which of the following autoantibodies will likely be found in this patient’s
    serum?
    A. Anti-DNA topoisomerase I (anti-Scl-70)
    B. Anti-double-stranded DNA (ds DNA)
    C. Anti-IgG
    D. Anti-Sm
    E. Anti-SS-A
A

The correct answer is A. This patient has systemic sclerosis, also called scleroderma. Antibodies to
topoisomerase I (anti-Scl-70) occur in up to 70% of patients with diffuse systemic sclerosis, but only
rarely in other disorders. Systemic sclerosis is characterized intitially by excessive fibrosis and edema
of the skin, especially the hands and fingers, producing sclerodactyly (characteristic changes in the
fingers, which resemble claws). Raynaud phenomenon is common. The diffuse type of systemic
sclerosis generally spreads to include visceral organs, such as the esophagus (producing dysphagia),
the lungs (producing pulmonary fibrosis), the heart (leading to heart failure or arrhythmia), and the
kidneys (renal failure causes 50% of scleroderma deaths). A more restricted variant of systemic
sclerosis with a somewhat more benign course is CREST syndrome (Calcinosis, Raynaud syndrome,
Esophageal dysmotility, Sclerodactyly, and Telangiectasia), characterized by the presence of anticentromere
antibodies (although 10% of CREST patients have antitopoisomerase antibody also).
Anti-ds DNA (choice B) and anti-Sm (Smith antigen; choice D) are characteristic of systemic lupus
erythematosus (SLE) but are not common in patients with systemic sclerosis. Rheumatoid factor is an
autoantibody directed against IgG (choice C). It is found in patients with rheumatoid arthritis. Anti-SSA
(choice E) is typically seen in Sjogren syndrome (although it may also be seen in SLE).

How well did you know this?
1
Not at all
2
3
4
5
Perfectly
22
Q
  1. A young patient presents with bilateral swelling of the parotid gland, accompanied by
    swelling of one testicle. Which of the following is the most likely diagnosis?
    A. Gonorrhea
    B. Mumps
    C. Nonspecific epididymitis and orchitis
    D. Syphilis
    E. Tuberculosis
A

The correct answer is B. Mumps, once a common childhood disease, is now much less common
because of immunization. It still should be considered as a potential cause of parotitis in medically
underserved populations, however. It is caused by the paramyxovirus and is spread by droplet
infection. Although testicular involvement by mumps in school-aged children is extremely uncommon,
postpubertal males who develop this viral infection have a 20-30% chance of subsequently
developing acute interstitial orchitis, usually in one testis. Microscopically, the inflammatory reaction
consists of lymphocytes, plasma cells, and macrophages. Although the process can be locally
destructive, sterility does not usually develop because typically only one testicle is involved. The
incubation period is 14-21 days before the onset of symptoms. Patients initially present with painful,
swollen salivary glands, usually the parotid. In unvaccinated individuals, other tissues frequently
become involved. These tissues include the testes, pancreas, and meninges. Neglected gonorrhea
infection (choice A) of the epididymis and testis occurs in the setting of previous sexual activity and
would not usually cause parotitis. Nonspecific epididymitis and orchitis (choice C) occur in children
with genital urinary congenital abnormalities (gram-negative rods), in sexually active young adults
(Chlamydia and gonorrhea), and in older men (gram-negative rods and Pseudomonas). Syphilis
(choice D) can cause orchitis with either gumma formation or a leukocytic infiltrate with prominent
plasma cells. Tuberculosis (choice E) can cause granulomatous involvement with acid-fast
organisms of the epididymis and testis.

How well did you know this?
1
Not at all
2
3
4
5
Perfectly
23
Q
  1. Which of the following cell surface markers is required for lysis of IgG-coated target cells
    (antibody-dependent, cell-mediated cytotoxicity, or ADCC) by natural killer cells?
    A. CD3
    B. CD16
    C. CD19
    D. CD21
    E. CD56
A

The correct answer is B. CD16 is a cell surface marker used to identify natural killer (NK) cells
(lymphocytes lacking most T- and B-cell markers). CD16 is an Fc receptor for IgG, allowing the NK
cells to bind to the coated target cell during ADCC, facilitating lysis. CD3 (choice A) is a fivepolypeptide
cluster that represents the nonvariable part of the T-cell receptor complex. The variable
part is able to rearrange itself to adapt to specific antigens. NK cells are CD3-negative. CD19 (choice
C) is a B-cell marker. It is a signal-transducing molecule that is expressed in early B-cell
differentiation. NK cells are negative for CD19. C21 (choice D) is also a B-cell marker. It is a
complement receptor, and is also the same receptor the Epstein-Barr virus uses to bind to cells during
infection. CD56 (choice E) is an NK cell marker, but is not involved with ADCC.

How well did you know this?
1
Not at all
2
3
4
5
Perfectly
24
Q
  1. Which of the following organisms would most likely cause infection after a partial
    sterilization procedure that killed vegetative cells but did not kill spores?
    A. Chlamydia
    B. Clostridium
    C. Escherichia
    D. Pseudomonas
    E. Streptococcus
A

The correct answer is B. Although bacterial spores are extensively discussed in microbiology
courses, you should be aware that only Bacillus (aerobic) and Clostridium (anaerobic) species
produce spores. This means that the list of diseases related to bacterial spore formation is also small:
anthrax (Bacillus anthracis), some forms of food poisoning (Bacillus cereus), botulism (Clostridium
botulinum), tetanus (Clostridium tetani), gas gangrene (Clostridium perfringens and others), and
pseudomembranous colitis (Clostridium difficile). This is one of the reasons why some clostridium
infections are difficult to treat. With respect to dentistry, clostridium difficile induced colitis can be seen
following treatment with antibiotics, especially clindamycin. Remember, clindamycin is commonly
used to treat a variety of dental-related infections because it has excellent anaerobic coverage. Note
also that spores (bacillus stearothermophilus) are used to test autoclave function.

How well did you know this?
1
Not at all
2
3
4
5
Perfectly
25
Q
  1. An elderly man develops a vesicular rash localized to a narrow circumferential band on one
    side of his chest. The rash is very painful and the vesicles are confluent with some ulceration.
    No other significant findings are demonstrated on physical examination. Which of the
    following diagnoses is most likely to be correct?
    A. Chicken pox
    B. Herpes simplex I infection
    C. Herpes simplex II infection
    D. Measles
    E. Shingles
A

The correct answer is E. This is shingles, the recurrent form of herpes zoster infection, which is
usually (except in the case of immunosuppressed patients) confined to a single, unilateral dermatome.
Isolated vesicles may be seen outside the dermatome. The primary herpes zoster infection (chicken
pox) precedes the development of shingles by years or decades; the prevalence of shingles rises
steadily with age, to the point that 1% of people older than 80 years of age have the condition.
Shingles lesions are infections and should be considered an infectious hazard in the hospital setting.
Acyclovir can ameliorate the condition. Varicella, or chicken pox (choice A), is the primary form of
herpes zoster infection and affects face and trunk diffusely. It is generally seen in young patients. The
macules evolve to papules and vesicles then crust over; all stages are simultaneously present.
Herpes simplex I (choice B) affects oral and perioral sites. Herpes simplex II (choice C) primarily
affects genital sites, producing a painful set of beefy red lesions. Measles (choice D) causes a
blotchy rash, rather than a dermatomal one. Patients present with a maculopapular rash that is brick
red. It begins on the head and neck and spreads downward and outward. Koplik spots often appear
on the buccal mucosa.

How well did you know this?
1
Not at all
2
3
4
5
Perfectly
26
Q
  1. A 35-year-old man who recently traveled to a third world country develops chronic, severe
    dysentery with ulceration of the cecum. Biopsy reveals 15-40 micron amoebae with ingested
    erythrocytes and small nuclei with distinctive tiny central karyosomes. If hepatomegaly is a
    late complication of this patient’s condition, which of the following organisms is the most
    likely culprit?
    A. Isospora belli
    B. Cryptosporidium parvum
    C. Entamoeba histolytica
    D. Giardia lamblia
    E. Microsporidia spp.
A

The correct answer is C.Entamoebahistolytica is the usual cause of intestinal amebiasis, and has
the microscopic features described in the question stem. A particularly helpful (but not always
present) feature of this organism is the presence of ingested red blood cells within the amoebae.
10
These amoebae cause flask-shaped ulceration of the intestinal mucosa and submucosa, with a
particular propensity for involving the cecum and ascending colon. The disease manifestations range
from none (asymptomatic carriers) to mild chronic diarrhea, to severe, purging dysentery. In
symptomatic cases, the liver may develop destructive amoebic liver abscesses that tend to become
secondarily (and potentially life-threateningly) infected by bacteria. Patients initially present with
recurrent diarrhea and abdominal cramps. As the condition worsens severe colitis may be present.
Then hepatic involvement may occur in fulminant cases. Isospora belli (choice A), Cryptosporidium
parvum (choice B), and Microsporidia spp. (choice E) are commonly seen in AIDS patients. These
organisms can cause severe diarrhea. None of these organisms are amebae. Giardia lamblia(choice
D) is a small intestinal protozoa with a distinctive pear-shaped morphology that appears to have a
“face.”

How well did you know this?
1
Not at all
2
3
4
5
Perfectly
27
Q
27. Which of the following characteristics would help to differentiate Streptococcus agalactiae
from Streptococcus pneumoniae?
A. Alpha-hemolysis
B. Carbohydrate capsule
C. Cytochrome enzyme system
D. Growth in bile
E. Oxacillin sensitivity
A

The correct answer is A. Streptococci are usually initially speciated by their hemolytic capacity on
sheep blood agar. Beta-hemolytic streptococci include groups A, B, and D. S. agalactiae is the classic
group B streptococcus. The non-beta-hemolytic streptococci consist principally of the pneumococci
and the viridans group. These groups are both alpha-hemolytic. Streptococcus is a genus of grampositive
facultative cocci occurring in pairs or chains. The genus is separable into the pyrogenic
group, viridans group, enterococcus group, and lactic group. Because streptococci are so common, it
is essential that the dental student understand how to differentiate all the different species. Note that
alpha hemolysis produces a green halo around the colony in RBC culture, whereas beta hemolysis
produces a clear halo. Both S. agalactiae and pneumococcus have a carbohydrate capsule (choice
B), an important virulence factor and means of subtyping streptococcal species. None of the
streptococci use cytochrome enzymes (choice C). They derive all of their energy from the
fermentation of sugars to lactic acid. Neither pneumococcus nor S. agalactiae can grow in bile
(choice D). This ability is specific for the enterococcus group (group D) of streptococci. Both
pneumococci and S. agalactiae are usually treated with penicillin-type antibiotics, although group B
streptococci require a penicillinase-resistant type such as oxacillin (choice E).

How well did you know this?
1
Not at all
2
3
4
5
Perfectly
28
Q
  1. A 73-year-old woman with a history of diabetes presents with left ear pain and drainage of
    pus from the ear canal. She has swelling and tenderness over the left mastoid bone. Which of
    the following microorganisms is the most likely causative agent?
    A. Haemophilus influenzae
    B. Klebsiella pneumoniae
    C. Mucor sp.
    D. Pseudomonas aeruginosa
    E. Streptococcus pyogenes
A

The correct answer is D.Pseudomonas aeruginosa causes malignant otitis externa, which is a
severe necrotizing infection of the external ear canal. Infection tends to spread to the mastoid bone,
temporal bone, sigmoid sinus, base of the skull, meninges, and brain. Patients at increased risk
include the elderly, those with diabetes, and the immunocompromised. Haemophilus
influenzae(choice A) produces a variety of clinical syndromes. H. influenzae is the third most
common cause of meningitis in children aged 1 month to 18 years. It is the most common cause of
acute epiglottitis, the most common cause of purulent bacterial conjunctivitis, and the second most
common cause of otitis media. Patients at risk include those with COPD and cystic fibrosis, alcoholics,
splenectomized patients, and young patients. Klebsiella pneumoniae(choice B) is a gram-negative
organism that produces a necrotizing pneumonia in diabetic patients and alcoholics. Patients typically
present with an abrupt onset of fever, shaking chills, and purulent, foul-smelling sputum. Mucor
(choice C) is a fungal infection that is particularly severe in the diabetic or immunocompromised
patient. In the acidotic diabetic, the fungus produces a life-threatening, invasive rhinocerebral
infection. The infection begins in the nasal passages, extends into the paranasal sinuses, and
11
spreads through the cribiform plate to the frontal lobes of the brain. Patients typically complain of
headache, facial pain, and orbital swelling. Streptococcus pyogenes(choice E) causes bacterial
pharyngitis, otitis media, and sinusitis. It is also associated with toxin-related diseases and skin
infections.

How well did you know this?
1
Not at all
2
3
4
5
Perfectly
29
Q
29. A poor African community is experiencing an epidemic of severe hepatitis. The mortality
among pregnant women is particularly high. Which of the following viruses is the most likely
cause of the epidemic?
A. Cytomegalovirus (CMV)
B. Hepatitis A virus (HAV)
C. Hepatitis C virus (HCV)
D. Herpes simplex I
E. Hepatitis E virus (HEV)
A

The correct answer is E. Although hepatitis E is not generally seen in this country, it is essential that
the student have an understanding of all forms of hepatitis. Hepatitis E is an important, and until
recently, unrecognized cause of epidemics of enterically transmitted (fecal-oral) acute hepatitis.
Hepatitis E is caused by a virus that occurs primarily in India, Asia, Africa, and Central America.
Infection is associated with a 10-20% mortality among pregnant women. This type of hepatitis needs
to be considered in patients traveling to and from endemic areas. CMV (choice A) can cause acute
hepatitis, but the disease is usually mild and often goes unrecognized, except in profoundly
immunosuppressed patients. Hepatitis A virus (choice B) is the major cause of epidemics of
enterically transmitted viral hepatitis, especially in the United States, but is not a significant cause of
mortality in pregnant women. Hepatitis C virus (choice C) is usually transmitted parenterally rather
than enterically and is not a significant cause of mortality in pregnant women. Herpes simplex (choice
D) usually causes significant hepatitis only in profoundly immunosuppressed patients.

How well did you know this?
1
Not at all
2
3
4
5
Perfectly
30
Q
  1. Several students ate lunch at a restaurant at which they all were served pork with
    vegetables and fried rice. All of the students developed nausea, vomiting, abdominal pain, and
    diarrhea within 4 hours of eating lunch. Which of the following is the most likely cause of
    these symptoms?
    A. Bacillus cereus
    B. Clostridium botulinum
    C. Clostridium perfringens
    D. EHEC (enterohemorrhagic escherichia coli)
    E. Staphylococcus aureus
A

The correct answer is A. Bacillus cereus produces a self-limited diarrhea caused by ingestion of the
preformed enterotoxin in contaminated fried rice (especially when reheated) and seafood. The
incubation period is typically approximately 4 hours. The degree of vomiting is usually greater than
that of diarrhea. B. cereus is also associated with keratitis, producing a corneal ring abscess.
Clostridium botulinum (choice B) produces a neurotoxin that blocks the release of acetylcholine,
resulting in a symmetric descending paralysis that may lead to respiratory complications causing
death. Symptoms include blurred vision, photophobia, dysphagia, nausea, vomiting, and dysphonia.
Most cases are associated with the ingestion of contaminated home-canned food.
Clostridium perfringens (choice C) produces a severe diarrhea with abdominal pain and cramping
(sometimes called “church picnic” diarrhea). The incubation period is 8-24 hours after ingesting
contaminated meat, meat products, or poultry. The meats have usually been cooked, allowed to cool,
and then warmed, which causes germination of the clostridial spores.
EHEC, enterohemorrhagic escherichia coli (choice D), produces a bloody, noninvasive diarrhea
caused by the ingestion of verotoxin found in undercooked hamburger at fast food restaurants. Some
patients develop a life-threatening complication called hemolytic-uremic syndrome. Staphylococcus
aureus (choice E) produces a self-limited food poisoning syndrome with nausea, vomiting, and
abdominal pain followed by diarrhea beginning 1-6 hours after ingestion of the enterotoxin. The
organism is found in foods such as potato salad, custard, milk shakes, and mayonnaise.

How well did you know this?
1
Not at all
2
3
4
5
Perfectly
31
Q
  1. Which of the following organisms is most likely to be implicated as a cause of urethritis
    that persists after antibiotic therapy for gonorrhea?
    A. Actinomyces
    B. Chlamydia
    C. Mycobacteria
    D. Nocardia
    E. Rickettsia
A

The correct answer is B.Chlamydia, unlike the other choices, is a type of sexually transmitted
disease, as is gonorrhea. Gonococcal infections typically produce profuse urethal discharge,
especially in men, yielding a positive smear. Fever, rash and arthritis can occur with disseminated
disease. Chlamydia, Mycoplasma, and Ureaplasma are not effectively treated by penicillins and
cephalosporins, and are important causes of post-gonococcal urethritis. Chlamydial urethritis can be
diagnosed by using fluorescent antibodies to identify inclusions in epithelial cells. Actinomyces(choice
A) is a mouth commensal that rarely causes a deeper oral infection. Mycobacteria(choice C) cause
chronic granulomatous diseases such as tuberculosis and leprosy. Nocardia(choice D) can cause
necrotizing pneumonia and disseminated disease. Rickettsia(choice E) cause typhus and Rocky
Mountain spotted fever.

How well did you know this?
1
Not at all
2
3
4
5
Perfectly
32
Q
32. Which of the following organisms would be most likely to cause an outbreak of enteritis in
a day care center in the United States?
A. Helicobacter jejuni
B. Salmonella typhi
C. Shigella species
D. Vibrio cholerae
E. Yersinia enterocolitica
A

The correct answer is E. Enteritis is an inflammation of the intestine, especially the small intestine.
Yersinia enterocolitica is an important cause of mini-epidemics of pediatric diarrhea. Adults can also
be affected, but less commonly than children. Some diarrheal cases are severe (and occasionally
fatal) and may be complicated by severe dysentery, appendicitis, or chronic relapsing ileocolitis that
may require antibiotics to shorten the course. Yersinia is a non-motile ovoid or rod shaped,
nonencapsulated gram-negative bacteria. Helicobacter jejuni (choice A) is an important cause of
hospital-acquired diarrhea, especially in immunocompromised individuals. Salmonella typhi (choice
B) causes typhoid fever. It is a gram-negative bacteria. Shigella species (choice C) causes epidemics
of dysentery in military camps and other close quarter areas. It is a gram-negative bacteria. Vibrio
cholerae (choice D) causes cholera. It is a gram-negative bacteria. Gastroenteritis can occur in the
United States upon eating improperly cooked seafood from other countries.

How well did you know this?
1
Not at all
2
3
4
5
Perfectly
33
Q
33. In a closed system, spores are formed during which of the following phases of bacterial
growth?
A. Decline phase
B. Exponential phase
C. Lag phase
D. Log phase
E. Stationary phase
A

The correct answer is E. Spore formation usually occurs during the stationary phase, when cell
growth ceases because of a developing lack of nutrients or accumulation of toxins. During the phase
of decline (choice A), the lack of nutrients and the accumulation of toxin become so severe that any
viable organisms usually die before they can form spores.
The exponential phase (choices B) and log phase (choice D) are descriptors for active growth
occurring after the lag phase and before the stationary phase. Many antibiotics are most effective in
this period.
The lag phase (choice C) is the initial period of adaptation, prior to growth, which occurs when
organisms are introduced to a new environment.

How well did you know this?
1
Not at all
2
3
4
5
Perfectly
34
Q
  1. On physical examination, a man has several disfiguring lesions on his face and loss of
    cutaneous sensation to fine touch, pain, and temperature. An acid-fast organism is observed
    in scrapings from a skin lesion. Which of the following organisms is the most likely cause of
    this patient’s disease?
    A. Bartonella henselae
    B. Listeria monocytogenes
    C. Mycobacterium avium-intracellulare
    D. Mycobacterium leprae
    E. Nocardia asteroides
A

The correct answer is D. The disease in question is leprosy, or Hansen disease. The key feature in
the description is the fact the organism is acid-fast. These acid-fast bacilli are commonly found in skin
lesions or nasal scrapings. Both of the mycobacteria, M. avium-intracellulare and M. leprae are
strongly acid-fast, that is, they retain the carbol fuchsin dye in the face of acid-alcohol decolorization.
M. leprae has a predilection for the skin and cutaneous nerves, thereby producing the symptoms of
depigmentation and anesthetic cutaneous lesions. This loss of peripheral nerve function leads to
many of the disfiguring features of the disease; because patients do not have normal pain sensation,
they sustain repeated injuries. In addition, the organism attacks cartilage and causes granuloma
formation in the skin, leading to some of the facial disfigurement. Patients often have a history of
residence in an endemic area in childhood.
Bartonella henselae(choice A) is a very small, gram-negative bacterium that is closely related to the
rickettsia, although it is able to be cultured on lifeless media. It is the cause of cat-scratch disease, a
local, chronic lymphadenitis most commonly seen in children. Listeria monocytogenes(choice B) is a
ubiquitous microbe that causes disease in more than 100 animal species. Although it is best known as
an agent of meningitis in the newborn, it is a cause of multiple other diseases. A characteristic feature
of these infections is the development of granulomas at the site of the infection. The organism is not
acid-fast. M. avium-intracellulare(choice C) causes tuberculosis-like pulmonary disease in the
immunosuppressed, primarily in HIV infected individuals. Nocardia asteroides(choice E) primarily
produces pulmonary infections in humans. The organism is considered to be “weakly” acid-fast.

How well did you know this?
1
Not at all
2
3
4
5
Perfectly
35
Q
  1. A 28-year-old HIV-positive man complains of pain on swallowing. Physical examination
    shows white plaque-like material on his tongue and buccal mucosa, which is biopsied. The
    man is diagnosed with acquired immunodeficiency syndrome (AIDS). With which of the
    following agents is the man most likely infected?
    A. Candida albicans
    B. Cytomegalovirus
    C. Herpes simplex I
    D. Human herpesvirus 8
    E. Human papilloma virus
A

The correct answer is A. The most common early form of oral lesions in HIV infected individuals
includes thrush, hairy leukoplakia, and aphthous ulcers. Candida albicans produces oral thrush, an
AIDS-defining lesion, which is common in acute HIV disease, and becomes increasingly common as
the CD4 + cell count falls, especially as the CD4 decreases to less than 300. As the CD4 count
decreases to less than 100, the esophagus, trachea, bronchi, or lungs may be infected. The lesions
are usually painless. Diagnosis is by demonstration of pseudohyphae using a wet smear with
confirmation by culture. Although cytomegalovirus (choice B) is associated with numerous clinical
scenarios in the AIDS population, including odynophagia (painful swallowing), it would not produce
white plaques on the oral mucosa. Herpes simplex I (choice C) produces vesicular lesions occurring
in clusters in the oral cavity. There is an increased risk for herpes infections in the AIDS group, but the
lesions do not resemble those described in the question. These lesions are commonly red and
extremely painful. Human herpesvirus 8 (choice D) is the causative agent of Kaposi sarcoma, a
malignancy arising from endothelial cells that appears as hemorrhagic nodules in different organ
systems. It is the most common cancer in the HIV infected population. Human papilloma virus (choice
E) is associated with a variety of lesions, including warts and intraepithelial neoplasias of the vulva
and cervix. It is associated with anal condyloma, which can occur in the AIDS population.

How well did you know this?
1
Not at all
2
3
4
5
Perfectly
36
Q
  1. A viral organism was isolated from a painful blister on the lip of a teenage girl. The agent
    was found to double-stranded, linear enveloped DNA. The patient had a similar sore
    approximately 2 months ago following a week long trip to the beach. Which of the following is
    the most likely causative organism?
    A. Adenovirus
    B. Coxsackie virus
    C. Herpes simplex type 1 virus
    D. Herpes zoster virus
    E. Papilloma virus
A

The correct answer is C. Herpes simplex often causes recurrent grouped vesicles on an
erythematous base. The Tzanck smear is positive for multi-nucleated epithelial giant cells. Herpes
simplex is an enveloped, linear DNA virus that is a very common infectious agent; most adults will
have anti-Herpes simplex antibodies in their serum, although many may never have had any clinical
signs of disease. The hallmark of this disease is these painful skin vesicles, often called “cold” sores,
or “fever” blisters to denote the precipitating event that preceded the appearance of the lesions. The
virus has a propensity to become latent in the host’s nervous tissue. Activation of the infection occurs
following mild trauma (e.g., a visit to the dentist), hormonal changes (e.g., menses),
immunosuppression (e.g., following organ transplantation), or may follow minor infections, stress or
sun exposure. Other, more serious, manifestations of disease include encephalitis, pneumonia, and
hepatitis; these are particularly likely to be seen in immunodeficient patients such as those with AIDS.
Adenoviruses (choice A) are naked, linear, double-stranded DNA viruses that cause acute, usually
self-limiting, influenza-like illnesses occurring in the fall and winter. The symptoms include pharyngitis,
fever, cough, and general malaise. Epidemic pharyngoconjunctivitis and pneumonia can occur in
closed populations such as military installations.
Coxsackie viruses (choice B) are naked, single-stranded, polycistronic viruses with an RNA genome.
They are divided into groups A and B based on their virulence. Coxsackie A causes herpangina and
hand-foot-and-mouth disease, while Coxsackie B is seen in patients with pleurodynia, myocarditis,
and pericarditis. Both groups cause upper respiratory infections, febrile rashes, and meningitis.
Herpes zoster (choice D), the varicella virus, is an enveloped, double-stranded DNA virus that is a
very common infectious agent in children. Chickenpox is a mild, self-limiting illness in children that is
evidenced as a fever followed by a macular rash that progresses to papules, then vesicles of the skin
and mucous membranes. Shingles is a recurrence of a latent varicella infection in which the virus has
taken refuge in sensory ganglia of spinal or cranial nerves. Various factors that decrease the immune
status of the patient contribute to the exacerbation of the infection. Severe dermatomal pain occurs
with a vesicular eruption, fever, and malaise.
Papilloma viruses (choice E) are members of the Papovavirus family. They are non-enveloped and
possess a double-stranded, circular DNA genome. They cause skin, plantar, and genital warts.

How well did you know this?
1
Not at all
2
3
4
5
Perfectly
37
Q
  1. Electron micrographs of glomeruli reveal prominent deposits between the podocytes and
    the basement membrane of the glomerular capillaries. If the patient was noted to have a skin
    infection before this condition, these findings are most likely related to prior infection with
    which of the following genera?
    A. Escherichia
    B. Klebsiella
    C. Neisseria
    D. Pseudomonas
    E. Streptococcus
A

The correct answer is E. The disease is postinfectious glomerulonephritis, which is usually related to
prior sore throat or skin infection by Streptococcal species; Staphylococcus also causes some cases.
The prominent deposits are also known as subepithelial humps, and consist of immunoglobulin and
complement. Postinfectious glomerulonephritis commonly occurs after impetigo. Onset occurs within
1-3 weeks after infection (average 7-10 days). Other causes of postinfectious glomerulonephritis
include bacteremic states, such as systemic staphylococcus infections.
15
Gram-negative rods such as Escherichia(choice A), Klebsiella (choice B), and Pseudomonas
(choice D) have not been implicated as significant causes of postinfectious glomerulonephritis.
The gram-negative cocci Neisseria(choice C) have not been implicated in postinfectious
glomerulonephritis.

How well did you know this?
1
Not at all
2
3
4
5
Perfectly
38
Q
  1. Which of the following interleukins is produced by macrophages and stimulates fever
    production by its action on hypothalamic cells?
    A. IL-1
    B. IL-2
    C. IL-3
    D. IL-4
    E. IL-5
A

The correct answer is A. IL-1 is produced by macrophages and other antigen-presenting cells. It has
several actions, including stimulation of T cells to secrete IL-2, chemotactic activity for neutrophils and
monocytes, increased expression of intercellular adhesion molecules (ICAMs) on vascular endothelial
cells, and activation of macrophages and natural killer (NK) cells. It is also pyrogenic (fever inducing)
because of a direct effect on the hypothalamus.
IL-2 (choice B) is produced by activated T helper cells. It stimulates the proliferation of other T cells
as well as activated B cells. It also activates NK cells and stimulates lymphokine secretion.
IL-3 (choice C) stimulates all stem cells to produce hematopoietic cells; it is also known as
multilineage colony stimulating factor (CSF).
IL-4 (choice D) is produced by T helper 2 (Th 2) cells and mast cells. It has several functions,
including inducing cells to express class II major histocompatibility complex (MHC) antigens and
stimulation of B-cell proliferation. It is involved in the induction of atopic allergies by its mitogenic
activity for mast cells and its enhancement of immunoglobulin class switching to IgG and IeE.
IL-5 (choice E) is secreted by activated T helper cells. It promotes B-cell proliferation, production of
eosinophils, and stimulates B-cell class switching to IgA.

How well did you know this?
1
Not at all
2
3
4
5
Perfectly
39
Q
  1. A patient received second- and third-degree burns over his body. He later developed a
    wound infection with a bluish-green exudate. Treatment with chloramphenicol and tetracycline
    was unsuccessful. A gram-negative, motile organism was isolated. Which of the following
    organisms was most likely isolated?
    A. Candida albicans
    B. Clostridium perfringens
    C. Escherichia coli
    D. Pseudomonas aeruginosa
A

The correct answer is D. Although this is a clinically-based question, it is essentially asking about
the microbiology of each bacteria. Pseudomonas aeruginosa is a very common opportunist in burn
patients and patients on ventilators, in whom it classically causes secondary wound infections and
septicemia. It may also cause cystitis in patients with urinary catheters and pneumonia in patients with
cystic fibrosis. The organism is found in water and usually gains access to the body as a contaminant
in the water used in respirators or in water baths, etc., used to cleanse wounds. This organism is a
nonfermenter, that is, it does not metabolize sugars by classic pathways. It produces a blue-green,
water-soluble pigment (pyocyanin), and has a fruity odor when grown on laboratory media. It has a
propensity for developing antibiotic resistance; current therapy uses the synergistic combination of an
aminoglycoside, such as amikacin, with a cell wall synthesis inhibitor (carbenicillin, ticarcillin, or
piperacillin).
Candida albicans (choice A) is a normal flora yeast that appears as large, weakly gram-positive,
spherical to ovoid organisms with budding daughter cells in Gram-stained preparations. Candidiasis is
an opportunistic infection in individuals with a compromised immune system. The fungus usually
causes mucocutaneous lesions, but in severely compromised individuals like patients with AIDS,
systemic disease may occur. Oral candidiasis appears as creamy white patches of exudate that can
be scraped off an inflamed tongue or buccal mucosa to reveal a painful lesion that may bleed.
16
Clostridium perfringens(choice B) is a gram-positive, spore-forming, anaerobic rod. It is a common
cause of gas gangrene when introduced into a wound. The organism produces a variety of toxins and
enzymes that enable it to destroy muscle tissue and spread through the soft tissues of the body.
Escherichia coli(choice C) is a lactose-fermenting, gram-negative rod seen as normal flora of the
intestine. It is the most common cause of urinary bladder infections, pyelonephritis, and sepsis in
patients with indwelling urinary catheters. It is also the major cause of traveler’s diarrhea and is a very
important pathogen in neonates, who become infected during passage through the birth canal.

How well did you know this?
1
Not at all
2
3
4
5
Perfectly
40
Q
  1. An 8-year-old girl is bitten in the leg by a neighbor’s cat. She presents the next day with
    fever and bone pain localized to her right calf. An x-ray film reveals a lytic lesion of the right
    tibia. Which of the following is the most likely pathogen?
    A. Brucella melitensis
    B. Eikenella corrodens
    C. Francisella tularensis
    D. Pasteurella multocida
    E. Yersinia pestis
A

The correct answer is D. This patient has osteomyelitis due to a cat bite that penetrated the
periosteum. Whenever you see dog or cat bites in a question stem, consider Pasteurella multocida as
a primary cause of wound infection. This organism is a short, encapsulated, gram-negative rod
demonstrating bipolar staining. Rapidly arising cellulitis is particularly indicative of this organism.
Brucella melitensis (choice A) is a small, acapsular, gram-negative rod that causes brucellosis
(undulant fever) and is associated with contact with goats or sheep. Brucella abortus and Brucella
suis are variants associated with contact with cows and pigs, respectively. The organisms enter the
body through the skin or through contaminated dairy products, such as unpasteurized imported goat’s
milk or cheeses.
Eikenella corrodens(choice B) is commonly found in human bites. It is a gram-negative rod that is
part of the normal flora of the human mouth. Francisella tularensis(choice C) is a small, pleomorphic,
gram-negative rod that causes tularemia (rabbit fever). It occurs most commonly in rural areas. In the
U.S., rabbits are the main reservior for this organism, which is transmitted to humans by the
Dermacentor tick or by contact with infectious animal tissues. Yersinia pestis(choice E) is responsible
for bubonic plague, which has been known to occur in the western U.S. Its main reservoir is the prairie
dog, and its vector is the rat flea.

How well did you know this?
1
Not at all
2
3
4
5
Perfectly
41
Q
  1. A 27-year-old IV drug user presents with difficulty swallowing. Examination of the
    oropharynx reveals white plaques along the tongue and the oral mucosa. Which of the
    following best describes the microscopic appearance of the microorganism responsible for
    this patient’s illness?
    A. Budding yeast and pseudohyphae
    B. Encapsulated yeast
    C. Mold with nonseptate hyphae
    D. Mold with septate hyphae
A

The correct answer is choice A. The patient has Candida esophagitis. Any time a patient presents
with dysphagia or odontophagia, along with whilte plaques in the oropharynx (thrush), you can
assume that the Candida is affecting the esophagus as well. The fact that the patient is an IV drug
user make an opportunistic infection such as Candida more likely. Candida appears as budding yeast
with pseudohyphae in vivo. The other answer choices represent the morphology of other important
opportunistic fungi: Cryptococcus are encapsulated yeast (choice B). You should think about
Cryptococcus neoformans when you’re presented with an immunocompromised patient with
neurologic symptoms. The classic clue is the presence of encapsulated organisms observable in an
India ink preparation. Mucor and Rhizopus are molds with nonseptate hyphae (choice C). You should
think about Mucor when you are presented with a diabetic (especially ketoacidotic) or a leukemic
patient with a severe sinus infection. Aspergillus is a mold with septate hyphae (choice D). In
immunocompromised patients, aspergillosis can present with acute pneumonia, often with a cavitation
(aspergillomas = fungus balls in the lungs).

How well did you know this?
1
Not at all
2
3
4
5
Perfectly
42
Q
  1. A 32-year-old woman with increased frequency of urination, suprapubic pain, and dysuria
    for the past 3 days comes to the emergency department. She has no fever, nausea, or
    vomiting. A Gram stain reveals gram-negative rods. Which of the following is the most likely
    pathogen?
    A. Escherichia coli
    B. Neisseria gonorrhoeae
    C. Shigella dysenteriae
    D. Streptococcus pneumoniae
    E. Treponema pallidum
A

The correct answer is A. This patient has the symptoms of a urinary tract infection (UTI).
Escherichia coli is the leading cause of community-acquired UTIs. The proximity of the urinary tract to
the anus facilitates colonization of the tract by fecal flora. Other gram-negative rods causing UTIs
include Enterobacter cloacae, Klebsiella pneumoniae, Serratia marcescens, Proteus mirabilis, and
Pseudomonas aeruginosa. None of the other choices listed cause UTIs.

How well did you know this?
1
Not at all
2
3
4
5
Perfectly
43
Q
  1. A 24-year-old AIDS patient develops chronic abdominal pain, low-grade fever, diarrhea, and
    malabsorption. Oocysts are demonstrated in the stool. Which of the following organisms is
    most likely to be the cause of the patient’s diarrhea?
    A. Chlamydia psittaci
    B. Entamoeba histolytica
    C. Giardia lamblia
    D. Isospora belli
    E. Microsporidia
A

The correct answer is D. All of the organisms listed are protozoa. There are two intestinal protozoa
specifically associated with AIDS that can cause transient diarrhea in immunocompetent individuals
but can cause debilitating and potentially life-threatening chronic diarrhea in AIDS patients. These
organisms are Isospora belli, treated with trimethoprim-sulfamethoxazole (or other folate antagonists)
and Cryptosporidium parvum (no treatment presently available).
Chlamydia psittaci (choice A) is associated with the development of fever, chills, cough, and atypical
pneumonia with slightly delayed appearance of signs of pneumonitis. Entamoeba histolytica (choice
B) and Giardia lamblia(choice C) are both causes of diarrhea, but they are not specifically associated
with AIDS. Giardiasis, for example, can cause mild to severe bulky, greasy, frothy, malodorous stools,
free of blood and pus. Microsporidia (choice E) are a protozoan cause of diarrhea, but produce
spores rather than oocysts.

How well did you know this?
1
Not at all
2
3
4
5
Perfectly
44
Q
  1. A middle-aged man presents with fever, anorexia, and jaundice. He is diagnosed with
    hepatitis caused by an enveloped DNA virus. What is the most likely causative organism?
    A. Hepatitis A
    B. Hepatitis B
    C. Hepatitis C
    D. Hepatitis E
A

The correct answer is B. Hepatitis B is a hepadnavirus, which is enveloped DNA. Hepatitis A
(choice A) is a picornavirus and is a naked-capsid RNA. Hepatitis C (choice C) is a flavivirus, which
is RNA enveloped. Hepatitis E (choice D) is a Calicivirus, which is a naked-capsid RNA.

How well did you know this?
1
Not at all
2
3
4
5
Perfectly
45
Q
45. A 6 month-old child who has been breast-fed since birth develops voluminous, non-bloody,
watery diarrhea and vomiting. Which of the following viruses is the most likely cause of the
child's diarrhea?
A. Coronavirus
B. Lymphocytic choriomeningitis virus
C. Norwalk agent
D. Orbivirus
E. Rotavirus
A

The correct answer is E. Rotavirus is the major cause of diarrhea in infants and children under the
age of 2. It is a major cause of diarrheal morbidity worldwide due to dehydration. The virus replicates
in the intestinal mucosa, producing a profuse, watery, non-bloody diarrhea, often coupled with nausea
and vomiting. Transmission is by the fecal-oral route. Treatment is symptomatic with fluid and
electrolyte replacement. Coronaviruses (choice A) usually cause cold-like illnesses. Lymphocytic
choriomeningitis virus (choice B) can cause headache, malaise, myalgia, conjunctivitis, and,
occasionally, meningitis. Norwalk agent (choice C) can also cause diarrhea, but usually affects
patients older than 2 years. This causes profuse vomiting and diarrhea and is often transmitted by
food, especially shellfish. Orbivirus (choice D) is the cause of Colorado tick fever, which is the only
tick-borne viral disease in the United States.

How well did you know this?
1
Not at all
2
3
4
5
Perfectly
46
Q
  1. With respect to the bacterial spectrum of coverage, which of the following antibiotics has
    the broadest activity against gram-negative organisms?
    A. Cefuroxime (second generation cephalosporin)
    B. Cephalexin (first generation cephalosporin)
    C. Ceftriaxone (third generation cephalosporin)
    D. Metronidazole
    E. Vancomycin
A

The correct answer is C. For NBDE Part 1, it is important to know general properties of antibiotic
families. Ceftriaxone is a third generation cephalosporin commonly used in the treatment of a variety
of life-threatening infections caused by gram-negative organisms. The agent has moderate grampositive
activity. A general rule to remember about the cephalosporins is that when progressing from
first generation to third generation, the gram-negative activity is increased and gram-positive activity is
decreased. In other words, first generation cephalosporins have the most activity against grampositive
organisms and third generation cephalosporins have the most activity against gram-negative
organisms. Metronidazole (choice D) is an antibiotic with excellent anaerobic bacterial coverage.
Vancomycin (choice E) is an antibiotic with excellent gram-positive coverage; it is generally used in
the treatment of life-threatening infections.

How well did you know this?
1
Not at all
2
3
4
5
Perfectly
47
Q
47. If a newborn child develops an infection with Chlamydia trachomatis, how does infection
with this organism cause blindness?
A. Cataract formation
B. Hemorrhage into the anterior chamber
C. Hemorrhage into the posterior chamber
D. Retinal detachment
E. Scarring of the cornea
A

The correct answer is E.Chlamydiae are obligate intracellular parasites. Studies in the United States
demonstrate that 5-25% of pregnant women have C. trachomatis infections of the cervix. In these
women, approximately one half to two thirds of infants are exposed and if untreated can develop
inclusion conjunctivitis. The lesions begin with formation of lymphoid follicles in the conjunctiva. With
disease progression, there is tissue necrosis, granulation tissue deposition, and scar formation,
leading to lacrimal duct obstruction and distortion of the eyelids. With the loss of an adequate tear
system, the cornea becomes vulnerable to dehydration and opacification. Also, the vigorous
inflammatory response can directly involve the cornea, with resulting opacity.

How well did you know this?
1
Not at all
2
3
4
5
Perfectly
48
Q
49. Which of the following is a feature of gram-positive bacteria rather than gram-negative
bacteria?
A. Lipid A-containing lipopolysaccharide
B. Lipoprotein in periplasmic space
C. Outer membrane
D. Peptidoglycan in periplasmic space
E. Thick peptidoglycan cell wall
A

The correct answer is E. Most of the features listed are those of gram-negative bacteria, which have
a complex cell envelope consisting of a cytoplasmic (inner) membrane, a periplasmic space
containing peptidoglycan (choice D) and lipoprotein (choice B), an outer membrane (choice C), and
sometimes a capsule. The outer membrane contains lipopolysaccharide (choice A) which is a major
component of endotoxin. The peptidoglycan cell wall of the gram-negative bacteria is thin, while that
of the gram-positive bacteria is thick. Other features of gram- positive bacteria include a fairly simple
surface of cytoplasmic membrane, peptidoglycan, cell wall, and sometimes, an outer capsule. The cell
wall contains lipoteichoic acids. Gram-positive bacteria retain the stain or resist decolorizationby
alcohol in Gram’s method of staining. This is primarily characteristic of bacteria whose cell wall is
composed of peptidoglycan or teichoic acid.

How well did you know this?
1
Not at all
2
3
4
5
Perfectly
49
Q
  1. A diabetic patient has chronic sinusitis, which is eventually treated by evacuation of the
    contents of the maxillary and ethmoid sinuses. Mucor species are found when the material is
    examined histologically. The pathologist should notify the clinician immediately because
    Mucor can cause a virulent
    A. meningitis
    B. pneumonia
    C. septicemia
    D. skin infection
    E. urinary tract infection
A

The correct answer is choice A. Mucormycosis, to which diabetics are particularly vulnerable
(perhaps because of the high glucose content of the diabetic nasal secretions), typically involves the
nasal sinuses. It is especially dangerous because the organism can erode into the bones of the
cranium, causing life-threatening meningitis and/or encephalitis that is very difficult to treat. Mucor can
cause pneumonia (choice B), but this is not the most immediate threat. Mucor can cause septicemia
(choice C), but this usually occurs in more seriously ill patients and is not an immediate risk in this
patient. Mucor can cause a skin infection (choice D), but this generally occurs in conjuncition with
severe burns. Mucor does not usually cause urinary tract infections (choice E).

How well did you know this?
1
Not at all
2
3
4
5
Perfectly
50
Q
51. In trying to sterilize some dental materials, the dentist notices that one of the materials
melts at 115 C. Which of the following sterilization methods is most appropriate for this
material?
A. Steam Autoclave
B. Dry heat autoclave
C. Glutaraldehyde (Cidex) soak (6 hours)
D. Ethylene oxide
E. Ethyl alcohol
A

The correct answer is choice D. Note firstly that ethyl alcohol and glutaraldehyde are chemicals that
can only high level disinfect, and cannot be used for sterilization. In particular, they are not sporicidal.
Choices A, B and D are the three most common methods of instrument sterilization. However, the
steam autoclave operates at 121 C, and the dry heat sterilizer at 160 C, both too hot for the material
which melts at 115 C. So we can only use a non-heat sterilization method. The only non-heat method
is ethylene oxide, at 8-12 hours of contact time.

How well did you know this?
1
Not at all
2
3
4
5
Perfectly
51
Q
  1. Global eradication of Lyme disease is unlikely because
    A. Borrelia burgdorferi can be maintained in nature indefinitely by a tick vector
    B. Borrelia burgdorferi is resistant to antibiotics and disinfectants
    C. Borrelia burgdorferi is resistant to environmental stresses
    D. human disease may reactivate after the primary infection (Brill-Zinsser disease)
    E. humans are the primary reservoir for Borrelia burgdorferi
A

The correct answer is A. Borrelia burgdorferi can be maintained in nature indefinitely by a tick
vector. This organism is the tick-transmitted spirochete that causes Lyme disease. The tick, Ixodes
dammini, can infect the white-footed mouse and large mammals such as deer during its life cycle,
making them reservoirs. The tick itself is a reservoir, however, because it acquires the disease
through transovarial passage of the organism. Together, these factors make Lyme disease an
endemic infection with little hope for eradication. Lyme disease is characterized by erythema migrus,
a flat or slightly raised red lesion that expands with central clearing. Headache or stiff neck are
common. Arthritis is common, chronic, and recurrent. Borrelia burgdorferi is not resistant to antibiotics
and disinfectants (compare with choice B). The spirochete can be successfully treated with
penicillins, tetracycline, and ceftriaxone. Borrelia burgdorferi is a delicate spirochete and is not
resistant to environmental stresses (compare with choice C). Brill-Zinsser disease (choice D) is the
reactivation of epidemic typhus infection caused by Rickettsia prowazekii. It can occur many years
after an infection that was not treated with antibiotics. Humans are incidental hosts, rather than the
primary reservoir, for Borrelia burgdorferi(choice E). The primary reservoirs are ticks, mice, and large
mammals.

How well did you know this?
1
Not at all
2
3
4
5
Perfectly
52
Q
  1. A chef cut his finger. After a week, the site of the injury is warm, red, and swollen, and
    begins draining pus. He contaminates some pastries with drainage from the lesion which later
    were eaten by patrons of the restaurant. Within 4 hours they developed diarrhea and vomiting
    with no fever. Which of the following organisms would be most likely to cause these
    symptoms?
    A. Bacillus cereus
    B. Clostridium perfringens
    C. Escherichia coli
    D. Shigella sonnei
    E. Staphylococcus aureus
A

The correct answer is E. Cellulitis is an acute infection involving the epidermis and dermis. Cellulitis
can lead to the development of an abscess, as seen in this patient. The most common types of skin
infections (cellulitis or abscess) usually involve gram-positive organisms such as staphylococcus and
streptococci. The chef had a staphylococcal abscess on his finger. S. aureus, produces enterotoxin A,
which was likely transferred to the pastries. When ingested, the toxin causes severe nausea and
vomiting within a few hours (the average incubation time is 3-6 hours). There is little diarrhea
associated with this type of food poisoning outbreak.
Bacillus cereus(choice A) is a gram-positive spore-forming rod that is associated with food poisoning
outbreaks following the ingestion of fried rice. The time of onset and symptoms would mimic
staphylococcal disease; the major differentiating feature is the food involved. The organism survives
the boiling of the rice because it is a spore-former. It germinates as the rice cools, grows, and
elaborates an enterotoxin that is responsible for the nausea and vomiting characteristic of the
disease. Clostridium perfringens(choice B) is a gram-positive spore-forming anaerobe that can cause
a longer incubation (18-24 hour) food poisoning, typically with marked diarrhea. Once again, the
spores allow the organism to survive the heating process used in the preparation of the food. Both
Clostridium perfringens and Clostridium botulinum are associated with home-canned vegetable and
sausages.
21
The symptoms of Escherichia coli(choice C) food poisoning are usually watery diarrhea (traveler’s
diarrhea) with minimal nausea and vomiting, or a bloody diarrhea caused by enteroinvasive strains of
the agent. Also, E. coli would be an unlikely cause of the primary infection in the chef. Shigella sonnei
(choice D) causes enterocolitis characterized by fever, cramps, and diarrhea after an incubation
period of one to four days. Transmission is fecal-oral, associated with poor hygiene. A wide range of
foods has been implicated.

How well did you know this?
1
Not at all
2
3
4
5
Perfectly
53
Q
  1. A Native American man is brought to a rural hospital in New Mexico with severe
    bronchopneumonia, chills, fever, and headache. One day later, the man complains of chest
    pain and difficulty breathing, and coughs up blood-tinged sputum. Chest x-ray reveals patchy
    infiltrates and segmental consolidation. Which of the following organisms is the most likely
    cause of this man’s pneumonia?
    A. Poliomyelitis virus
    B. Clostridium perfringens
    C. Paramyxovirus
    D. Listeria monocytogenes
    E. Yersinia pestis
A

The correct answer is E. Any previously healthy person in the southwestern United States who
develops septic shock or severe pulmonary disease should be evaluated for plague. Plague is not an
extinct disease, but is still encountered in sporadic cases in various places, including Asia, Africa,
parts of Europe, and the American Southwest. The causative organism is Yersinia pestis, which is
endemic in many wild animal populations, and can be transmitted to humans either by direct contact
or by arthropod bite. Human plague may take many forms, including pestis minor (mild
lymphadenopathy); bubonic plague (prominent lymphadenopathy); pneumonic plague (as described
above); and septicemic plague. Antibiotics are most effective if given within the first 24 hours, which
can be problematic if medical staff do not suspect the disease. Because plague is rare in the United
States, a high degree of clinical suspicion is required to make a rapid diagnosis and to institute timely
treatment. If the diagnosis is missed, the mortality rate is high.
Poliomyelitis virus (choice A) is an enterovirus that causes muscle weakness, headache, stiff neck,
sore throat, fever, nausea, and vomiting. Lower motor neuron lesions (flaccid paralysis) can be seen
as well as deep tendon reflexes and muscle wasting. Clostridium perfringens(choice B) causes gas
gangrene and gastroenteritis. This is the most common cause of gangrene. Paramyxovirus (choice
C) is the causative organism of the mumps. In this condition, exposure to paramyxovirus occurs 14-21
days before onset of symptoms. Patients present with painful swollen salivary glands, usually the
parotids. Frequent involvement of the other tissues is common, such as the testes, pancreas, and
meninges. Listeria monocytogenes(choice D) causes listeriosis. Infection during pregnancy may
result in sepsis, abortion, or premature delivery. Infection in the neonate may produce meningitis. In
immunocompromised adults, either meningitis or sepsis may occur.

How well did you know this?
1
Not at all
2
3
4
5
Perfectly
54
Q
  1. A patient with large, penetrating vegetations on his mitral and aortic valves develops
    severe headaches. Acute bacterial endocarditis is diagnosed. Which of the following
    organisms is the most likely cause of the patient’s disorder?
    A. Herpesvirus
    B. Mycobacterium tuberculosis
    C. Staphylococcus aureus
    D. Streptococcus pneumoniae
    E. Treponema pallidum
A
The correct answer is C. The cardiac lesions described are characteristic of acute bacterial
endocarditis. Intravenous drug users are among those susceptible to acute endocarditis. The fact the
valvular vegetations are large and penetrating strongly suggests that they are caused by a virulent,
pyogenic pathogen. Staphylococcus aureus is the most common cause of acute bacterial
endocarditis. Brain abscess, which produces ring-enhancing radiologic lesions, is a known
complication of bacterial endocarditis, occurring when the vegetations fragment and release septic
emboli. The essential diagnostic characteristic for infective endocarditis includes a preexisting organic
heart lesion, fever, new or changing heart murmur, evidence of systemic emboli, positive blood
culture, and evidence of vegetation on endocardiography.
22
Herpesvirus (choice A) can cause encephalitis, but is not a cause of vegetative endocarditis.
Mycobacterium tuberculosis(choice B) can cause brain masses, but does not usually cause
endocarditis. Streptococcus pneumoniae(choice D) can cause acute pyogenic meningitis, but does
not usually cause endocarditis. Treponema pallidum(choice E), the causative agent of syphilis, can
cause aortic aneurysms and a variety of neurologic problems, but does not usually cause endocarditis
or brain abscess.
How well did you know this?
1
Not at all
2
3
4
5
Perfectly
55
Q
  1. A can of disinfectant spray states that it kills HIV virus. This statement:
    A. is important because of the difficulty in killing this virus
    B. shows that the disinfectant will kill most other pathogens
    C. illustrates the use of a “benchmark” organism
    D. is not a good indication of disinfectant strength
A

The correct answer is D. HIV virus is notoriously easy to kill on most environmental surfaces. Many
disinfectants and other common physical and chemical exposures will kill the virus fairly easily.
Therefore, ability to kill HIV is NOT an important criterion in choosing disinfectants. Mycobacterium
tuberculosis is chosen as the “benchmark organism” (the standard against which the disinfectant is
compared). It is very resistant to surface disinfectants, partially because of its waxy cell wall. Another
highly resistant pathogen on environmental surfaces is Hepatitis A virus.

How well did you know this?
1
Not at all
2
3
4
5
Perfectly
56
Q
  1. A 35-year-old, sexually active man presents with a painless penile vesicle and inguinal
    lymphadenopathy. The infecting organism is definitively diagnosed and is known to exist in
    distinct extracellular and intracellular forms. Which of the following is the most likely
    pathogen?
    A. Calymmatobacterium granulomatis
    B. Chlamydia trachomatis
    C. Haemophilus ducreyi
    D. Neisseria gonorrhoeae
    E. Treponema pallidum
A

The correct answer is choice B. This patient has lymphogranuloma venereum caused by Chlamydia
trachomatis (type L1, 2, or 3). Chlamydia exhibit distinct infectious and reproductive forms. The
extracellular infectious form is known as the elementarly body (EB), which cannot reproduce. It
attaches to the host cell and enters through endocytosis. Once inside the cell, the EB is transformed
into the reticulate body (RB) within the endosome. The RB is capable of binary fission and divides
within the endosome; fusion with other endosomes occurs to form a single large inclusion. Eventually,
the RBs undergo DNA condensation and disulfide bond bridgings of the major outer membrane
protein, forming EBs. The EBs are then released. Note that C. trachomatis is responsible for several
sexually or perinatally transmitted diseases, including ocular trachoma (types A, B and C), neonatal
conjunctivitis, nongonococcal urethritis, cervicitis, and pelvic inflammatory disease (types D-K).
Calymmatobacterium granulomatis (choice A) is a gram-negative rod that causes superficially
ulcerated genital or inguinal papules that coalesce to form substantial lesions. The appearance of
Donovan bodies in histiocytes is diagnostic of this infection. Haemophilus ducreyi (choice C) is a
gram-negative rod that causes a soft, painful penile chancre, unlike that of a chlamydial or syphilitic
lesion. This infection is common in the tropics. Neisseria gonorrhoeae (choice D) is a gram-negative
diplococcus responsible for gonorrhea. Patients typically present with purulent penile discharge, not
genital lesion. Treponema pallidum (choice E) is the spirochete responsible for syphilis. It may cause
a firm, painless ulcer as a manifestation of primary syphilis, but the organism does not exist in distinct
extracellular and intracellular forms as does Chlamydia. Secondary syphilis is associated with the
appearance of condyloma lata – flat, gray wart-like lesions.

How well did you know this?
1
Not at all
2
3
4
5
Perfectly
57
Q
  1. A 23-year-old man develops explosive watery diarrhea with blood, fecal leukocytes, and
    mucus approximately 3 days after eating chicken that was improperly cooked. Comma-shaped
    organisms were also found in the fecal smear. Which of the following pathogens is the most
    likely cause of these symptoms?
    A. Campylobacter jejuni
    B. Enterotoxigenic E. coli
    C. Shigella sonnei
    D. Staphylococcus aureus
    E. Vibrio cholera
A

The correct answer is A. This is another question in which knowing the microbiological
characteristics is essential. Campylobacter jejuni is a pathogen causing an invasive enteric infection
associated with ingestion of raw or undercooked food products, or through direct contact with infected
animals. In the United States, ingestion of contaminated poultry that has not been sufficiently cooked
is the most common means of acquiring the infection. The patients typically have bloody diarrhea,
abdominal pain, and fever. The presence of fecal leukocytes indicates an invasive infection.
Campylobacter are microaerophilic, motile, gram-negative “comma-shaped” rods.E. coliare not
comma-shaped.
Enterotoxigenic E. coli(choice B) causes the classic traveler’s diarrhea. The infection is noninvasive
and is acquired by way of the fecal-oral route through consumption of unbottled water or uncooked
vegetables. The major manifestation is a copious outpouring of fluid from the gastrointestinal tract
presenting as explosive diarrhea. This is caused by the action of one of two types of enterotoxins on
the gastrointestinal tract mucosa. Shigella sonnei(choice C) produces a syndrome very similar to C.
jejuni. The organism, however, appears as a gram-negative rod on Gram’s stain. It does not have a
comma shape. Transmission is from person to person by way of the fecal-oral route. Infection
requires a low infective dose because the organism is fairly resistant to gastric acidity.
Staphylococcus aureus(choice D) produces food poisoning through the ingestion of a preformed
enterotoxin. The organism is present in food that is high in salt content, such as potato salad, custard,
milk shakes, and mayonnaise. The patient presents with nausea, vomiting, and abdominal pain,
followed by diarrhea beginning 1-6 hours after ingestion of the enterotoxin.S. aureus are seen as
clusters of coli and are not comma-shaped. Vibrio cholerae(choice E) produces a secretory diarrhea
caused by increases in cAMP in the intestinal cells. The organism is comma-shaped, but is not
invasive. The patient presents with the sudden onset of painless, watery diarrhea that becomes
voluminous, followed by vomiting. The stool appears nonbilious, gray, and slightly cloudy with flecks
of mucus, no blood, and a sweet odor.

How well did you know this?
1
Not at all
2
3
4
5
Perfectly
58
Q
  1. A 3-year-old boy presents with a 1-day history of loose stools, fever, abdominal cramping,
    headache, and myalgia. He has no blood in the stool. A careful history reveals that he has had
    several pet turtles. Which of the following is the most likely pathogen?
    A. Chlamydia psittaci
    B. Entamoeba histolytica
    C. Salmonella spp.
    D. Staphylococcus aureus
    E. Yersinia enterocolitica
A

The correct answer is C. Salmonella spp., including S. enteriditis and S. typhimurium, produce a
gastroenteritis or enterocolitis. Patients with decreased gastric acidity, sickle cell disease, or defects in
immunity, and children younger than 4 years of age have a more severe course of disease.
Salmonella spp. are carried in nature by animal reservoirs such as poultry, turtles, cattle, pigs, and
sheep. The incubation period is 8-48 hours after ingestion of contaminated food or water. Chlamydia
psittaci (choice A) produces an interstitial pneumonitis accompanied by headache, backache, and a
dry, hacking cough. A pale, macular rash is also found on the trunk (Horder spots). Patients at risk
include pet shop workers, pigeon handlers, and poultry workers. Entamoeba histolytica (choice B)
produces a diarrhea (frequently bloody or heme-positive), right lower quadrant crampy abdominal
pain, and fever. Patients frequently have weight loss and anorexia. There is usually a history of travel
outside the United States. Most cases are chronic. Complications include liver abscesses.
Staphylococcus aureus (choice D) produces a self-limited gastroenteritis caused by the production of
preformed, heat-stable enterotoxins. The incubation period is 16 hours. The toxins enhance intestinal
peristalsis and induce vomiting by a direct effect on the CNS. Yersinia enterocolitica (choice E)
usually produces a chronic enteritis in children. These patients have diarrhea, failure to thrive,
hypoalbuminemia, and hypokalemia. Other findings include acute right lower quadrant abdominal
pain, tenderness, nausea, and vomiting. The infection mimics appendicitis or Crohn disease.

How well did you know this?
1
Not at all
2
3
4
5
Perfectly
59
Q
  1. A 48-year-old presents with malaise, loss of appetite, nausea, moderate fever, and
    jaundice. Laboratory tests indicate a marked increase in serum transaminases and presence
    of HBsAg, HBc IgM antibody, and HCV antibody. Antibody tests for HBsAb and HAV are
    negative. The results indicate:
    A. A dual infection of HBV and HAV
    B. Chronic hepatitis A infection
    C. Chronic hepatitis B infection
    D. Hepatitis C infection
    E. The presence of an acute HBV infection.
A

The correct answer is E.The presence of hepatitis B surface antigen (HBsAg) along with hepatitis B
core IgM antibody (HBc IgM Ab), and the absence of hepatitis B surface antibody (HBsAb) indicates
the presence of the early stages of an acute infection with Hepatitis B. The presence of antibody to
Hepatitis C (HCV) only indicates exposure, but not a specific time of exposure; however, 85% of
patients who are infected with HCV develop chronic infections, indicating that this patient has an 85%
chance of having a dual infection with HBV and HCV. A dual infection of HBV and HAV (choice A) is
not plausible since the IgM anti-HAV serology is negative. Hepatitis A does not cause chronic disease
(choice B). Chronic HBV infection (choice C) is unlikely because the patient has HBc IgM Ab, which
is characteristic of an acute infection, rather than a chronic infection. Hepatitis C infection (choice D)
is not confirmed by these data because the presence of HCV Ab only indicates exposure to the virus,
and not the state of infection. This could be caused by exposure at some earlier time (the elevated
serum transaminases might be due to HBV infection). An active or chronic HCV infection can only be
confirmed by PCR.

How well did you know this?
1
Not at all
2
3
4
5
Perfectly
60
Q
  1. A 47-year-old man presents with a nonproductive cough and rales. A chest x-ray film
    suggests atypical pneumonia. The pneumonia resolves after treatment with azithromycin. A
    diagnosis of psittacosis is established by the presence of complement fixing antibodies
    against Chlamydia psittaci in the convalescent serum. Which of the following is the most likely
    occupation of this patient?
    A. Cat breeder
    B. Florist
    C. Homeless shelter worker
    D. Poultry farmer
    E. Slaughterhouse worker
A

The correct answer is D. When you see Chlamydia psittaci, one word should come to mind: birds!
Infection with this organism is an occupational hazard for anyone who works with birds (e.g.,
veterinarians, pet store employees), including poultry farmers. Individuals who work with cats (choice
A) would be at an increased risk for infection with Pasteurella multocida (which is acquired primarily
through cat bites), Bartonella henselae (cat-scratch fever), and the protozoa Toxoplasma gondii
(which can be acquired from ingestion of food contaminated with cat feces). Florists (choice B) are at
increased risk for infection with the fungus Sporthrix schenckii, a primary pathogenic fungus acquired
by inoculation (e.g., a rose thorn puncturing the skin). A person who works in a homeless shelter
(choice C) would be at increased risk for infection with Mycobacterium tuberculosis, because this
organism is spread through the air and is prevalent in the homeless community. Slaughterhouse
workers (choice E) are at an increased risk for infection with Brucella, a bacterium that is acquired by
handling infected animals.

How well did you know this?
1
Not at all
2
3
4
5
Perfectly
61
Q
  1. A 3-year-old child with cystic fibrosis presents with weight loss, irritability, and a chronic
    productive cough. On physical examination, he is febrile and wheezing with rhonchi and rales.
    Chest x-ray demonstrates patchy infiltrates and atelectasis, and Gram stain of the sputum
    reveals slightly curved, motile, gram-negative rods that grow aerobically. The microorganism
    responsible for this child’s pneumonia is also the most common cause of which of the
    following diseases?
    A. Croup
    B. Epiglottitis
    C. Meningitis
    D. Otitis externa
    E. Otitis media
A

The correct answer is D. This is an example of a two-step question. Step 1: what is the organism
that causes pneumonia in cystic fibrosis? Then, this organism is commonly seen in cases of…?
Anytime you see pneumonia in a cystic fibrosis patient you should suspect Pseudomonas aeruginosa.
The Gram stain revealing aerobic, gram-negative rods confirms your suspicion in this case. Now the
question is: which of the diseases listed is also caused by Pseudomonas? The answer is otitis
externa. P. aeruginosa is often found in the external ear, especially if the conditions are moist
(“swimmer’s ear”) and there is any sort of inflammation. External otitis is usually a benign process with
the only symptoms being an itchy, painful ear. If, however, the organism penetrates the epithelium
and invades the soft tissue, cartilage, and cortical bone, the process becomes malignant otitis
25
externa, which can progress to osteomyelitis leading to cranial nerve palsies. This condition is most
common in diabetics. Croup (choice A), also called laryngotracheobronchitis, is a respiratory disease
of children that presents with a characteristic “barking” cough. Croup is caused by parainfluenza virus.
Epiglottitis (choice B) is a potentially fatal infection in children, caused by H. influenzae, which
presents with drooling, difficulty breathing, and stridor. The incidence of this disease has dropped
dramatically with the introduction of the H. influenzae type b (Hib) vaccine. Meningitis (choice C) is
caused by numerous different bacteria, depending on the age of the patient. The most common
causes include S. pneumoniae (elderly patients), H. influenzae (unvaccinated children), Group B strep
and E. coli (neonates) and N. meningitidis (1 month to adult). Although P. aeruginosa can cause
meningitis, it is not a common cause. The most common causes of otitis media (choice E) include S.
pneumoniae and H. influenzae. Even in cases of external ear infections with P. aeruginosa, the
middle ear is typically spared.

62
Q
63. A combined chemical/heat autoclave (Chemiclave) uses what set of chemicals/conditions?
A. Alcohol at 121 C
B. Ethylene oxide at 160 C
C. Alcohol/formaldehyde at 132 C
D. Glutaraldehyde at 121 C
E. Glutaraldehyde/formaldehyde at 160 C
A

The correct answer is C. Besides the steam autoclave, dry heat autoclave, and ethylene oxide
sterilizer, there is a heat/chemical combination known as a Chemiclave or heat/chemical sterilizer. Its
conditions are formaldehyde and alcohol at a temperature of 132 C for periods of 20-30 minutes at
20-40 psi (pounds per square inch). The chemical mixture is provided by the manufacturer and is recollected
for disposal.

63
Q
  1. A 10-year-old boy develops an itchy, vesicular rash, which is maximal on his face and
    trunk. Physical examination demonstrates a mixture of lesions, with macules, papules,
    vesicles, and crusted lesions. The mother reports that the lesions seem to be occurring in
    crops. Which of the following is the most likely diagnosis?
    A. Herpes simplex I
    B. Herpes simplex II
    C. Measles
    D. Shingles
    E. Varicella
A

The correct answer is E. This is varicella (chicken pox), which is the primary form of infection by the
herpes zoster (varicella-zoster) virus. Recurrence due to virus harbored in neurons tends to be
dermatomal in distribution and is called shingles. Fever, malaise, headache, and myalgia may also be
present, particularly in the prodromal phase. The rash is pruritic, papular, changing to vesicular (“dew
drops on a rose petal”), pustular and finally crusting. Tzanck smear of the base of a vesicle may
demonstrate multinucleated giant cells. Immunocompromised patients can be treated with acyclovir to
prevent dissemination. Chicken pox may be complicated by secondary bacterial infection, pneumonia
and systemic spread (immunosuppressed patients). Herpes simplex I (choice A) causes oral vesicles
and ulcers. The lesions are often very painful. A rash is not seen with herpes simplex I. Herpes
simplex II (choice B) causes genital vesicles and ulcers. The lesions are described as being beefy
red and very painful. Measles (choice C) causes a blotchy, nonvesicular rash. Prodrome consists of
fever, coryza, malaise, photophobia and Koplik’s spots. The rash is brick red, irregular and
maculopapular. Shingles (choice D) is the recurrent form of herpes zoster infection and usually is
localized to a single dermatome. It is typically seen in the elderly.

64
Q
65. Which of the following organisms is the most common cause of community-acquired
pneumonia?
A. Chlamydia pneumoniae
B. Haemophilus influenzae
C. Mycoplasma pneumoniae
D. Staphylococcus aureus
E. Streptococcus pneumoniae
A

The correct answer is E. The most common bacterium implicated in community-acquired pneumonia
is the pneumococcus Streptococcus pneumoniae. When community-acquired pneumonia occurs in
elderly patients or patients with comorbidity, aerobic gram-negative bacilli and Staphylococcus aureus
are added to the list. The organisms listed in choices A, B, and C are important causes of communityacquired
pneumonia without comorbidity in patients younger than 60 years of age, but are not the
most frequent causes. Staphylococcus aureus (choice D) is an important cause of communityacquired
pneumonia (particularly in the elderly and in patients with comorbidity) but is not the most
frequent cause.

65
Q
  1. A 21-year-old man presents with cough, fever, and hemoptysis. Blood tests show
    significantly elevated BUN and creatinine. Immunofluorescent microscopy reveals a diffuse
    linear pattern of fluorescence along the basement membranes of alveolar septa and
    glomerular capillaries. Which type of hypersensitivity is associated with this disease?
    A. I
    B. II
    C. III
    D. IV
A

The correct answer is B. This patient has Goodpasture syndrome, which affects both the renal and
pulmonary systems. In the kidney, it causes a rapidly progressive glomerulonephritis associated with
antibodies directed against a collagen component of the glomerular basement membrane (anti-GBM
antibody a classic clue to this diagnosis). They are also active against the basement membrane of
respiratory alveoli, accounting for the pulmonary component of the disease. These antibodies create a
linear pattern on immunofluorescence.
Autoimmune reactions such as those found in Goodpasture syndrome, certain drug allergies, blood
transfusion reactions, and hemolytic disease of the newborn, are classified as Type II
hypersensitivities (antibody-mediated cytotoxicity). IgG or IgM antibody reacts with membraneassociated
antigen on the surface of cells, causing activation of the complement cascade and,
ultimately, cell destruction.
Type I (choice A) reactions (immediate, atopic, or anaphylactic) require an initial (sensitizing)
exposure to an antigen. On re-exposure to the antigen, cross-linking of IgE receptors occurs on the
surface of basophils and mast cells. The mast cells then release a variety of mediators, including
histamine. Clinical syndromes include asthma, atopic dermatitis, eczema, hives, and allergic rhinitis.
Type III (choice C) hypersensitivity (immune complex-mediated hypersensitivity) is caused by
antibodies to foreign antigens. Immune complexes of IgG or IgM with the antigen activate
complement. This results in the generation of C3b, which promotes neutrophil adherence to blood
vessel walls. The complexes also generate C3a and C5a (anaphylatoxins), which lead to inflammation
and tissue destruction. The hallmark signs of Type III sickness, which occur 7-14 days after exposure
to the offending antigen, include urticaria, angioedema, fever, chills, malaise, and glomerulonephritis.
Clinical syndromes include serum sickness (e.g., penicillin, streptomycin, sulfonamide,
phenylbutazone hypersensitivity) and the Arthus response. Immune complexes are also observed in
systemic lupus erythematosus (SLE). Type III glomerulonephritis (e.g., poststreptococcal
glomerulonephritis) is characterized by a lumpy bumpy appearance on immunofluorescence using
labeled antibody specific for immunoglobulin or complement.
Type IV (choice D) is also known as delayed-type hypersensitivity (DTH). Unlike the other types,
which are mediated by antibody, DTH depends on TDTH cells that have been sensitized to a
particular antigen. T cells react with antigen in association with MHC class I gene products and
release lymphokines. Examples include tuberculin skin sensitivity and contact dermatitis (e.g., poison
ivy rash).

66
Q
67. Which of the following is found only in gram-negative microorganisms?
A. Cell envelope
B. Exotoxin
C. Peptidoglycan
D. Periplasmic space
E. Teichoic acids
A

The correct answer is D. This question requires you to appreciate the key structural difference
between gram-positive and gram-negative microorganisms. Most gram-negatives (other than
exceptional microorganisms, such as Mycoplasma, that lack a cell wall) have a more complex cell
envelope than gram-positive microorganisms. It includes both a cytoplasmic membrane and an outer
membrane. Between these two membranes is the periplasmic space, which contains enzymes such
as phosphatase and penicillinase, binding proteins for the transport of various nutrients, and
peptidoglycan, as well as a portion of the lipoprotein that firmly anchors the outer membrane to the
peptidoglycan. Gram-positives do not have outer membranes.
Choice A, the cell envelope, is incorrect because both gram-positive and gram-negative
microorganisms have this structure, which is defined as all the layers that enclose the cytosol of the
bacterium. It is the composition of the envelope that differs between gram-positive and gram-negative
microorganisms. Choice B, exotoxin, is not exclusive to gram-negative microorganisms, but is also
found in some gram-positives. By contrast, endotoxin (lipopolysaccharide; LPS) is found exclusively in
gram-negatives. Choice C, peptidoglycan, is found in the cell walls of both gram-positive and gramnegative
microorganisms. Note that there is a larger amount of peptidoglycan in gram-positive
microorganisms. Choice E, teichoic acids, are found exclusively in gram-positive organisms.

67
Q
  1. A 6-week-old infant has a 10-day history of coughing and choking spells. The child is
    gasping for breath and experiencing paroxysms of coughing. Encapsulated, gram-negative
    rods are cultured. The organism is most likely
    A. Bordetella pertussis
    B. Haemophilus influenzae type b
    C. Klebsiella pneumoniae
    D. Legionella pneumophila
    E. Mycoplasma pneumoniae
A

The correct answer is A. The child has whooping cough, or more accurately if one goes by the
chronology of the symptoms, “coughing whoop,” as the patient is subject to a series of coughing
episodes that are followed by a rapid inspiration of air through a narrow airway (the “whoop”). The
infection of the respiratory tract by Bordetella pertussis causes a hyperreactivity of the respiratory
apparatus; even the slightest stimulus will trigger a coughing episode. The offending pathogen
excretes adenylate cyclase and also produces an exotoxin that inactivates the inhibitory subunit of the
G-protein complex, thus activating adenylate cyclase within the cells of the respiratory system. The
symptoms of pertussis last about 6 weeks and are divided into 3 stages: 1) Catarhal stage: Insidious
onset with lacrimation, sneezing, anorexia, coryza and a hacking cough at night 2) Paroxysmal stage:
Characterized by rapid bursts of consecutive coughs followed by a deep, high-pitched inspiration
(whoop) 3) Convalesent stage: Begins 4 weeks after onset and is associated with a reduction in
frequency and severity of paroxysms of cough
Haemophilus influenzae type b (choice B) was the major cause of infant meningitis before the Hib
conjugate vaccine nearly eradicated this pathogen from the United States. Nonencapsulated strains
cause otitis media in children and pneumonia in adults. Klebsiella pneumoniae (choice C) is a gramnegative,
encapsulated rod that is a significant pulmonary pathogen in individuals with a compromised
respiratory apparatus. It is a common cause of aspiration pneumonia and pulmonary abscesses in
alcoholics and patients with chronic obstructive pulmonary disease. Legionella pneumophila (choice
D) is another fastidious, gram-negative, respiratory pathogen that may cause either a fulminating
disease or a mild “walking pneumonia-like” condition (i.e., an atypical pneumonia). Mycoplasma
pneumoniae (choice E) is the most common cause of primary atypical pneumonia. The disease is
considered “atypical” because the patients have a very mild disease with low-grade fever, little in the
way of constitutional signs, and a non-productive cough. These organisms are fastidious and are
difficult to grow in the laboratory.

68
Q
69. An AIDS patient develops symptoms of pneumonia, and Pneumocystis carinii is suspected
as the causative organism. Which of the following stains would be most helpful in
demonstrating the organism's cysts?
A. ELISA
B. Hematoxylin and eosin
C. Methenamine silver
D. Prussian blue
E. Western blot
A

The correct answer is C. The appropriate stain is methenamine silver, since routine hematoxylin and
eosin does not adequately demonstrate the organisms. The cysts, when stained with methenamine
silver, have a characteristic cup or boat shape; the trophozoites are difficult to demonstrate without
electron microscopy. It is also worth knowing that sputum samples are not nearly as effective as
bronchial washes in demonstrating the organisms. ELISA (choice A) is enzyme-linked
immunosorbent assay. It is the primary test for HIV antibody. It is not a stain. Hematoxylin and eosin
(choice B) is the routine tissue stain used in pathology laboratories. Prussian blue (choice D) is good
for demonstrating iron. The western blot (choice E) is the confirmatory test of choice for diagnosing
HIV infection. The western blot test is generally performed once the ELISA test has been shown to be
positive.

69
Q
  1. A sexually active man presents to a dermatologist because of a severe mucocutaneous
    rash that involves most of his body, including his palms and soles. These mucous patches are
    described as being painless, silvery ulcerations of mucous membrane with surrounding
    erythema. If iritis and arthritis are also seen, which of the following is the most likely causative
    agent of this rash?
    A. Herpes simplex I
    B. Herpes simplex II
    C. HIV
    D. Neisseria gonorrhoeae
    E. Treponema pallidum
A

The correct answer is E. The rash described is that of secondary syphilis, caused by Treponema
pallidum. Involvement of palms and soles by a rash is unusual, and secondary syphilis should come
to mind. Not all patients with secondary syphilis have a severe form of the rash, and consequentially
some cases are missed. Primary syphilis takes the form of a painless, button-like mass called a
chancre. Tertiary syphilis, which is now rare, has a propensity for involving the aorta and central
nervous system and can also cause “gummas” (granulomatous-like lesions) in many sites, notably
including liver and bone. The mucous patches are described as being painless silvery ulcerations of
mucous membrane with surrounding erythema. Other symptoms include cranial neuropathies (II -
VIII), iritis, glomerulonephritis, arthritis and periositis. Herpes simplex I (choice A) usually causes
perioral vesicular lesions. Herpes simplex II (choice B) usually causes genital vesicular lesions. HIV
(choice C) does not itself cause a rash, although co-infection with other organisms can result in a
rash. Neisseria gonorrhoeae(choice D) does not typically cause a rash but does not cause urethral
discharge and dysuria.

70
Q
  1. An African child develops massive unilateral enlargement of his lower face in the vicinity of
    the mandible. Biopsy demonstrates sheets of medium sized blast cells with admixed larger
    macrophages. This type of tumor has been associated with which of the following?
    A. Epstein-Barr virus
    B. Hepatitis B
    C. Herpesvirus
    D. HIV
    E. Human papillomavirus
A

The correct answer is A. The patient has Burkitt lymphoma. This high-grade B-cell lymphoma occurs
endemically in Africa (it is the most common neoplasm in children in an equatorial belt that includes
Africa and New Guinea) and sporadically in the U.S. and Europe. The sporadic form is often in an
abdominal site and occurs in young adults. The African form of Burkitt lymphoma has been strongly
associated with antibodies directed against Epstein-Barr virus; the association is weaker in sporadic
cases. Hepatitis B (choice B) is associated with hepatocellular carcinoma. Herpesvirus (choice C)
type 8 is associated with Kaposi sarcoma. HIV (choice D) is linked to AIDS. Some patients also
develop primary lymphomas (not usually Burkitt). Human papillomavirus (choice E) is linked with
common warts, genital condylomata, and genital cancers.

71
Q
  1. A 4-month-old infant presents with failure to thrive, progressive muscular weakness, poor
    head control, dysphagia, and dry mouth. The infant often eats soy-based formula sweetened
    with honey. Which of the following organisms is most likely to be responsible for the child’s
    presentation?
    A. Clostridium botulinum
    B. Clostridium difficile
    C. Clostridium perfringens
    D. Clostridium tetani
    E. Corynebacterium diphtheriae
A

The correct answer is A. The child has infant botulism (floppy baby syndrome)that is caused by
germination of Clostridium botulinum spores (found in honey) in the baby’s gastrointestinal tract.
Patients improve when honey is removed from the diet. This disorder is most common in children
under the age of 6 months; older children and adults do not seem to be vulnerable to this form of
botulism, but are susceptible to botulism caused by ingestion of preformed toxin. Patients usually
present with a sudden onset of dry mouth, diplopia, dysphagia, dysphonia, and muscle weakness
progressing to respiratory paralysis. Pupils are often fixed and dilated. Clostridium difficile(choice B)
causes pseudomembranous colitis, especially after antibiotic therapy. It causes severe diarrhea
through the production of toxin from these bacteria. Clostridium perfringens(choice C) causes gas
gangrene and gastroenteritis, and it is not associated with ingestion of honey. Clostridium
tetani(choice D) causes tetanus and does not cause a food-borne illness in infants. Corynebacterium
diphtheriae(choice E) causes diphtheria in susceptible individuals.

72
Q
  1. A 66-year-old man with an untreated urinary tract infection develops a spiking fever,
    tachypnea, and severe hypoxemia. If adult respiratory distress syndrome is caused as a result
    of the urinary tract infection, blood cultures would most likely reveal
    A. Gram-negative diplococci
    B. Gram-negative rods
    C. Gram-positive cocci
    D. Gram-positive diplococci
    E. Gram-positive rods
A

The correct answer is B. Escherichia coli is a very common pathogen associated with urinary tract
infections and is a common cause of cystitis. In fact, more than 90% of all “first” infections are caused
by Escherichia coli. It is part of the normal flora of the gastrointestinal tract. Patients with cystitis can
develop bacteremia and subsequent septic shock and adult respiratory distress syndrome (ARDS).E.
coli is a gram-negative rod. The key to this question is knowing that most urinary tract infections are
caused by E. coli.
Gram-negative diplococci (choice A) might be Neisseria spp. or Moraxella catarrhalis. Neisseria
gonorrhea is the cause of gonorrhea, a sexually transmitted disease that presents with urethritis or
may be asymptomatic. Patients are usually young, sexually active males. Moraxella spp. are gramnegative
cocci that can cause a wide variety of infections, usually implicated as a cause of otitis media
and sinusitis in children, or as a cause of purulent tracheobronchitis and pneumonia in a population of
patients who are over 50 years of age and have underlying obstructive lung disease. Gram-positive
cocci (choice C), such as Staphylococcus aureus, S. epidermidis and Streptococci, rarely cause
cystitis. Staphylococcus saprophyticus causes urinary tract infections, but the patients are typically
young, sexually active women. Gram-positive diplococci (choice D) would be a description of
Streptococcus pneumoniae, which is the most common cause of community-acquired pneumonia.
This is rarely a cause of UTIs. Gram-positive rods (choice E) would include members of the following
genera: Clostridium, Bacillus, Listeria, and the coryneform bacteria. The only significant member of
this group to produce urinary tract infections is Corynebacterium urealyticum.The organism creates an
alkaline urine environment with the potential for stone formation. Patients are usually
immunocompromised or have had recurrent urinary tract infections.

73
Q
74. Which organism is LEAST likely to be involved in lesions of chronic adult periodontitis?
A. Spirochetes
B. Bacteroides melaninogenicus
C. Capnocytophaga
D. Streptococcus mitis
E. Fusobacteria
A

The correct answer is choice D. Periodontal disease pathogens are usually anaerobic or
capnophilic. The term capnophilic means “carbon-dioxide loving”. In the anaerobic environment deep
in the sulcus, the following genera are often said to contribute to the tissue destruction of periodontal
disease: Bacteroides, Porphyromonas gingivalis, Borelia, Spirochetes,Capnocytophaga, Fusobacteria
and Eichenella. Streptococci (mutans, mitis, sanguis, salivarius) are all gram positive facultative
anaerobes that do not require anaerobic conditions to survive. They are Viridans streptococci,
generally non-pathogenic, and not linked to periodontal disease, although S. mutans is clearly linked
to caries.

74
Q
  1. If a dentist were to administer a bacteriostatic antibiotic, such as a macrolide, and a
    bactericidal antibiotic, such as a cephalosporin, concomitantly to a patient with a severe
    intraoral infection, the most likely pharmacological result for the patient would be which of the
    following?
    A. A small extension of bacterial coverage between the two agents
    B. Antagonism of the antibacterial effects of both agents
    C. Profound toxicity for the patients taking both agents
    D. Synergism of the antibacterial effects of both agents
    E. The appearance of mild antifungal activity
A

The correct answer is B. The concomitant administration of bacteriostatic and bactericidal antibiotics
would result in antagonism of the antibacterial effects of both agents. The reason for the antagonism
between the two is that each agent interferes with the mechanism of action of the other agent. By
definition, a bacteriostatic agent is one that prohibits the growth and development of bacteria and a
bactericidal agent is one that directly causes the death of a microbe during the growth and
development stages. An extension of bacterial coverage between two agents (choice A) is typically
seen when two bacteriostatic or two bactericidal agents are given together. The administration of a
macrolide and a cephalosporin may cause some discomfort for the patient, such as nausea or
stomach cramps; however, profound toxicity for the patients taking both agents (choice C) is very
unlikely. Synergism (choice D) is the correlated action of two agents that results in a final effect that is
greater than that of each agent acting separately. A classical example of synergism is seen when an
aminoglycoside, such as gentamicin, is administered concomitantly with extended spectrum penicillin,
such as piperacillin, for the treatment of a pseudomonas infection. The appearance of mild antifungal
activity (choice E) would not be seen when bacteriostatic and bactericidal antibiotics are given
together, if neither have any antifungal activity

75
Q
76. Which of the following viruses is capable of replication in enucleated cells?
A. Adenovirus
B. Cytomegalovirus
C. Influenza virus
D. JC virus
E. Poliovirus
A

The correct answer is E. Most RNA viruses (eg, poliovirus) replicate in the cytoplasm and therefore
can replicate in enucleated cells. Poliovirus belongs to the family Picornaviridae. These viruses are
nonenveloped and have an icosahedral nucleocapsid that contains positive-sense RNA.
The exception to the rule regarding RNA viruses is the family Orthomyxoviridae, the influenza viruses
(choice C). Orthomyxoviruses undergo transcription and RNA replication in the nucleus of the host
cell because they need to cannibalize the capped 5’ termini of cellular RNAs for use as primers for
viral mRNA transcription. Adenoviruses (choice A) are nonenveloped and have an icosahedral
nucleocapsid that contains a double-stranded linear DNA genome. Cytomegalovirus (choice B) is a
member of family Herpesviridae. It is an enveloped virus with an icosahedral nucleocapsid that
contains a double-stranded linear DNA genome. JC virus (choice D) belongs to family Papovaviridae.
It is nonenveloped and has an icosahedral nucleocapsid that contains a double-stranded circular DNA
genome. For most DNA viruses, transcription and DNA replication occur in the nucleus of the host
cell. The exception to this observation is the family Poxviridae, which carries out its replication in the
cytoplasm. Poxviridae includes variola virus, vaccinia virus, molluscum contagiosum, and orf virus.

76
Q
  1. Blood cultures for a patient are positive for alpha-hemolytic gram-positive diplococci.
    Immunity to the causative organism is based on
    A. alternative complement pathway activation
    B. antibody to an alpha-helical coiled fimbria
    C. IgA antibodies to C carbohydrate
    D. IgG antibodies to C carbohydrate
    E. IgG antibodies to a surface acidic polysaccharide
A

The correct answer is E. The patient in this question has pneumococcal pneumonia. It is caused by
Streptococcus pneumoniae, an alpha-hemolytic, gram-positive coccus that grows in chains. It can be
31
easily distinguished from other alpha-hemolytic streptococci because it is exquisitely sensitive to bile
and bile-like compounds, such as optochin. It is the most common cause of community-acquired
pneumonia and the most common cause of community-acquired meningitis in adults older than 30
years of age. The only recognized virulence factor of S. pneumoniae is its carbohydrate capsule
(which contains acidic polysaccharides). Antibody to a specific capsule type is necessary to overcome
infection. More than 80 capsule types have been recognized. The 23 types that most commonly
cause disease are contained in a vaccine (pneumovax) that is recommended for high-risk groups,
including the elderly and those undergoing splenectomy. Increased susceptibility is also found in
patients with Hodgkin disease, chronic lymphocytic leukemia, and myeloma. The vaccine should still
be given to patients with these conditions, but it is less successful. The alternative complement
pathway (choice A) is important in clearing Neisseria infections. Individuals with deficiencies in C5
through C8 are at increased risk for disease from Neisseria. The fimbria of Streptococcus pyogenes
(Group A beta-hemolytic streptococcus) is composed of an alpha-helically coiled M protein. Antibody
against a specific M type (choice B) will prevent infection. Raising antibodies to M proteins, however,
can lead to rheumatic fever, so strep throat infections are routinely treated with penicillin to prevent an
antibody response. The C carbohydrate is an antigen of beta-hemolytic streptococci used to divide
them into different groups. Antibody against C carbohydrate (choices C and D) is not protective.

77
Q
78. A microbiology laboratory reports growth of Staphylococcus aureus from pus drained from
a breast abscess. What is the most likely condition predisposing the patient to the
development of a breast abscess?
A. Breast feeding
B. Endocarditis
C. Inflammatory breast carcinoma
D. Menopause
E. Paget disease of the breast
A

The correct answer is A. Acute mastitis, frequently complicated by breast abscess formation,
typically develops in the postpartum period when the nipples are first subjected to the physical
stresses of breast feeding. Bacteria enter the breast by way of cracks in the nipple and flourish in the
microenvironment of the lactating breast. Acute mastitis causes redness, pain, and swelling in the
affected breast; S. aureus is the most common pathogen. There is a very apparent cellulitis and fever
may be present. Penicillin antibiotics are generally administered because they are effective and safe
to use during breast-feeding. Although endocarditis (choice B) could send septic emboli to the breast,
S. aureus endocarditis is more commonly associated with bacteremia and fevers, proliferative
glomerulonephritis, valvular dysfunction, and emboli to the brain, kidneys, heart, and gut.
Inflammatory breast carcinoma (choice C) does not invoke an inflammatory response, and it is not
associated with an infection. Inflammatory breast carcinoma is characterized by tumor spread into the
dermal lymphatics, producing diffuse induration and skin tenderness with the typical peau d’orange
(orange peel) appearance. Mastitis arising in the perimenopausal period (choice D) is usually a
chronic mastitis caused by obstruction of ducts from inspissated secretions. Chronic mastitis is sterile.
The breast tissue shows lymphocytes and plasma cells surrounding dilated ducts filled by cellular
debris. Paget disease (choice E) presents as skin changes on the breast or nipple in association with
ductal carcinoma in the underlying breast that is percolating out into the epidermis. Paget disease
produces an eczematous skin lesion that may be crusted or weeping, but it is not related to bacterial
infection.

78
Q
  1. A 3-year-old child has multiple isolated lesions on his face and neck. The lesions are up to
    4 cm in diameter with golden crusts, whereas in other sites small blisters and weeping areas
    are seen. Which of the following is the most likely diagnosis?
    A. Aphthous ulcers
    B. Erysipelas
    C. Herpes simplex I
    D. Impetigo
    E. Measles
A

The correct answer is D. This is impetigo, which is typically seen in preschool children with poor
hygiene, particularly in the summer in warm climates. The characteristic lesion has a large golden
crust. Most cases are caused by Staphylococcus aureus; Streptococcus pyogenes is occasionally
32
implicated. Impetigo is highly infectious, and mini-epidemics can occur in daycare settings. The initial
treatment is typically with penicillins and topical preparations, primarily muciprocin ointment.
Methicillin-resistant strains are presently rare in this setting, but can occur. Itching is the only
symptom. The lesions consist of macules and vesicles. Aphthous ulcers (choice A), commonly known
as “canker sores,” are painful, shallow ulcers of the oral cavity. Erysipelas (choice B) is a different
type of skin infection, often caused by Streptococcus pyogenes (also sometimes others including
Staphylococcus), and is characterized by large erythematous patches. Herpes simplex I (choice C)
causes tiny oral and perioral vesicles, but not large golden crusts. Measles (choice E) causes a
blotchy erythematous rash on the face and all over the body. These lesions typically do not “weep” as
seen with impetigo.

79
Q
80. A woman delivers a full term baby with possible toxoplasmosis, but no obvious signs of
infection with the protozoan parasite. The best test to diagnose acute infection in the newborn
would be to look for:
A. IgA
B. IgE
C. IgG 1
D. IgG 4
E. IgM
A

The correct answer is E. IgM immunoglobulin directed against Toxoplasma would provide evidence
of infection in the newborn baby. IgM is the only antibody that a baby can form with an acute infection.
It is the first antibody that is formed in an infection and it is also the antibody that is present on the
surface of immature and mature B cells. IgA (choice A) is the antibody that is produced in response
to mucosal infections. It cannot cross the placenta. The baby’s immune system is still not well
developed at birth and cannot produce IgA immunoglobulin. In adults or older children, IgA exists in
serum in a monomeric form but is present in seromucous secretions as a dimer. IgE (choice B) is the
antibody that is produced in response to an allergen. It cannot cross the placenta, and is not present
in the baby at birth or for some time after birth. The baby’s immune system is still not well developed
at birth and can initially only form IgM immunoglobulin. IgG 1 (choice C) and IgG4 (choice D)
immunoglobulins might be present in the baby, but would not indicate infection in the infant. The
presence of these isotypes would indicate that the mother was infected with the organism and
produced antibody that was transported across the placenta in utero. IgG is the only immunoglobulin
that can cross the placenta, providing protection to the baby during the first few months of life.

80
Q
  1. Two weeks after birth, a neonate develops sepsis, skin vesicles, and conjunctivitis. Over
    the next several days, the infant’s condition deteriorates with development of seizures, cranial
    nerve palsies, and lethargy. The infant dies approximately 1 week after onset of symptoms.
    Which of the following infectious agents would most likely cause this clinical presentation?
    A. Cytomegalovirus
    B. Herpes simplex
    C. Rubella
    D. Syphilis
    E. Toxoplasmosis
A

The correct answer is B. All of the agents listed can cause devastating congenital infections with
high mortality and often major organ malformation (the TORCH agents: Toxoplasma, other, rubella,
cytomegalovirus, herpes simplex). It is herpes simplex type II, however, typically acquired during
delivery, that causes the devastating neonatal encephalitis described in the question stem. The
mortality rate for neonatal herpes is approximately 65%. Only 10% of these infants escape without
neurologic sequelae. A distinguishing feature is the propensity to involve the temporal lobe, with mass
effect on imaging studies and temporal lobe seizure foci on EEGs. A point worth remembering is that
adult herpes encephalitis (in nonimmunosuppressed individuals) is usually caused by herpes simplex
I, whereas neonatal and congenital herpes are usually caused by herpes simplex II.

81
Q
  1. A young woman presents with a confluent maculopapular rash that began on her face then
    spread downward over her trunk. She also has fever and headache, with bilateral neck pain
    and joint pain. Which of the following diseases does she most likely have?
    A. Infectious mononucleosis
    B. Lyme disease
    C. Roseola
    D. Rubella
    E. Rubeola
A

The correct answer is D. Rubella, also called German measles or 3-day measles, is a disease
caused by a Togavirus, which is a small, enveloped, single-stranded (+) linear RNA virus.
Approximately 40% of patients are asymptomatic or have mild symptoms. In symptomatic patients,
the clinical presentation typically consists of an erythematous rash that begins on the head and
spreads downward to involve the trunk, and lasts for approximately 3 days. In addition, symptoms
include fever, posterior cervical lymphadenopathy, and arthralgias. The greatest danger from rubella
is to the fetus. If clinical rubella develops or seroconversion is demonstrated, there is a high risk for
congenital abnormalities or spontaneous abortion. Women of childbearing age should be warned not
to become pregnant within 2-3 months from the time of immunization. Mild arthralgias and other
symptoms may develop in 25% of immunized women. Enteroviral rashes may mimic rubella and
rubeola.
Infectious mononucleosis (choice A) is caused by the Epstein-Barr virus, a herpesvirus. Classic
findings include fever, exudative pharyngitis, generalized lymphadenopathy, severe malaise (most
common complaint), and hepatosplenomegaly. A rash is not a characteristic feature unless the patient
has been treated with ampicillin. Lyme disease (choice B) is caused by the spirochete Borrelia
burgdorferi. The disease is transmitted by the bite of the tick Ixodes dammini. The initial lesion is an
annular rash with central clearing and a raised red border (erythema chronicum migrans) at the bite
site. Patients also have fever, malaise, myalgias, arthralgias, headache, generalized
lymphadenopathy, and, occasionally, neurologic findings. Roseola (choice C) is caused by human
herpesvirus 6. Other names include exanthem subitum or sixth disease. Children have a febrile period
of 3-5 days with rapid defervescence followed by an erythematous maculopapular rash lasting 1-3
days. Rubeola (choice E), or regular measles, is a disease caused by a paramyxovirus. Patients
present with an upper respiratory prodrome and characteristic oral lesions (Koplik spots) that precede
the rash. The nonpruritic maculopapluar rash begins on the face and spreads to the trunk and
extremities, including palms and soles. The incubation period is 10-14 days. Patients also have a
posterior cervical lymphadenopathy. The virus is not associated with risk for a fetus.

82
Q
  1. A 29-year-old woman AIDS has had a progressive blurring of vision in her right eye. Which
    of the following is the most appropriate therapy for this patient?
    A. Acyclovir
    B. Amantadine
    C. Flucytosine
    D. Ganciclovir
    E. Zidovudine
A

The correct answer is choice D. The best drug treatment for CMV this infection of the retina is
ganciclovir. Acyclovir (choice A) is not effective in CMV infections. It is used more for HSV type 1 and
2 infections. Amantadine (choice B) is used either therapeutically or prophylactically for the influenza
A virus. Flucytosine (choice C) is an antifungal agent. Zidovudine (choice E) is a reverse
transcriptase inhibitor and is used as a first-line drug for the treatment of AIDS. The drug by itself is
ineffective against CMV retinitis.

83
Q
84. The causative organism of gonorrhea has which of the following characteristics?
A. Both hyphae and spores
B. Can live within neutrophils
C. Cannot make ATP
D. No true cell wall
E. Stains with silver stains
A

The correct answer is B. The causative organism is Neisseria gonorrhoeae, which is a sexually
transmitted gram-negative coccus that can live in phagocytic vacuoles in neutrophils and
macrophages. The organism is the most common cause of septic arthritis in otherwise healthy,
sexually active adults. Cervicitis in women with purulent discharge or women who are asymptomatic,
yielding a positive culture, is a common charactristic. Fever, rash, tenosynovitis and arthritis occur
with disseminated disease. Hyphae and spores (choice A) would be features of Candida, which
usually causes arthritis as a rare complication of systemic candidiasis in immunocompromised
patients. Furthermore, there would be the appearance of other classic characteristics, such as the
appearance of creamy white lesions. Chlamydia is an intracellular organism that cannot make ATP
34
(choice C) and does not usually cause septic arthritis. Mycoplasma do not have true cell walls
(choice D) and do not usually cause septic arthritis, but are commonly associated with respiratory
conditions, such as mycoplasma tuberculosis. Syphilis is caused by a spirochete that stains with silver
stains (choice E). Secondary syphilis can (uncommonly) cause an acute arthritis, but the interval
between acquiring the infection and developing the arthritis is usually months to years.

84
Q
  1. A large lung abscess is biopsied from which pure Fusobacterium nucleatum is cultured.
    What is the most likely source of the Fusobacterium?
    A. Blood
    B. Infected aerosols
    C. Oral cavity
    D. Stomach
    E. Upper respiratory tract
A

The correct answer is C. Anaerobic bacteria make up most of the normal human oral flora. For
example, prominent members of the normal microbial flora of the mouth include anaerobic
spirochetes, fusobacteria, and prevotella. Anaerobic lung abscesses commonly arise from aspirated
oral secretions in patients with compromised neurologic status (e.g., anesthesia, coma, intoxication),
or in individuals with a depressed cough reflex. The most common anaerobic organisms isolated from
lung abscesses include Bacteroides, Fusobacterium, and Peptococcus species, all of which are
typical oral flora.
Blood-borne pathogens (choice A) producing lung abscesses probably originate as emboli from
vegetations on the right heart. The most likely pathogens in endocarditis are Staphylococcus and
Streptococcus species; Fusobacterium would be very unusual. Fusobacterium usually is not
pathogenic and lives in a commensal relationship with the host. Infective aerosols (choice B)
generally contain organisms that are intrinsically pathogenic (e.g., mycobacteria, influenza virus). It is
the acidity of the gastric aspirate that most seriously injures the lung, not the gastric bacteria.
Fusobacterium do inhabit the gastrointestinal tract, but viable bacteria are much more likely to be
transmitted from the oral secretions than from the stomach (choice D). The upper respiratory tract
(choice E) is generally sterile, although COPD is frequently complicated by chronic bronchitis.
Numerous bacterial and viral pathogens can be involved in chronic bronchitis; however,
Fusobacterium is not an upper respiratory tract pathogen.

85
Q
  1. A 67-year-old woman in a skilled nursing facility complains of flu-like symptoms of high
    fever, dyspnea, and a productive cough. A chest x-ray shows a cavitary lesion in her left lung
    which is confirmed to be an abscess. If a gram-positive bacteria is noted, which of the
    following organisms would most likely be identified from examination of her sputum?
    A. Candida albicans
    B. Klebsiella pneumoniae
    C. Pneumocystis carinii
    D. Staphylococcus aureus
    E. Streptococcus pneumoniae
A

The correct answer is D. The woman has developed a pulmonary abscess, as evidenced by the
chest radiograph. Of the organisms listed, Staph. aureus is the most likely cause of bacterial
pneumonia complicated by abscess formation. Bacteria commonly producing pneumonia developing
in the context of influenza include Staphylococcus aureus, Haemophilus influenzae, and
Streptococcus pneumoniae, but Streptococcus pneumoniae is not a frequent cause of lung
abscesses. Staph. aureus pneumonia/abscess are more common in patients in chronic care facilities,
nosocomial infections, influenza epidemics and cystic fibrosis. Candida albicans(choice A) would not
be a likely cause of this woman’s abscess, or of the pneumonia that preceded it.Candida pneumonia
in a nonimmunosupressed individual is rare. Note also that Candida is a fungus. Klebsiella
pneumoniae(choice B) is more likely in men who are alcoholic or diabetic, or suffering from chronic
obstructive lung disease. It is also a plump gram-negative encapsulated rod. Pneumocystis
carinii(choice C) is associated with pneumonia in immunocompromised hosts, especially AIDS
patients, but there is no indication of immune compromise in this patient. Furthermore, it would not
show as a gram-positive organism, as it is a fungus. Streptococcus pneumoniae(choice E) is only
rarely associated with lung abscess. It commonly causes pneumonia in chronic lung diseases. It is a
gram-positive diplococcius.

86
Q
  1. Following a barbecue hosted by a hunter who served “bear hamburgers,” several guests
    develop abdominal pain, diarrhea, fever, myalgia, periorbital edema, and petechial
    hemorrhages in the conjunctiva. A peripheral blood smear shows an increased eosinophil
    count. Which of the following is the most likely cause of these symptoms?
    A. Anthrax
    B. Bacillus cereus food poisoning
    C. Botulism
    D. Escherichia coli gastroenteritis
    E. Trichinosis
A

The correct answer is E. Although inadequately cooked pork and trichinosis are commonly
associated, the quality of purchased pork and the degree of cooking in this country are sufficiently
high that pork-related trichinosis is uncommon. The more common scenario is actually the one
described in the question stem, for two reasons. First, bear meat is not inspected, and it may often be
served “rare,” or relatively uncooked. The infected meat contains cysts that are released in the
gastrointestinal tract after ingestion. The Trichinella spiralis larvae penetrate the intestinal mucosa and
develop into adult worms in 30-40 hours. After reaching adulthood, they mate, and the females
produce larvae that grow to maturity and seek out muscle in which to encyst (often in the orbital
muscles) within approximately 5-8 days. If you didn’t know the answer to this question, you still might
have noticed the presence of eosinophilia, which should have clued you in to the presence of a
parasitic infestation, making trichinosis the only plausible answer.
Anthrax (choice A) after eating bear meat is very unlikely. In addition, anthrax would not produce the
described symptoms; instead, cutaneous lesions (95% of cases) or respiratory disease culminating in
death (5% of cases; wool sorter disease) would occur. Bacillus cereus food poisioning (choice B) and
Escherichia coli gastroenteritis (choice D) might lead to similiar gastrointestinal symptoms but would
not cause the eye findings. Botulism (choice C) causes flaccid paralysis, not the symptoms
described.

87
Q
  1. An alcoholic patient is brought to the emergency department in respiratory distress. A
    chest x-ray film demonstrates lobar consolidation of the right lower lung. Which of the
    following organisms should be highest on the differential diagnosis?
    A. Klebsiella pneumoniae
    B. Legionella spp.
    C. Mycoplasma pneumoniae
    D. Pneumocystic carinii
    E. Staphylococcus aureus
A

The correct answer is A. Lobar pneumonia, in which an entire lobe of the lung becomes rapidly
affected with pneumonia, is acutally a relatively uncommon pattern for pneumonia. Common
causative organisms include Streptococcus pneumoniae (pneumococcus), Haemophilus influenzae,
and Klebsiella pneumoniae. K. pneumoniae is specifically associated with alcohol abuse, diabetes
mellitus, and nosocomial infections. Legionella spp. (choice B) can cause either a patchy or lobar
consolidation but are not specifically associated with alcoholism. Instead, Legionella infection is
associated with inspiration of aerosolized contaminated water. Mycoplasma pneumoniae (choice C)
causes extensive patchy infiltrates rather than lobar pneumonia. It is the most common cause of
pneumonia in young adults. Pneumocystis carinii (choice D) causes diffuse interstitial and alveolar
infiltrates, typically in patients immunosupressed by AIDS, cytotoxic drug therapy, or cancer.
Staphylococcus aureus (choice E) produces patchy infiltrates. Staphylococcal pneumonia may
complicate influenza during epidemics, or it may be a nosocomial infection. It is associated with
empyema formation.

88
Q
  1. An otherwise healthy patient who has just received a prosthetic aortic valve develops
    postoperative fever. Blood cultures are done, and she is given broad-spectrum antibiotics.
    Two days later she is still febrile and clinically deteriorating. Which of the following is the most
    likely pathogen?
    A. Actinomyces israelii
    B. Candida albicans
    C. Histoplasma capsulatum
    D. Nocardia asteroides
    E. Trichophyton rubrum
A

The correct answer is B. The patient likely has candidal infection of the prosthetic aortic valve. That
is why she did not respond favorably to antibacterial therapy, which is known to promote fungal
infection. Note that Candida tends to colonize foreign bodies, such as intravenous and Foley
catheters, prosthetic valves, and ventricular shunts. Actinomyces israelii (choice A) is known to cause
cervicofacial infection in patients undergoing dental work. Histoplasma capsulatum (choice C) causes
histoplasmosis, a pulmonary infection common in the midwestern river valleys. Multiorgan
involvement is usually seen only in the immunocompromised. Transmission of the organism occurs
through the inhalation of airborne microconidia (infectious) spores. The organism is found in bird and
bat droppings and in the soil. Nocardia asteroides (choice D) is an actinomycete that causes a
chronic lobar pneumonia that may metastasize to the brain. It is more common in
immunocompromised patients. It is found in the soil and in aquatic environments. Trichophyton
rubrum (choice E) is one of the organisms that commonly produces a variety of cutaneous mycoses,
including tinea corporis (ringworm), tinea cruris (jock itch), and tinea pedis (athlete’s foot).

89
Q
  1. A formula-fed, 1-month-old boy is exposed to his sister, who has chickenpox. He does not
    develop signs of varicella. His mother had the infection 5 years ago. Which class of
    immunoglobulins did he acquire from his mother in utero that protected him from this virus?
    A. IgA
    B. IgD
    C. IgE
    D. IgG
    E. IgM
A

The correct answer is D. This infant is exhibiting passive immunity acquired from his mother in utero.
IgG is the only class of immunoglobulin that can cross the placenta. As such, IgG molecules diffuse
into the fetal circulation, providing immunity. This circulating maternal IgG protects the newborn during
the first 4-6 months of life. Note that IgG is also capable of opsonization and complement activation (a
feature shared with IgM). IgA (choice A) functions in the secretory immune response. The secretory
form of this immunoglobulin (sIgA) is found in tears, colostrum, saliva, breast milk, and other
secretions. It is produced by the plasma cells in the lamina propria of the gastrointestinal and
respiratory tracts. IgD (choice B) functions as a cell surface antigen receptor on undifferentiated B
cells. IgE (choice C) is involved in the allergic response and immediate hypersensitivity reactions.
The Fc region of IgE binds to the surface of basophils and mast cells. Antigen binding to two IgE
molecules leads to mast cell degranulation and the release of leukotrienes, histamine, eosinophil
chemotactic factors, and heparin. IgM (choice E) is the first antibody detected in serum after
exposure to antigen. IgM circulates as a pentamer and thus has five Fc regions. This structure makes
it especially effective in fixing complement.

90
Q
91. Which of the following organisms is a natural transformer?
A. Escherichia coli
B. Neisseria gonorrhoeae
C. Plasmodium vivax
D. Pseudomonas aeruginosa
E. Staphylococcus aureus
A

The correct answer is B. Transformation is the uptake and integration of naked DNA by a bacteria
from the environment. Transformation can be induced in the laboratory (a technique used to introduce
gene-carrying plasmids into bacteria) or, relatively uncommonly, occurs naturally. Only a few
medically important species undergo natural transformation: Haemophilus species, Streptococcus
species, Neisseria gonorrhoeae, and Helicobacter pylori.

91
Q
  1. A patient presents with chronic, symmetric polyarthritis involving the knees. He gives a
    history of having developed an extensive rash after a deer hunting trip in Connecticut after
    which he felt “sick,” and developed knee pain that prevented him from climbing stairs for
    several months, but then it partially resolved. Which of the following organisms is most likely
    etiologically related to the patient’s arthritis?
    A. Fungus
    B. Gram-negative cocci
    C. Gram-negative rod
    D. Gram-positive cocci
    E. Spirochete
A

The correct answer is E. The history is characteristic for Lyme disease, caused by the spirochete
Borrelia burgdorferi. The clues are deer, Connecticut, rash, knees, and acute arthritis followed by
chronic arthritis. Patients with Lyme disease initially have erythema migrus, a flat or slightly raised
lesion that expands with central clearing. Headache and stiff neck are also common. Arthralgias,
arthritis, and myalgias are seen. The arthritis is often chronic and recurrent. Fungal arthritis (choice
A) can be caused by coccidioidomycosis, blastomycosis, sporotrichosis, and candidiasis. N.
gonorrhoeaeare gram-negative cocci (choice B) that can cause septic arthritis. Septic arthritis is
associated with acute arthritis, usually monarticular, most often in large weightbearing joints. Previous
joint damage or intravenous drug abuse are common risk factors. Haemophilus (in young children)
and Salmonella (in sickle cell patients) are gram-negative rods (choice C) that can cause septic
arthritis. Staphylococcus are gram-positive cocci (choice E) that can cause septic arthritis.

92
Q
  1. What is the mechanism of action of the antibiotic ciprofloxacin?
    A. It inhibits dihydrofolate reductase
    B. It inhibits DNA-dependent RNA polymerase
    C. It inhibits protein synthesis by binding to the 30s ribosomal subunit
    D. It inhibits protein synthesis by binding to the 50s ribosomal subunit
    E. It inhibits topoisomerase II (DNA gyrase)
A

The correct answer is E. Antibiotic mechanism questions are NBDE Part 1 favorites. Ciprofloxacin
and norfloxacin belong to a category of antibiotics called the fluroquinolones. They are bactericidal
and work by inhibiting topoisomerase II (DNA gyrase). They are effective against gram-negative rods
and are the only oral agents effective against Pseudomonas. Inhibition of dihydrofolate reductase
(choice A) is the mechanism of action of trimethoprim, which is typically used in combination with
sulfonamides which inhibit an earlier step in folate synthesis (dihydropteroate synthase).
Trimethoprim-sulfa is used in the treatment of Shigella, Salmonella, recurrent urinary tract infections,
and Pneumocystis carinii pneumonia. Inhibition of DNA-dependent RNA polymerase (choice B) is the
mechanism of action of rifampin. Rifampin is used (along with other drugs) in the treatment of
tuberculosis. Inhibition of the 30s ribosomal subunit (choice C) is the mechanism of action of two
important classes of antibiotics - the tetracyclines (tetracycline, doxycycline, democlocycline) and the
aminoglycosides (gentamycin, tobramycin, streptomycin, etc.). The tetracyclines inhibit the
attachment of the aminoacyl-tRNA to the ribosome, whereas the aminoglycosides inhibit the formation
of the intitiation complex. Inhibition of the 50s ribosomal subunit (choice D) is the mechanism of
action of the macrolides (e.g., erythromycin), the lincosamides (e.g., lincomycin, clindamycin), and
chloramphenicol. Chloramphenicol inhibits the 50s peptidyl transferase, whereas erythromycin blocks
translocation.

93
Q
  1. An injection of lipopolysaccharide (LPS) into the vascular system will rapidly produce
    myocardial dysfunction, hypotension, disseminated intravascular coagulation, and coma. This
    sequence of events most closely mimics what type of shock?
    A. Anaphylactic
    B. Cardiogenic
    C. Hypovolemic
    D. Neurogenic
    E. Septic
A

The correct answer is E. Since lipopolysaccharide can be seen in gram-negative bacteria, it is
important to understand how it will affect the body. Septic shock is a complex, multisystem organ
failure that can be produced either by LPS (which is present in the cell wall of all gram-negative
bacteria) or certain toxins released by gram-positive bacteria and fungi. LPS binds to a serum protein
and stimulates CD14 receptors on endothelial cells and circulating inflammatory cells, eliciting a broad
range of end-organ responses. Anaphylactic shock (choice A) is brought about by an exaggerated
Type I hypersensitivity reaction mediated by IgE antibodies bound to mast cells and basophils. The
resulting degranulation produces massive histamine and adenosine release, which produces
constriction of the bronchi and pulmonary circulation. Cardiogenic shock (choice B) reflects the
inability of the heart to maintain arterial pressure sufficiently to perfuse the systemic vasculature.
Cardiogenic shock is intrinsic to the heart and usually a consequence of ischemia, arrhythmia, or
obstruction. Hypovolemic shock (choice C) occurs when blood volume decreases to a point at which
it is inadequate to maintain arterial pressure in the vital organs. Hypovolemic shock is due to
hemorrhage, fluid loss from burns, or severe diarrhea and vomiting. Neurogenic shock (choice D) is
an unusual form of shock that occurs in catastrophic nervous system injuries that cause diffuse
vasodilation and hypotension.

94
Q
  1. A 28-year-old male intravenous drug user presents with fever, chills, weakness, dyspnea,
    cough, arthralgia, diarrhea, and abdominal pain. On examination, a heart murmur is present,
    and small, tender nodules are found on the finger and toe pads, along with small hemorrhages
    on the palms and soles. Infective endocarditis, caused by which organism, should be
    suspected?
    A. Enterococci
    B. Pseudomonas aeruginosa
    C. Staphylococcus aureus
    D. Streptococcus pneumoniae
    E. Streptococcus pyrogenes
A

The correct answer is C. The question is essentially asking “Which of the following is the most
common cause of infective endocarditis (IE) in intravenous drug users?” Being an intravenous drug
user places him at a very high risk for the development of IE. The factors that determine the clinical
presentation of IE include the nature of the organism, the valve infected, and the route of the infection.
Although the clinical characteristics of IE can vary, most patients present with a febrile illness lasting
several days to 2 weeks. This illness is often accompanied by a variety of nonspecific signs and
symptoms, such as chills, weakness, dyspnea, cough, arthralgia, diarrhea, and abdominal pain. Heart
murmurs occur in approximately 90% of all patients, but may be absent in patients with right-sided
infections. Other clinical signs include Osler nodules (purplish or erythematous subcutaneous papules
or nodules on the pads of the fingers and toes), Janeway lesions (hemorrhagic painless plaques on
the palms and soles), petechiae (small erythematous painless hemorrhagic lesions that may appear
anywhere), and splinter hemorrhages (thin linear hemorrhages found under the nail beds of fingers
and toes). The diagnosis of IE is dependent on positive blood cultures and echocardiographic
evidence of “valvular vegetation” or valvular injury. Staphylococci, Streptococci, and Enterococci are
possible pathogens; however, Staphylococcus aureus accounts for most cases of acute IE.
Enterococci(choice A) cause a minority of cases of valvular endocarditis, but are not usually involved
in endocarditis associated with intravenous drug abuse. Gram-negative organisms such as
Pseudomonas aeruginosa(choice B) are rarely the cause of IE. They primarily cause pneumonia in
cystic fibrosis patients and those on ventilators. Streptococcus pneumoniae(choice D) and
Streptococcus pyogenes(choice E) are not common causes of IE. They are, however, common
causes of pneumonia and pharyngitis, respectively.

95
Q
96. A construction worker with sustained trauma later develops tetanus. What is the causative
organism?
A. Gram-negative anaerobic rod
B. Gram-positive cocci
C. Gram-positive anaerobic rod
D. Gram-positive aerobic rod
A

The correct answer is C. The causative organism of tetanus is Clostridium tetani , which is a grampositive,
spore-forming anaerobic rod. Bacteroides is gram-negative anaerobic rod (choice A).
Examples of gram-positive cocci (choice B) are staphylococci and streptococci. The causative
organism of anthrax, Bacillus anthracis , is a gram-positive, spore-forming aerobic rod (choice D).

96
Q
97. Last year's influenza A vaccine is unlikely to be effective today because influenza A
A. has a heavy polysaccharide coat
B. immunosuppresses the patient
C. kills lymphocytes
D. resists inactivation by complement
E. undergoes genetic reassortment
A

The correct answer is E. The difficulty with developing a vaccine against influenza A arises because
the influenza virus genome is composed of eight strands of single-stranded RNA. Minor shifts
(antigenic drift) in surface antigens that occur as point mutations in the genes accumulate. Influenza A
can also undergo larger, abrupt changes in antigen expression (antigenic shift), however, as a
consequence of reassortment of some of the RNA fragments between human and nonhuman hosts.
39
Thus, last year’s vaccine does not necessarily work against this year’s virus. Polysaccharide coats
(choice A) are a virulence factor of some bacteria, not of viruses. Influenza A can compromise the
lungs sufficiently to predispose to secondary infections, producing a functional immunosupression
(choice B), but this attribute does not make it difficult to produce vaccines against the virus. Unlike
AIDS, influenza virus does not selectively target lymphocytes (choice C). Influenza A, bound to
antibody and complement, can be effectively phagocytized (compare with choice D).

97
Q
  1. Infections in hyaline cartilage typically destroy the cartilage because
    A. cartilage contains chondroitin sulfate
    B. cartilage contains collagen
    C. cartilage is not innervated
    D. cartilage is relatively avascular
    E. cartilage tends to become calcified
A

The correct answer is D. Hyaline cartilage contains very few, sparsely distributed, small blood
vessels. The tissue consequently is very vulnerable to infection because the immune system (white
cells and serum with clotting factors and antibodies) cannot access the cartilage in an efficient
fashion. Chondroitin sulfate (choice A) is an important constituent of cartilage that can be attacked by
bacteria, but this is not the cause of uncontrollable infections in the cartilage, because if the infection
is controlled, chondrocytes will continue to synthesize chondroitin. Collagen (choice B) is present in
cartilage, but does not predispose the cartilage for uncontrollable infection. Collagen is not innervated
(choice C), but this does not contribute to the development of uncontrollable infection. The tendency
of cartilage to undergo calcification (choice E), even when not acting as the site of ossification, does
not predispose for uncontrollable infection.

98
Q
  1. The highest level of chronic disease state is associated with which of the following
    diseases?
    A. Hepatitis A
    B. Hepatitis B
    C. Hepatitis C
    D. Hepatitis D
    E. None of these diseases have chronic carrier stages
A

The correct answer is choice C. Hepatitis C is a disease about which much has been learned
recently, but much more is unknown. It seems to be transferred through both bloodborne and sexual
routes (not fecal/oral). It is harder to contract than HBV, but easier than HIV. It is distinguished by
having the highest percentage of cases going into a chronic destructive stage, of all hepatitis types. It
is a single stranded RNA virus. It was formerly known as NANB (Non-A Non-B) or “transfusion
hepatitis” because tests for hepatitis A or B did not detect it. Hepatitis B also has a destructive carrier
state, but it is less common. Hepatitis A has NO carrier state, and exposure to Hepatitis A gives
lifelong immunity.

99
Q
100. Which of the following organisms would most likely be isolated from the vagina of a
normal 5-year-old girl?
A. Candida
B. Lactobacillus
C. Neisseria
D. Staphylococcus
E. Streptococcus
A

The correct answer is D. The vagina of prepubertal girls and postmenopausal women is colonized
by colonic and skin bacteria, including Staphylococcus epidermidis, which is normally found on the
skin. The vagina of women of childbearing age tends to be colonized by Lactobacillus (choice B)
species, yeasts such as Candida (choice A), and Streptococcus species (choice E). The presence of
Neisseria (choice C), such as N. gonorrhoeae (the cause of gonorrhea), in the vagina of a 5-year-old
girl strongly suggests sexual abuse.

100
Q
  1. In cases of strepococcus pneumonia, why is the patient’s sputum filled with pus?
    A. Teichoic acids and peptidoglycan are chemotactic for neutrophils
    B. The capsule of the causative agent is chemotactic for neutrophils
    C. The causative agent is an intracellular organism
    D. The causative agent is beta hemolytic
    E. The organism produces an IgA protease
    40
A

The correct answer is A. The answer to this question requires that the student realize that pus
consists of bacteria and dead and dying neutrophils. This fact reflects that the patient has typical
(rather than atypical) pneumonia. In the United States, the most common agent of this would be
Streptococcus pneumoniae, a gram-positive extracellular pathogen rich in teichoic acids and
peptidoglycan that elicit the neutrophilic exudate. The capsule of this organism is a polysaccharide
and primarily elicits an antibody response rather than attracting neutrophils (choice B). Streptococcus
pneumoniae is an extracellular, not an intracellular (choice C) organism. Streptococcus pneumoniae
is alpha hemolytic, not beta hemolytic (choice D). This organism can be distinguished from other
bacteria by the inhibition of growth in the presence of optochin or bile. Another common alpha
hemolytic bacteria is the Viridans streptococci group. Streptococcus pneumoniae does produce an
IgA protease (choice E) that enhances the ability of the organism to infect the respiratory mucosa, but
this does not contribute to pus formation.

101
Q
102. Ethylene oxide sterilization procedures usually require exposure times of
A. 20-30 minutes
B. 1-2 hours
C. 2-3 hours
D. 8-10 hours
E. 24 hours
A

The correct answer is D. Ethylene oxide is used for sterilization of heat sensitive instruments and
materials. Its advantage is that it does not melt, damage, or corrode instruments, as heat may. Its
disadvantages include long cycle time (8-10 hours) and additional time to air out the materials so that
they do not contain ethylene oxide residue. A period of 20-30 minutes is a common steam autoclave
time. A period of 1-2 hours is a common dry heat autoclave time. A period of 24 hours was a time
recommended for “cold sterilization” by chemicals. Cold sterilization is a misnomer, as it is actually
high level disinfecton.

102
Q
  1. Approximately 7-8 days after starting an antibiotic therapy for an intraoral abscess, the
    patient begins to experience severe diarrhea. If a diagnosis of antibiotic-induced colitis caused
    by clostridium difficile is made, the colitis is most likely to be treated with which of the
    following?
    A. Attapulgite
    B. Clindamycin
    C. Loperamide
    D. Metronidazole
A

The correct answer is D. Pseudomembranous colitis is caused by a necrolytic toxin produced by
Clostridium difficile, which can proliferate in the bowel secondary to the use of antibiotics that disrupt
the normal bowel flora. Almost all antibiotics (but most commonly clindamycin and ampicillin) can
cause this disorder, which results in profuse watery or bloody diarrhea. The most effective treatment
for pseudomembranous colitis is vancomycin or metronidazole. Metronidazole is a bactericidal agent
that is indicated for the treatment of serious infections caused by anaerobic bacteria and is active
against most protozoa. It is primarily used for the treatment of amebiasis and trichomoniasis in
outpatient settings and for prophylactic use in colorectal, gynecologic, and abdominal surgery. The
use of antidiarrheal agents, such as attapulgite (choice A) and loperamide (choice C), is
contraindicated. Clindamycin (choice B) is a bacteriostatic antibiotic used primarily in the treatment of
many different anaerobic infections. This is one of the primary antibiotics implicated in the
development of pseudomembranous colitis, and so will not be used to treat it.

103
Q
104. What is the treatment of choice for an infection with Cryptococcus neoformans?
A. Amphotericin B
B. Isoniazid
C. Clotrimazole
D. Metronidazole
E. Nystatin
A

The correct answer is A. Amphotericin B is the most appropriate drug listed for the treatment of
cryptococcal meningitis. It is a polyene antibiotic that binds to ergosterol in the fungal cell membrane,
41
creating an artificial pore. Flucytosine is often prescribed as an adjunct medication. Fluconazole is
used long term to prevent recurrence in AIDS patients. These are all systemic antifungal. Isoniazid
(choice B) inhibits the biosynthesis of mycolic acids in the mycobacterial cell wall. It is the primary
drug used against tuberculosis. It is used alone for TB prophylaxis and in combination with other
antituberculars to treat patients with active disease. Clotrimazole (choice C) is a typical antifungal
that, like nystatin, is used primarily in the treatment of oral candidiasis. It is from the imidazole class of
antifungals. Metronidazole (choice D) is an antiprotozoal drug useful in treating a variety of parasitic
infections. It is the drug of choice for trichomoniasis and giardiasis and provides general anaerobic
coverage. Nystatin (choice E) is an antifungal polyene that is usually used topically but can be taken
orally for oral and esophageal candidiasis. Candidal infections of the skin, mucous membranes, and
vagina usually respond well to this drug. It may also be used to prevent intestinal fungal overgrowth in
patients on chemotherapy.

104
Q
105. Which of the following terms describes a different condition from all of the others?
A. ARC
B. AIDS prodrome
C. Pre-AIDS
D. AIDS
E. AIDS-related complex
A

The correct answer is D. The disease caused by HIV virus progresses in stages that vary greatly in
length from individual to individual. HIV+ individuals early in the disease are often asymptomatic and
otherwise healthy. The HIV test shows only exposure to the virus and formation of antibody (the test
tests for antibody, not virus). This asymptomatic stage may continue for several years. Much later,
AIDS defining conditions (e.g., Kaposi sarcoma, cryptococcal meningitis, PCP pneumonia) combined
with HIV+ blood test, combine to give the diagnosis of AIDS. In between however, an early
symptomatic stage is known as ARC (AIDS-related complex), AIDS prodrome or pre-AIDS. Oral
candidiasis, night sweats, and weight loss are common at this stage.

105
Q
  1. Which of the following is the first antibody that an infant makes and is characteristically
    produced during the primary immune response?
    A. Immunoglobulin (Ig)G
    B. IgM
    C. IgA
    D. IgE
    E. IgD
A

The correct answer is B. IgM, produced during the immune response and is the first antibody that an
infant makes, is the most efficient immunoglobulin at activating complement. IgG (choice A) is
produced during the secondary immune response and is the only maternal immunoglobulin to cross
the placenta. IgA (choice C) is the main immunoglobulin in external secretions, such as saliva and
tears. It also protects mucosal surfaces. IgE (choice D) mediates immune hypersensitivity reactions
and is important in providing immunity against some parasites. IgD (choice E) is present in the
membrane of mature B cells and functions in antigen recognition by B cells.

106
Q
  1. An antibiotic, such as penicillin, which modifies cell wall synthesis, tends to be most
    effective during which phase of bacterial growth in a closed system?
    A. Lag phase
    B. Log phase
    C. Phase of decline
    D. Stationary phase
A

The correct answer is B. Penicillin is a bactericidal agent with broad gram-negative and grampositive
coverage. This agent inhibits the biosynthesis of cell wall mucopeptides. Bacterial growth in a
closed system is characterized by four phases: (1) In the initial lag phase, no growth occurs as the
organisms adapt to the new environment. (2) In the exponential, or log phase, the organisms grow at
the fastest rate and antibiotics that interfere with cell growth or division are most likely to be effective.
(3) In the stationary phase, when nutrients have been largely exhausted, organisms tend to stop
growing but may remain viable for long periods of time. (4) In the phase of decline, cell deaths
increase due to cell starvation or exposure to toxins.

107
Q
  1. A child with sickle cell anemia states that she has been feeling very tired lately, and then
    states that she may have “come down with a virus.” The girl is very pale except for her red
    cheeks, and a complete blood count shows severe anemia. A bone marrow aspirate contains
    no erythroid precursor cells. The girl was probably infected with which of the following
    viruses?
    A. Coxsackie virus
    B. Echovirus
    C. Hepadnavirus
    D. Herpes virus
    E. Parvovirus
A

The correct answer is E. Patients with sickle cell anemia typically present with a fiery red “slapped
cheek” appearance. Parvoviruses are small single-stranded DNA viruses, of which only serotype B19
is pathogenic for humans. This virus causes three distinct syndromes: a childhood febrile rash known
as erythema infectiosum (“Fifth disease”); aplastic crisis in individuals with chronic hemolytic diseases
(e.g., sickle cell anemia, thalassemia); and congenital infections that can present as stillbirth, hydrops
fetalis (analogous to severe Rh incompatibility), or severe anemia. In immunosuppressed patients and
those with sickle cell disease, anemia caused by red cell hypoplasia occurs as a consequence of
binding to erythrocyte P antigen. Coxsackie viruses (choice A) usually cause cold-like illness, but can
cause herpangina, myocarditis, and meningitis. Echoviruses (choice B) can infect a variety of organ
systems (gastrointestinal, CNS, eyes, heart, respiratory, skin), but are not a cause of aplastic crises.
Hepadnavirus (choice C) is the causative agent of hepatitis B. Herpes viruses (choice D) cause a
variety of acute to chronic infections, including herpes simplex types I and II, chicken pox, chronic
herpes zoster, cytomegalovirus infection, and Epstein Barr virus infections.

108
Q
  1. A baby born at 32 weeks gestation developed a fever and leukocytosis; lumbar puncture
    revealed pleocytosis with increased protein, decreased glucose, and gram-positive rods.
    Which one of the following organisms was most likely isolated from the CSF?
    A. Escherichia coli
    B. Listeria monocytogenes
    C. Neisseria meningitidis
    D. Streptococcus agalactiae
    E. Streptococcus pneumoniae
A

The correct answer is B. The three major causes of neonatal meningitis are group B streptococci
(Streptococcus agalactiae; choice D), Escherichia coli (choice A), and Listeria monocytogenes. All
can be found in the vaginal tract of normal women and may contaminate the infant during passage
through the birth canal. They colonize the upper respiratory tract and can cause pneumonia,
septicemia, and/or meningitis in the neonate. They are readily distinguished on morphologic grounds;
the streptococci are gram-positive cocci in chains, E. coli is a gram-negative rod, and L.
monocytogenes is a gram-positive pleomorphic rod. There are other gram-positive rods that resemble
Listeria (e.g., the diphtheroid bacilli found in the upper respiratory tract and on the skin), hence a
motility test is done to confirm the identification. L. monocytogenes is also associated with drinking
unpasteurized milk. Neisseria meningitidis(choice C) is the most common cause of meningitis in
school-age children and young adults. It is a fastidious, non-motile, gram-negative diplococcus that
would be a very rare cause of meningeal disease in very young patients, such as this one.
Streptococcus pneumoniae(choice E) is a gram-positive coccus that grows in pairs and short chains.
It is the number one cause of pneumonia, septicemia, and meningitis in the elderly. There is a vaccine
composed of the capsular carbohydrate of 23 serotypes of this organism that is routinely given to
individuals over the age of 60, as well as to individuals with splenic abnormalities (e.g., sickle cell
disease) who are at increased risk for the development of pneumococcal sepsis.

109
Q
  1. A hospitalized burn patient has developed a severe pneumonia caused by a gramnegative,
    nonfermenting rod. The organism produces a blue-green pigment on growth media
    and has a grape-like fruity odor. The organism isolated is most likely
    A. Escherichia coli
    B. Klebsiella pneumoniae
    C. Legionella pneumophila
    D. Pseudomonas aeruginosa
    E. Moratella catarrhalis
A

The correct answer is D.Pseudomonas aeruginosa is a common opportunist in burn patients, in
whom it classically causes secondary wound infections and septicemia. Pseudomonal infections are
also commonly seen in patients on ventilators. It may also cause cystitis in patients with urinary
catheters and pneumonia in patients with cystic fibrosis. The organism is found in water and usually
gains access to the body as a contaminant in the water used in respirators or in water baths used to
cleanse wounds. This organism is a nonfermenter, that is, it does not metabolize sugars by classic
pathways. It produces a blue-green, water-soluble pigment (pyocyanin), and has a fruity odor when
grown on laboratory media. Escherichia coli(choice A) is a lactose-fermenting, gram-negative rod
commonly seen as normal flora of the intestine. It is the most common cause of urinary bladder
infections, pyelonephritis, and sepsis in patients with indwelling urinary catheters. It is also the major
cause of traveler’s diarrhea and is a very important pathogen in neonates who become infected during
passage through the birth canal. Klebsiella pneumoniae(choice B) is a gram-negative, highly
encapsulated rod that is a significant pulmonary pathogen in individuals with a respiratory
compromise. It is a common cause of aspiration pneumonia and pulmonary abscesses in alcoholics
and patients with chronic obstructive pulmonary disease. Legionella pneumophila(choice C) is a
fastidious, gram-negative respiratory pathogen that may cause either a fulminating disease or a mild
“walking pneumonia-like” condition (i.e., an atypical pneumonia). It usually affects older or debilitated
patients. Moratella catarrhalis (choice E) is generally seen in patients with preexisting lung disease,
elderly patients, and immunosupressed patients. It is a gram-negative diplococcus that produces a
patchy infiltrate with occasional lobar consolidation.

110
Q
  1. debilitated 72-year-old woman develops dry cough, fever, headache, and muscular pains.
    She treats herself with ampicillin without any improvement. Chest x-ray films reveal scattered
    opacities, suggestive of interstitial infiltration. Laboratory investigations demonstrate the
    presence of cold agglutinins. She is treated with erythromycin, and her symptoms rapidly
    improve. Which of the following is the most likely etiologic agent of this condition?
    A. Influenza virus
    B. Mycoplasma pneumoniae
    C. Pneumocystis carinii
    D. Respiratory syncytial virus
    E. Streptococcus pneumoniae
A

The correct answer is B. The patient’s clinical presentation is typical of primary atypical pneumonia.
Gram stains will often show PMN’s and monocytes. No bacteria is seen. In contrast to bacterial
pneumonia, primary atypical pneumonia presents with the following features: - Caused by M.
pneumoniae; less frequently by viruses (influenza, respiratory syncytial virus, adenovirus,
rhinoviruses, rubeola and varicella virus), Chlamydia, or Coxiella burnetii - Characterized
pathologically by interstitial, rather than intra-alveolar, inflammation - Characterized clinically by
nonspecific symptomatology and few “localizing” symptoms. Why is M. pneumoniae, and not
influenza virus (choice A) or respiratory syncytial virus (choice D), the cause of this patient’s
pneumonia? First, M. pneumoniae infections are often associated with the appearance of cold
agglutinins in the serum, detection of which is diagnostically important. Second, the patient responded
quickly to treatment with erythromycin, an antibiotic effective against M. pneumoniae, but obviously
not effective in treating viral infections. As a side note: the macrolides, such as erythromycin are the
drugs of choice for atypical pneumonia. Pneumocystis carinii(choice C) is a fungal organism causing
pneumonia in severely immunocompromised hosts, especially AIDS patients. P. carinii pneumonia
(PCP) is characterized by accumulation of a frothy exudate containing numerous organisms within
alveolar spaces. Also, P. carinii is not sensitive to erythromycin. The drug of choice for treatment of
PCP is trimethoprim-sulfamethoxazole. Streptococcus pneumoniae(choice E) is the usual causative
agent of lobar pneumonia, characterized by consolidation of a single lobe due to intra-alveolar acute
inflammatory exudation. Lobar pneumonia is more prevalent in young, healthy individuals, whereas
primary atypical pneumonia favors old, debilitated patients. S. pneumoniae is highly sensitive to
penicillin.

111
Q
  1. Microscopic examination of a urine sample demonstrates many gram-negative rods,
    many neutrophils, and a few white blood cell (WBC) casts. Which of the following is the
    specific significance of the presence of WBC casts?
    A. One or both kidneys are involved in the infection
    B. One or both ureters are involved in the infection
    C. The bladder is involved in the infection
    D. The urethra is involved in the infection
    E. The uterus is involved in the infection
A

The correct answer is A. The patient described has a urinary tract infection. The presence of white
blood cell casts specifically implies renal involvement, because these form when the leukocytes are
compressed together in the renal tubules. Similarly, the presence of red blood cell casts in a bloody
urine would indicate that at least some of the bleeding was occurring in the kidney. White cell casts
are seen in pyelonephritis and interstitial nephritis and are indicative of infection or inflammation. Note
also that the presence of bacteria merely indicates infection somewhere in the excretory system.
Infection of other sites in the urinary tract, including ureters (choice B), bladder (choice C), and
urethra (choice D), does not cause cast formation. Although spread of a urinary tract infection to the
uterus (choice E) is always of concern in a pregnant woman, the presence of casts does not
specifically suggest that this has occurred.

112
Q
  1. A 3-month-old infant presents with a 3-day history of fever, cough, and poor feeding. The
    infant seems ill and has a temperature of 102 F and a respiratory rate of 32. A chest x-ray film
    shows bilateral patchy infiltrates in the lungs. Which of the following is the most likely
    etiologic agent?
    A. Coronavirus
    B. Influenza type A
    C. Parainfluenza type 1
    D. Respiratory syncytial virus
    E. Rhinovirus
A

The correct answer is D. Respiratory syncytial virus is the most common cause of bronchiolitis,
tracheobronchitis, and pneumonia in children younger than 1 year of age. The average incubation
period is 5 days. Outbreaks occur seasonally in winter and early spring. Infection does not result in
lasting immunity, and reinfection can occur. In bronchiolitis, proliferation and necrosis of bronchiolar
epithelium develop, producing obstruction from sloughed epithelium and increased mucus secretion.
Signs often include fever, tachypnea, and wheezes. Hyperinflated lungs and otitis media are frequent
conditions. Coronavirus (choice A) causes approximately 20% of common colds (nasal obstruction
and discharge, sneezing, no fever or mild fever, occasional sore throat, and cough) and acute
pharyngitis (sore throat, with or without cervical adenopathy, ulceration, and conjunctivitis). Influenza
type A (choice B) is the leading cause of influenza. Influenza is a systemic illness characterized by
the sudden onset of fever, headache, myalgias, malaise, and prostration, followed by cough, nasal
obstruction, and sore throat. The lower respiratory tract may also be involved. Parainfluenza viruses
(choice C) are the leading cause of croup, or acute laryngotracheobronchitis, in children. This
infection involves the upper and lower respiratory tracts. Inflammation in the subglottal area leads to
hoarseness, dyspnea, a barking cough, and inspiratory stridor. Rhinovirus (choice E) is the most
common cause of the common cold.

113
Q
114. Which of the following substances takes part in the nonoxidative killing pathway by
which pathogenic bacteria can be killed?
A. Hydrogen peroxide
B. Hypochlorous acid
C. Lysozyme
D. Myeloperoxidase
E. Superoxide ions
A

The correct answer is C. This is a microbiology question that requires knowledge about the
physiology of bacteria. Lysozyme is present in tears, saliva, mucus, vaginal secretions, and several
other body fluids. This material lyses the peptidoglycan layer of the cell wall of bacteria without
participation of any of the elements of the oxidative killing pathway. Hydrogen peroxide (choice A) is
formed in the oxidative killing pathway from the superoxide ions. This material is also very toxic to
bacterial pathogens. Hypochlorous acid (choice B) is formed from hydrogen peroxide and halide ions
in the presence of myeloperoxidase. Myeloperoxidase (choice D) is an enzyme present in the
oxidative killing pathway that breaks down hydrogen peroxide. Superoxide ions (choice E) are formed
by NADPH oxidase in the first reaction of the oxidative killing pathway. These superoxide ions are
extremely toxic to bacterial organisms.

114
Q
115. A 65-year-old man undergoes coronary artery bypass surgery that entails grafting the left
internal mammary artery to the left main coronary artery to form an alternative conduit for
blood flow. This patient received a(n)
A. allograft
B. autograft
C. homograft
D. isograft
E. xenograft
A

The correct answer is B. Tissue grafts in which the same individual acts as both the donor and the
recipient are termed autografts. Allografts (choice A) refer to tissue transplants from one person to
another. Homograft (choice C) is a synonym for allograft. Isografts (choice D) refer to tissue
transplants between genetically identical individuals (e.g., an identical twin donates her kidney to her
twin sister). Xenografts (choice E) refer to tissue transplants from another species (e.g., baboon heart
transplanted into a human).

115
Q
  1. A 10-year-old boy is attending summer camp, complains of a sore throat, headache,
    cough, malaise, a low-grade fever, patchy exudate, and keratoconjunctivitis. Within hours,
    several other campers show similar symptoms. The most likely pathogen involved is:
    A. Adenovirus
    B. Chlamydia spp
    C. Gram-negative diplococcus
    D. Gram-positive enterococcus
    E. Herpesvirus
A

The correct answer is A. There are more than 40 antigenic types of adenoviruses that produce a
variety of symptoms. The incubation period is 4-9 days. Adenoviruses are nonenveloped (naked),
icosahedral DNA viruses causing a variety of clinical syndromes. Adenoviruses cause a
pharyngoconjunctivitis that affects children and sometimes adults who are living in the same
household. Contaminated swimming pools have been implicated as sources for the virus. The virus is
latent in the lymphoepithelial tissue of the nasopharynx and other sites. Adenoviruses also cause
watery, nonbloody diarrhea. Fever often lasts 2-12 days and is accompanied by malaise and myalgia.
Sore throat is often manifested by a patchy exudate and cervical lymphadenopathy. Conjunctivitis is
often present. Chlamydia spp. (choice B) produce a variety of clinical syndromes, including a sexually
transmitted urethritis, pelvic inflammatory disease, neonatal pneumonia and inclusion conjunctivitis,
lymphogranuloma venereum, adult interstitial pneumonia, and a zoonotic pneumonitis. A gramnegative
diplococcus (choice C) would be a Neisseria spp. or Moraxella. Neisseria meningitidis would
be the logical choice, because it is associated with outbreaks among children. The clinical syndrome
caused by infection with the meningococcus, however, is one of a fulminating, progressive septicemia
or meningitis with fever, vascular collapse, and disseminated intravascular coagulation. N.
gonorrhoeae does not cause a clinical syndrome as described in the question. Moraxella catarrhalis
causes pneumonia in patients with chronic obstructive pulmonary disease. Gram-positive
enterococcus (choice D) would refer to Group D streptococcus. These organisms are associated with
endocarditis and genitourinary infections. Herpesviruses (choice E) are large, enveloped DNA viruses
with an icosahedral shape. Possible diagnoses for this patient include Epstein-Barr virus, producing
infectious mononucleosis, or cytomegalovirus, producing a mononucleosis-like syndrome. The
patients in the question did not have hepatosplenomegaly (characteristic of mononucleosis), but did
have keratoconjunctivitis. The organism causing the outbreak in the question also has a higher level
of infectivity and a short incubation time.

116
Q
  1. A 70-year-old nursing home resident develops pneumonia. Examination of sputum
    demonstrates many neutrophils and many lancet-shaped gram-positive cocci in pairs. Which
    of the following organisms is the most likely cause of the patient’s pneumonia?
    A. Haemophilus influenzae
    B. Mycobacterium tuberculosis
    C. Mycoplasma pneumoniae
    D. Staphylococcus aureus
    E. Streptococcus pneumoniae
A

The correct answer is E. Pneumonia can be caused by a large variety of agents including bacteria,
Mycoplasma, fungi, and viruses. Streptococcus pneumoniae, or Pneumococci are classically
described as lancet-shaped gram-positive cocci in pairs, or diplococci. Historically, this organism was
specifically associated with lobar pneumonia, but with modern rapid antibiotic therapy, a
bronchopneumonia pattern may also be seen. Pneumococcal pneumonia remains common in
community and hospital-acquired settings. Complications include bacteremia, meningitis,
endocarditis, pericarditis, and empyema. Haemophilus influenzae(choice A) is a gram-negative rod,
more specifically a pleomorphic gram-negative coccobacilli. Mycobacterium tuberculosis(choice B) is
an acid-fast rod. Mycoplasma pneumoniae(choice C) is an intracellular organism and an atypical
bacteria. Gram stains typically show PMNs and monocytes but no bacteria. Staphylococcus
aureus(choice D) is a gram-positive coccus that grows in grape-like clusters. Complications include
empyema and endocarditis.

117
Q
  1. Bilateral tonsillectomy is performed on an 11-year-old female with recurrent upper
    respiratory tract infections and trismos. On sectioning the tonsils, numerous small, yellow
    granules are noted which have dense, gram-positive centers and numerous branching
    filaments at the periphery. The granules are most likely composed of which of the following
    organisms?
    A. Actinomyces israelii
    B. Aspergillus fumigatus
    C. Blastomyces dermatitidis
    D. Candida albicans
    E. Corynebacteria diphtheriae
A

The correct answer is A. The patient is presenting with cervicofacial actinomycosis. This type of
infection commonly follows a tooth extraction or other trauma. Actinomyces are normal inhabitants of
the gastrointestinal tract that grow under anaerobic and microaerophilic conditions. Although they are
gram-positive rods, they grow as branching filaments and have been confused with fungi. The yellow
colonies (“sulfur granules”) are found in low-oxygen niches like the tonsils and in actinomycotic
abscesses. Cervicofacial actinomycosis often develops slowly. The affected area becomes markedly
indurated and the overlying skin becomes reddish or cyanotic. Abscesses draining to the surface may
also present. There is little pain unless a secondary infection sets in. Trismus indicates that the
muscles of mastication are involved. Aspergillus fumigatus (choice B) may be present in the
respiratory tract as an opportunistic pathogen; however, fungus balls are generally seen only in preexisting
cavities (e.g., bronchiectasis, TB), not in the tonsils. Blastomyces dermatitidis (choice C) is a
respiratory pathogen that is seen as thick-walled yeasts within granulomas. Candida albicans (choice
D), also a normal inhabitant of the oral cavity, would present as whitish plaques and would appear
microscopically as budding yeasts. When these white plaques are removed, they reveal a painful
bleeding lesion. Diphtheria, caused by Corynebacteria diphtheriae(choice E), is a gram-positive rod.
The disease presents with a gray-white membrane in the oropharynx, and large colonies would not be
noted.

118
Q
  1. A 4-year-old child has repeated infections with staphylococci and streptococci, but has
    normal phagocytic function and delayed hypersensitivity responses. Lymph node biopsy
    would most likely reveal
    A. absence of postcapillary venules
    B. absence of germinal centers
    C. defective chemotactic response of neutrophils
    D. depletion of paracortical areas
    E. depletion of thymus-dependent areas
A

The correct answer is B. The combination of repeated staphylococcal infections, normal phagocytic
function, and normal delayed hypersensitivity responses is suggestive of Bruton
hypogammaglobulinemia. The disease is characterized by an IgG level less than 100 mg/dL, other
immunoglobulins deficient or absent, B cells deficient or absent, intact cellular (T-cell) immunity, and
onset of infections after the sixth month of life, when maternal antibodies are no longer present. Bcell-
dependent areas, therefore, such as germinal centers, would be absent or greatly diminished in
size and number. Neither postcapillary venules (choice A) nor thymus-dependent areas (choice E)
are depleted in this disease. The chemotactic response of neutrophils (choice C) is part of the
phagocytic function assay, which was normal in the case presented. Because T-cell functions are
intact, the architecture of the portions of lymphoid tissues where T cells reside (e.g., paracortical
areas, choice D) should be normal.

119
Q
120. A patient with tertiary syphillis has aortic regurgitation. Which of the following organisms
is the most likely culprit?
A. Haemophilus influenzae
B. Herpes simplex I
C. Neisseria gonorrhoeae
D. Neisseria meningitidis
E. Treponema pallidum
A

The correct answer is E. The findings described are those of tabes dorsalis, a form of tertiary
syphilis caused by Treponema pallidum. Tabes dorsalis and other forms of tertiary syphilis are now
47
uncommon in this country, possibly because the common use of antibiotics may “treat” many
unsuspected cases of syphilis. Late (tertiary) syphilis is associated with infiltrative tumors of skin,
bones, and liver (gummas). Aortic regurgitation and CNS disorders (including meningovascular and
degenerative changes), shooting pains, and abnormal reflexes are seen. Haemophilus influenzae
(choice A) and Neisseria meningitidis (choice D) can cause meningitis. Neisseria gonorrhoeae
(choice C) causes gonorrhea, which usually does not involve the CNS. Herpes simplex I (choice B)
can cause an encephalitis that typically involves the frontal and temporal lobes. Cardiac and liver
involvement, however, are not seen.

120
Q
  1. A 19-year-old man presents to the emergency department with pneumonia. Since the age
    of 6 months, he has had recurrent pneumonia and sinusitis caused by Streptococcus
    pneumoniae and Haemophilus influenzae. Careful assessment of his immune function would
    likely reveal abnormal function of the
    A. B-lymphocytes
    B. macrophages
    C. natural killer cells
    D. platelets
    E. T-lymphocytes
A

The correct answer is A. The symptoms in the question suggest a deficiency known as common
variable hypogammaglobulinemia, characterized by very low serum levels of IgG. In this congenital
disease, the number of B cells is normal, but their ability to synthesize IgG and the other
immunoglobulins is severely compromised, leading to recurrent streptococcal and Haemophilus
infections. The onset of the recurrent infections usually begins between 6 and 12 months of age,
concurrent with the decreasing levels of maternal IgG in the newborn. Treatment often involves giving
intravenous gamma-globulin to reduce the number of infections. There are no known diseases or
infections associated with deficiencies in macrophages (choice B) or natural killer (NK) cells (choice
C). Furthermore, one would not expect a deficiency in macrophage or NK cell function to present as
recurrent bacterial infections, because these cells are primarily involved in cellular, rather than
humoral, immunity. Platelets (choice D) are not directly involved in the immune response.
Deficiencies in platelets lead to problems in clotting and produce bleeding disorders. T-cell
deficiencies (choice E) result in severe viral, fungal, and protozoal infections, rather than recurrent
bacterial infections.

121
Q
  1. Which of the following viruses produces disease or sequelae that is/are more severe if
    the infection occurs at a very young age?
    A. Epstein-Barr virus
    B. Hepatitis B virus
    C. Measles virus
    D. Poliovirus
    E. Varicella zoster virus
A

The correct answer is B. Infection with hepatitis B virus (HBV) at birth or a very young age is
associated with chronic HBV infection and the development of hepatocellular carcinoma later in life. In
fact, infants born to hepatitis B surface antigen (HBsAg)-positive mothers are commonly infected, and
approximately 90% become chronic carriers of the virus. In chronic carriers, hepatocellular carcinoma
develops at an incidence more than 200 times higher than in noncarriers. The current
recommendation for infants born of HBsAg-positive mothers is administration of hepatitis B
immunoglobulin (HBIg) in the delivery room, with the first dose of the hepatitis B vaccine given at the
same time or within 1 week. The second and third doses of the vaccine are then given at 1 and 6
months. With this protocol, 94% protection is achieved. The Epstein-Barr virus (EBV; choice A) is the
agent of heterophile-positive infectious mononucleosis. In children, primary EBV infection is often
asymptomatic. The measles virus (choice C) often causes a more severe disease in adults. The
incidence of complications, including pneumonia, bacterial superinfection of the respiratory tract,
bronchopspasm, and hepatitis, is much higher in adults older than 20 than in children. Poliovirus
(choice D) causes asymptomatic or inapparent infections 95% of the time. Frank paralysis occurs in
approximately 0.1% of all poliovirus infections. However, the probability of paralysis increases with
increasing age. Varicella zoster virus (choice E) is the agent of chickenpox and shingles. In
immunocompetent children, it is a benign illness with a mortality rate of less than 2 per 100,000
cases. This risk is increased more than 15-fold in adults. Much of the increase is due to varicella
pneumonitis, a complication that occurs more frequently in adults.

122
Q
  1. A 22-year-old woman comes to the sexually transmitted disease (STD) clinic for her first
    visit. She tells the nurse practitioner that she has had four different sexual partners in the last
    6 months and only one of them used a condom. She also admits that she used IV drugs on
    several occasions 2 years ago. She notes fever, weight loss, lack of appetite, and periodic
    difficulty breathing over the past few months. She has an HIV test performed, which is
    positive. The physician decides to do a
    A. ELISA (enzyme-linked immunosorbent assay)
    B. FACS (fluorescence activated cell sorting)
    C. RAST (radioallergosorbent test)
    D. RID (radial immunodiffusion)
    E. Western blot
A

The correct answer is E.Because of the prevalence of AIDS, it is essential that the dental student be
familiar with these concepts. The Western blot is the most appropriate test for confirmation of HIV
infection. It identifies several different antibodies against HIV (anti-gp120, -gp41, -p24, and -p17). The
Western blot is generally performed when an ELISA test is positive. The initial HIV test this patient
had was most likely an ELISA. The ELISA (choice A) can be used to detect p24 antibody in the
patient, but is not as specific as the Western blot. The ELISA test is the standard test used in the
screening of HIV infections. Fluorescence activated cell sorting (choice B) is a technique used to
separate and count specific numbers and types of cells in a sample. An example of this would be to
count the number of B cells and T cells in a specific blood sample. RAST testing (choice C) is used to
determine the level of specific IgE present in a patient that reacts with a specific allergen that has
been applied to a disk or glass bead. Radial immunodiffusion (choice D) is an excellent test used for
quantitation of immunoglobulin levels in patients. This is used to determine the IgG, IgM, and IgA
levels in patient’s serum.

123
Q
  1. A one-week-old baby develops nuchal rigidity and fever. A lumbar puncture is performed
    and the cerebrospinal fluid demonstrates large numbers of neutrophils. The glucose level of
    the CSF is lower than normal. If a diagnosis of meningitis is made, which of the following is
    the most likely causative agent, taking into account the patient’s age?
    A. Coxsackievirus
    B. Escherichia coli
    C. Herpes virus
    D. Mycobacterium tuberculosis
    E. Neisseria meningitidis
A

The correct answer is B. The nuchal rigidity (stiff neck) suggests meningitis. Numerous neutrophils
in the cerebrospinal fluid suggest a bacterial pathogen. Furthermore, low CSF glucose is common in
bacterial meningitis. The best answer of those listed is Escherichia coli, which is normally a gut
organism, but can infect neonates who acquire the organism during passage through the birth canal.
The common characteristics of bacterial meningitis include: elevated polymorphonuclear neutrophils,
low CSF glucose, elevated CSF protein and markedly elevated opening pressure. Coxsackievirus
(choice A) is a cause of acute lymphocytic meningitis. Herpes virus (choice C) is a cause of
lymphocytic meningitis. Mycobacterium tuberculosis(choice D) is a cause of chronic meningitis and is
the causitive pathogen in tuberculosis. Neisseria meningitidis(choice E) causes bacterial meningitis,
usually in the second or third decade of life. It is rare to see this bacteria in infants.

124
Q
  1. Ciprofloxacin is believed to exert its antibacterial effect by which of the following
    mechanisms?
    A. Competitive inhibition of para-amino-benzoic acid
    B. Inhibition of bacterial cell wall synthesis
    C. Inhibition of DNA-gyrase
    D. Irreversible binding to the 30S subunit of bacterial ribosomes
    E. Irreversible binding to the 50S subunit of bacterial ribosomes
A

The correct answer is C. NBDE Part 1 questions often involve antibiotic mechanisms.
Fluoroquinolone antibiotics, such as ciprofloxacin and lomefloxacin, inhibit DNA-gyrase, which is an
enzyme necessary for bacterial DNA replication and repair. The fluoroquinolones are generally not
used for dental procedures because they have very poor anaerobic coverage. They have very good
49
gram-negative coverage, however. These agents are commonly used in the treatment of upper and
lower respiratory infections and in genitourinary tract infections. Sulfonamide antibiotics exert their
antibacterial effect through the competitive inhibition of para-amino-benzoic acid (PABA) (choice A),
thereby inhibiting folic acid biosynthesis required for bacterial growth. Cephalosporins (cephalexin,
cefuroxime, and cefixime) are believed to exert their antibacterial effect through the inhibition of
bacterial cell wall synthesis (choice B). These agents bind to one or more of the penicillin-binding
proteins located on the cell walls of susceptible organisms which results in the inhibition of the third
and final stage of bacterial cell wall synthesis. These effects account for the bactericidal effect of
cephalosporins and their cousins the penicillins. Aminoglycosides, such as gentamicin and
streptomycin, irreversibly bind the 30S subunit of bacterial ribosomes (choice D), which inhibits
bacterial protein synthesis. Lincosamides, such as clindamycin, irreversibly bind to the 50S subunit of
bacterial ribosomes (choice E), which suppresses bacterial protein synthesis. Note: macrolides, such
as erythromycin, also bind to the 50S subunit of bacterial ribosomes, which suppresses bacterial
protein synthesis.

125
Q
  1. If a dentist were to administer a bacteriostatic antibiotic, such as a macrolide, and a
    bactericidal antibiotic, such as a cephalosporin, concomitantly to a patient with a severe
    intraoral infection, the most likely pharmacological result for the patient would be which of the
    following?
    A. A small extension of bacterial coverage between the two agents
    B. Antagonism of the antibacterial effects of both agents
    C. Profound toxicity for the patients taking both agents
    D. Synergism of the antibacterial effects of both agents
    E. The appearance of mild antifungal activity
A

The correct answer is B. The concomitant administration of bacteriostatic and bactericidal antibiotics
would result in antagonism of the antibacterial effects of both agents. The reason for the antagonism
between the two is that each agent interferes with the mechanism of action of the other agent. By
definition, a bacteriostatic agent is one that prohibits the growth and development of bacteria and a
bactericidal agent is one that directly causes the death of a microbe during the growth and
development stages. An extension of bacterial coverage between two agents (choice A) is typically
seen when two bacteriostatic or two bactericidal agents are given together. The administration of a
macrolide and a cephalosporin may cause some discomfort for the patient, such as nausea or
stomach cramps; however, profound toxicity for the patients taking both agents (choice C) is very
unlikely. Synergism (choice D) is the correlated action of two agents that results in a final effect that is
greater than that of each agent acting separately. A classical example of synergism is seen when an
aminoglycoside, such as gentamicin, is administered concomitantly with extended spectrum penicillin,
such as piperacillin, for the treatment of a pseudomonas infection. The appearance of mild antifungal
activity (choice E) would not be seen when bacteriostatic and bactericidal antibiotics are given
together, if neither have any antifungal activity.

126
Q
  1. The term “biofilm” as it applies in dentistry is most often used to describe:
    A. A safer type of x-ray film
    B. a detector that protects tissue from x-ray damage
    C. the surface of dentin inside a cavity preparation
    D. the surface of dental waterlines
    E. none of the above
A

The correct answer is choice D. The problem of bacteria in dental waterlines is probably going to be
the next intensive infection control issue faced by dentistry. The narrow bore of most dental waterlines
encourages a thin film colony of bacterial slime (biofilm) to form on the inner circumference of the
waterline. Water entering the dental unit, although chlorinated at the source, is clean, but not sterile.
The microorganisms it contains can form extensive colonies inside the tubes. While no specific
disease cases have been linked to these microorganisms, it is clear that they should be minimized.
Several systems of filtration or chemical disinfection are now available. Other alternatives include
bottled irrigation water and flushing the lines for several minutes several times each day.

127
Q
128. After eating a dinner of leftovers that included rewarmed vegetable fried rice, a 17-yearold
boy develops diarrhea and stomach pain. Which of the following is the most likely
pathogen?
A. Bacillus cereus
B. Campylobacter jejuni
C. Clostridium botulinum
D. Clostridium difficile
E. Escherichia coli
A

The correct answer is A. Bacillus cereus contaminates grains, such as rice, and produces spores
resistant to quick frying and steaming. If you see the words fried rice on the NBDE, the odds are that
the correct answer is B. cereus. Campylobacter jejuni (choice B) causes enterocolitis with bloody
diarrhea, crampy abdominal pain, malaise, and fever. Clostridium botulinum (choice C) produces a
constellation of signs and symptoms, including bulbar palsy, descending weakness or paralysis,
progressive respiratory weakness, absence of fever, dry mucous membranes, and autonomic
dysfunction. Clostridium difficile (choice D) causes pseudomembranous colitis, classically resulting
from clindamycin use. Escherichia coli (choice E) comes in a variety of forms. The enterotoxigenic
type is the most common cause of traveler’s diarrhea.

128
Q
  1. A 7-year-old boy is brought to a physician because of a nearly confluent, fine,
    erythematous, macular rash that is most pronounced on his trunk. He has had a mild fever for
    36 hours, but does not seem very ill. Physical examination demonstrates a reddened throat
    with tonsillar exudates, enlarged cervical nodes including the occipital node, and questionable
    splenomegaly. The most likely cause of his condition is
    A. Bullous pemphigoid
    B. Dermatitis herpetiformis
    C. Herpes simplex
    D. Measles
    E. Rubella
A

The correct answer is E. This presentation (fine, nonblotchy, truncal rash in a not-very-ill child) is
characteristic of rubella, or German measles, that is caused by a togavirus. The rash typically spreads
from trunk to extremities. IgM specific for rubella can often be detected in serum within 1-2 days of
developing the rash. The principal significance of this disease is that it can cause a devastating
congenital infection characterized by ocular problems (cataracts, retinopathy, microphthalmos,
glaucoma), cardiovascular problems (patent ductus arteriosus, ventricular septal defect, pulmonary
stenosis), deafness, thrombocytopenic purpura, leukopenia, hepatosplenomegaly, CNS problems,
and bony lesions. Arthralgia is seen, particularly in women. Exposure occurs 14-21 days before onset.
Bullous pemphigoid (choice A) produces large, tense blisters in flexural areas that appear with
exacerbations and remissions. Dermatitis herpetiformis (choice B) causes recurrent crops of small
vesicles or papules that appear mainly on the elbows, knees, buttocks, posterior neck, and scalp.
Herpes simplex (choice C) is characterized by crops of vesicles on oral or genital sites, where herpes
simplex I causes oral and perioral manifestations and herpes simplex II causes genital lesions.
Measles (choice D) causes a blotchy, maculopapular erythematous rash that begins on the face and
spreads downward. Patients with measles are usually much sicker than those with German measles,
and Koplik spots may be seen on the buccal mucosa.

129
Q
  1. A patient with diabetes has chronic sinusitis that has not responded to a 6-week course of
    antibiotics. The physician should suspect infection with which of the following organisms?
    A. Actinomyces
    B. Aspergillus
    C. Cryptococcus
    D. Mucor
    E. Pneumocystis
A

The correct answer is D. There is a specific association between diabetes (particularly in brittle
diabetics who may have episodes of ketoacidosis) and chronic sinusitis caused by saprophytic
Zygomycetes, including Mucor and Rhizopus. These fungi can spread rapidly from the sinuses to the
nearby skull bones and brain, potentially causing massive tissue destruction and death. The term
“rhinocerebral mucormycosis” is used in these cases. Less commonly, other sites may be involved
(lung, gastrointestinal tract), depending on the port of entry. The physician should suspect
mucormycosis in any patient with chronic sinusitis who appears unusually ill and does not respond to
antibiotic therapy. Unfortunately, most cases are diagnosed at autopsy. Actinomyces (choice A) are
part of the normal flora of the mouth. Actinomycosis also occurs in humans and may affect the
cervicofacial region (typically following dental procedures or maxillofacial injuries), lungs, abdomen
(typically following surgery, trauma, or intestinal penetration), or pelvis (related to IUD use). There is
no specific association with diabetes mellitus. Aspergillus (choice B) can be present in the sinuses,
51
and does have somewhat increased incidence in patients with diabetes, but it is not the organism
about which the physician should be most concerned. Cryptococcus (choice C) is found in pigeon
feces and is usually introduced into the body by way of the respiratory tract. It can disseminiate to the
meninges and other sites in immunocompromised patients (often AIDS patients). It would not be of
particular concern in this patient with sinusitis. Pneumocystis carinii (choice E) causes pneumonia in
severely immunosupressed patient ( e.g., patients with AIDS).

130
Q
  1. A 39-year-old HIV-positive man has a seizure accompanied by loss of consciousness and
    leg and arm jerking. The patient is lethargic, unable to answer simple questions, and has an
    obvious left-sided hemiparesis. The causative organism is most likely:
    A. Cryptococcus neoformans
    B. Herpes simplex
    C. Mycobacteria tuberculosis
    D. Isospora belli
    E. Toxoplasma gondii
A

The correct answer is E. Although all five pathogens can cause central nervous system (CNS)
manifestations, toxoplasmosis (caused by Toxoplasma gondii) presents with seizures in 15-25% of
cases. It is the most common cause of secondary CNS infections in AIDS patients. The disease is
spread by ingestion of cysts from undercooked meat or from cat feces. Tachyzoites develop from
cysts phagocytized by macrophages, then spread to the brain, muscle, and other tissues, where they
encyst and multiply. Cryptococcal meningitis (choice A) usually presents as a subacute meningitis
with headache, nausea, vomiting, and confusion. Cranial nerve abnormalities are common with
cryptococcal infections. Herpes simplex encephalitis (choice B) typically has a subacute onset with
headache, meningismus, and personality changes. Mycobacterium tuberculosis(choice C) is a cause
of basilar meningitis, which can present insidiously with headache and mental changes over a week
or two, or can present acutely as confusion, lethargy, altered sensorium, and a stiff neck. Cranial
nerve palsies, focal cerebral ischemia, and hydrocephalus are characteristic. Isospora belli(choice D)
is a leading cause of diarrhea in AIDS patients. It typically occurs when the CD4 count is less than
300/uL.

131
Q
  1. An otherwise healthy patient who wears contact lenses develops a small ulceration of the
    eye. Which of the following organisms is most likely involved?
    A. Acanthamoeba
    B. Cytomegalovirus
    C. Herpes simplex
    D. Toxocara
    E. Toxoplasma
A

The correct answer is A. All the agents listed can infect the eyeball. The agent specifically
associated with contact lens use is Acanthamoeba, which can infect lens solution. This amoeba is
dangerous because it causes an intractable ulcerative keratitis that may progress to uveitis. If the
lesion is suspected, the clinical laboratory should be notified and specific directions for collecting
samples for culture obtained. The parasites may be difficult to see in histologic sections or corneal
scrapings. Cytomegalovirus (choice B) and herpes (choice C) infections are most often seen in
immunocompromised patients, particularly AIDS patients. Circulating larvae of the helminth
Toxocara(choice D) can lodge in the eye (particularly in the vitreous or retina); Toxocara infections
are seen more commonly in children. Toxoplasmosis (choice E) of the eye is most often congenital,
but it can be acquired.

132
Q
  1. A 4-year-old boy is brought to the emergency department in extreme respiratory distress,
    with a temperature of 103.8 F. He is drooling, has difficulty swallowing, and exhibits
    inspiratory stridor and swelling of the epiglottis. He has had no previous vaccinations. Which
    of the following agents is the most likely cause of these symptoms?
    A. Haemophilus influenzae
    B. Klebsiella pneumoniae
    C. Legionella pneumophila
    D. Mycoplasma pneumoniae
    E. Streptococcus pyogenes
A

The correct answer is A. Epiglottitis is the most common disease of the upper respiratory tract
produced by Haemophilus influenzae type b, a gram-negative encapsulated rod. Epiglottitis is
characterized by an abrupt onset of high fever, drooling, and inability to handle secretions. Stridor and
respiratory distress result from laryngeal obstruction. The epiglottis is described as being cherry red
and swollen. H. influenzae is also a common cause of otitis media in children and may cause
bronchitis, bronchiolitis, and pneumonia in adults. The incidence of serious disease caused by
Haemophilus influenzae type b decreased greatly with the introduction of an effective vaccine, which
is composed of the H. influenzae type b capsular polysaccharides coupled to a carrier molecule and
given to children from 2-15 months of age. The patient had not received the Hib conjugate vaccine
and therefore was susceptible to this organism. Klebsiella pneumoniae(choice B) causes pneumonia
and pulmonary abscesses, but is not considered to be a pathogen in the upper respiratory tract.
Klebsiella is a gram-negative encapsulated rod. Legionella pneumophila(choice C) causes
pneumonia in humans. The disease may be mild (an atypical pneumonia) or a fulminating disease
with high mortality (30%). On a gram stain, one would see no bacteria because it is an “atypical”
organism. Mycoplasma pneumoniae(choice D) causes community-acquired atypical pneumonia. It is
the most common cause of pneumonia in young adults. On a gram stain, one would see no bacteria
because it is an “atypical” organism. Streptococcus pyogenes(choice E) is the most common cause
of pharyngitis; however, this patient’s presentation strongly suggests epiglottitis. It is a gram-positive
organism.

133
Q
134. Which of the following is an example of type II hypersensitivity?
A. Allergic rhinitis
B. Erythroblastosis fetalis
C. Food allergy
D. Serum sickness
E. Tuberculosis
A

The correct answer is B. Type II hypersensitivity occurs when antibodies react with antigens present
on the surface of cells or other tissue components. It is conveniently subclassified into diseases
produced by three distinct mechanisms. One mechanism involves complement fixation by the
antibody-antigen complex, which facilitates phagocytosis; this mechanism occurs in transfusion
reactions, erythroblastosis fetalis, autoimmune hemolytic anemia, agranulocytosis, or
thrombocytopenia. In a second form of type II hypersensitivity, target cells coated with low levels of
IgG are lysed (without phagocytosis) by monocytes, neutrophils, or natural killer cells; this mechanism
is thought to operate in the destruction of large parasites, possibly some tumor cells, and in graft
rejection. The third mechanism involves antibody-mediated cellular dysfunction, such as occurs in
Graves disease or myasthenia gravis. Allergic rhinitis (choice A) is an example of type I (allergic or
anaphylactic) hypersensitivity. Food allergy (choice C) is an example of type I (anaphylactic)
hypersensitivity. Serum sickness (choice D) is an example of type III (immune complex diseases)
hypersensitivity. Tuberculosis (choice E) is an example of type IV (cell-mediated or delayed)
hypersensitivity

134
Q
  1. A gram stain of the sputum from a patient with lobar pneumonia involving the left lower
    lobe demonstrates gram-positive, encapsulated, lancet-shaped diplococci. Which of the
    following is the most probable causative organism?
    A. Haemophilus influenzae
    B. Neisseria gonorrhoeae
    C. Pneumocystic carinii
    D. Staphylococcus aureus
    E. Streptococcus pneumoniae
A

The correct answer is E. This is the classic microscopic description of the pneumococcus
Streptococcus pneumoniae, which is a common cause of lobar pneumonia. Most strains are still
sensitive to penicillins, although some drug-resistant strains have been isolated. Haemophilus
influenzae (choice A) is a gram-negative bacillus. Atypical strains may cause pneumonia in elderly
patients with chronic respiratory disease. Neisseria gonorrhoeae (choice B) is a gram-negative
diplococcus that is not typically associated with pneumonia. Pneumocystis carinii (choice C) is a
small, hat-shaped fungus that is a common cause of pneumonia in HIV-positive
patients.Staphylococcus aureus (choice D) occurs as grapelike clusters of large, gram-positive cocci.
It may cause pneumonia after surgery or after a viral respiratory infection, such as influenza, and is
associated with empyema formation.

135
Q
  1. Global eradication of Lyme disease is unlikely in the foreseeable future because Borrelia
    burgdorferi
    A. can be maintained in nature indefinitely by a tick vector
    B. has a tough outer coat that is resistant to environmental stresses
    C. has humans as its primary reservoir
    D. is resistant to antibiotics and disinfectants
    E. may reactivate and cause Brill-Zinsser disease
A

The correct answer is A.Borrelia burgdorferi, the tick-transmitted spirochete that causes Lyme
disease, can be maintained in nature indefinitely by a tick vector. The tick, Ixodes dammini, can infect
white-footed mice and large mammals such as deer during its life cycle, making these animals into
reservoirs. The tick itself, however, is a reservoir because it acquires the disease through transovarial
passage of the organism. Together these factors make Lyme disease an endemic infection with little
hope for eradication. B. burgdorferi is a delicate spirochete that is vulnerable to several chemical and
physical agents. It does not have a tough outer coat (unlike choice B). Humans are incidental hosts,
not primary reservoirs (choice C), for B. burgdorferi. The primary reservoirs are ticks, mice, and large
mammals. B. burgdorferi can be successfully treated with penicillins, tetracycline, and ceftriaxone
(unlike choice D). Brill-Zinsser disease (choice E) is actually the reactivation of epidemic typhus
infection caused by Rickettsia prowazekii. It can occur many years after an infection that was not
treated with antibiotics.

136
Q
  1. A patient experiences a severe bite wound from a pit bull. He receives a booster injection
    of tetanus toxoid and an injection of penicillin G. Several days later, the wound is inflamed and
    purulent. The exudate is cultured on blood agar and yields gram-negative rods. Antibiotic
    sensitivity tests are pending. Which of the following is the most likely pathogen?
    A. Bartonella henselae
    B. Brucella canis
    C. Clostridium tetani
    D. Pasteurella multocida
    E. Toxocara canis
A

The correct answer is D. Pasteurella multocida is a gram-negative rod that is normal flora of the oral
cavity of dogs and cats. It often causes a local abscess following introduction under the skin by an
animal bite. Most cases occur in children who are injured while playing with a pet. Note that penicillin
G is not highly effective against gram negative rods. Bartonella henselae (choice A)is a very small
gram-negative bacterium that is closely related to the rickettsia. It is the cause of cat-scratch disease
(a local, chronic lymphadenitis most commonly seen in children) and bacillary angiomatosis (seen
particularly in AIDS patients). In this latter patient population, the organism causes proliferation of
blood and lymphatic vessels, causing a characteristic “mulberry” lesion in the skin and subcutaneous
tissues of the afflicted individual. Brucella canis(choice B) is a gram-negative rod that is a zoonotic
agent. Its normal host is the dog, but when it gains access to humans, it causes an undulating febrile
disease with malaise, lymphadenopathy, and hepatosplenomegaly. The normal route of exposure is
by way of ingestion of the organism. Clostridium tetani (choice C) is a gram-positive spore-forming
anaerobic rod. It causes tetanus (a spastic paralysis caused by tetanospasmin, which blocks the
release of the inhibitory neurotransmitters glycine and gamma-aminobutyric acid [GABA]). There may
be no lesion at the site of innoculation and exudation would be extremely rare. Toxocara canis
(choice E), a common intestinal parasite of dogs, is a metazoan parasite that causes visceral larva
migrans. Young children are most likely to be affected, as they are most likely to ingest soil
contaminated with eggs of the parasite.

137
Q
  1. A patient with nuchal rigidity and headache undergoes lumbar puncture. The CSF
    contains markedly increased numbers of lymphocytes, leading to a presumptive diagnosis of
    viral meningitis. Which of the following groups of viruses is most likely to be involved?
    A. Adenoviruses
    B. Enteroviruses
    C. Human papillomaviruses
    D. Poxviruses
    E. Reoviruses
A

The correct answer is B. Viral meningitis is relatively common, accounting for 10,000 cases of
meningitis per year in the United States. The vast majority of cases occur in individuals younger than
30 years of age. Usually the symptoms are relatively mild and death is uncommon. Enteroviruses,
arboviruses, and type 2 herpes simplex virus are the most common causes of viral meningitis. Also,
up to 10% of HIV patients develop an acute meningitis, typically at the time of seroconversion.
Adenovirus (choice A) infection is associated with upper respiratory tract infections (URIs), sinusitis,
ocular disease, enteric infections, and bladder infections. It does not typically cause aseptic
meningitis. Human papillomaviruses (choice C) are associated with warts on the skin and genital
areas. Poxviruses (choice D) include the causative agents of smallpox, cowpox, and molluscum
contagiosum. Reoviruses include the most common family member, rotavirus, responsible for
gastroenteritis in young children. These agents do not typically cause meningitis.

138
Q
  1. A patient who is unable to tolerate clindamycin therapy for the treatment of an intraoral
    anaerobic infection would be most likely prescribed which of the following agents?
    A. Ciprofloxacin
    B. Metronidazole
    C. Tetracycline
    D. Tobramycin
    E. Vancomycin
A

The correct answer is B. Anaerobic bacteria exist as part of the normal flora on the mucosal
surfaces (the mouth, gastrointestinal tract, skin, and female genital tract) of humans and animals.
Anaerobic infections occur when the normal balance between aerobic and anaerobic bacteria is
disrupted.The two medications primarily used for the treatment of anaerobic infections are
clindamycin and metronidazole. Clindamycin is typically used in the treatment of serious anaerobic
infections when other less toxic medications cannot be used. A potential serious side effect is the
onset of pseudomembranous colitis. Metronidazole is a bacterial agent that is indicated for the
treatment of serious infections caused by anaerobic bacteria and is active against most protozoa. It is
primarily used for the treatment of gynecologic and abdominal infections and can be used for the
treatment of antibiotic-induced pseudomembranous colitis. The fluoroquinolones, such as
ciprofloxacin (choice A), have very poor anaerobic coverage. Ciprofloxacin therefore would be
relatively ineffective for the treatment of an oral infection caused by an anaerobic infection. These
agents are commonly used in the treatment of upper and lower respiratory infections and in
genitourinary tract infections. Tetracycline (choice C) is primarily used in the treatment of chlamydial
and mycoplasmal infections, gonorrhea, and acute exacerbations of chronic bronchitis. Tobramycin
(choice D) is an intravenous antibiotic typically reserved for the treatment of serious and lifethreatening
gram-negative bacterial infections and some gram-positive infections. Vancomycin
(choice E) is also typically reserved for the treatment of serious and life-threatening bacterial
infections; however, it is used primarily in the treatment of gram-positive infections.

139
Q
  1. Serum analysis of a symptomatic patient yields elevated ALT, HBsAg, Anti-HBc, HBeAg,
    and bilirubin. All other values are normal. What is the hepatitus B status of this recruit?
    A. Asymptomatic carrier
    B. Chronic active carrier
    C. Fulminant hepatitis B
    D. Recovered from acute self-limited HBV
    E. Vaccinated against HBV
A

The correct choice is B. The presence of elevated ALT, HBsAg, anti-HBc, HBeAg, and bilirubin all
point to active hepatitis B. An asymptomatic carrier (choice A) would not have elevated ALT and
bilirubin. The absence of other symptoms rules out fulminant hepatitis B (choice C). Recovery from
acute self-limited HBV (choice D) is associated with the presence of anti-HBs and a decrease in
HBsAg and HBeAg. Someone who is vaccinated with HBV (choice E) has only anti-HBs in serum.

140
Q
  1. A 23-year-old woman has Toxic Shock Syndrome. A diffuse erythematous rash with areas
    of desquamation over the hands and feet is noted. Infection with which of the following agents
    is the most likely cause of these signs and symptoms?
    A. Clostridium perfringens
    B. HIV-1
    C. Shigella dysenteriae
    D. Staphylococcus aureus
    E. Staphylococcus epidermidis
A

The correct answer is D. Toxic Shock Syndrome (TSS), a multisystem syndrome caused by a toxin
(TSST-1) formed by certain strains of S. aureus. TSS usually affects several organ systems
(gastrointestinal, renal, hepatic, hematopoietic, musculoskeletal, pulmonary) and can result in death.
TSS historically has been associated with the use of tampons (as well as cervical cups and
diaphragms) in young women, but also can occur in other patient populations. Symptoms often have a
very abrupt onset, including fever, hypotension, diarrhea, and diffuse rash with desquamation of the
hands and feet. Blood cultures are negative, because sypmtoms are caused by the toxin not the
invasive organism. Management of shock, renal failure, and adult respiratory distress syndrome
(ARDS) are a priority if these conditions are present, in addition to appropriate antibacterial treatment.
C. perfringens (choice A) causes gas gangrene, with necrosis of soft tissues, usually after a traumatic
wound. It is also a cause of food poisoning. Although HIV-1 (choice B) can cause many diverse
findings and should never be immediately ruled out, the findings in this patient are most specific for
TSS. Shigella dysenteriae(choice C) is a cause of dysentery characterized by fever, abdominal
cramps, and bloody diarrhea. S. epidermidis (choice E) is part of the normal skin flora, but is
notorious for causing infections of intravenous lines and prosthetic heart valves.

141
Q
  1. A patient was recently prescribed fludrocortisone inhalation therapy to improve his
    current asthma treatment regimen. Ten days after starting therapy he presents with white
    patches on the inside of the cheeks that can be easily wiped off leaving a red, bleeding, sore
    surface. This patient would be best treated with which of the following?
    A. Acyclovir Capsules
    B. Ampicillin Capsules
    C. Cephalexin Suspension
    D. Clarithromycin Tablets
    E. Clotrimazole Troches
A

The correct answer is E. Candidiasis can appear in any area of the oral mucosa. One of the most
common causes of candidiasis in the adult population is the use of oral inhalation corticosteroids,
such as fludrocortisone. The pseudomembranous form typically appears with white patches that can
be easily wiped off leaving red, bleeding, sore surfaces. There is also an erythematous form that
presents with flat red and white lesions that cannot be rubbed off. Based on the description of the
signs and symptoms in this question, the patient has the pseudomembranous form of oral candidiasis
or oral thrush. Oral candidiasis responds very well to antifungal therapy. Clotrimazole and nystatin are
antifungal agents used locally for treatment of infections caused by many different Candida species.
Clotrimazole Troches are the generic name for the brand name product Mycelex Troches. A troche is
essentially an orally disintegrating tablet that dissolves in the mouth and is generally used in adults
with candidiasis. Acyclovir (choice A) is an antiviral agent used in the treatment of infections caused
by herpes simplex virus types 1 and 2 and varicella-zoster virus. This agent would be indicated in
individuals with herpes zoster infections that typically appear as vesicular eruptions and/or ulcers on
the cheek, tongue, gingival or palate. Both ampicillin (choice B) and cephalexin (choice C) are betalactam
antibiotics used to treat a number of bacterial infections. Both agents would be ineffective in
fungi. Clarithromycin (choice D) is a macrolide antibiotic that may be safely and effectively used for
the treatment of bacterial pharyngitis, as well as various other infections caused by susceptible
organisms. It is also used in SBE prophylaxis for dental procedures.

142
Q
  1. A patient is suffering from pneumonia, and culture shows numerous gram-positive cocci
    that are identified as Streptococcus pneumoniae. Which of the following immune effector
    mechanisms is most important in completely clearing this infection?
    A. ADCC (antibody-dependent cell cytotoxicity)
    B. Complement-mediated opsonization
    C. Cytotoxic T cell lymphocytes
    D. LAK cells
    E. Natural killer cells
A

The correct answer is B. One of the most efficient mechanisms for eliminating extracellular
pathogenic bacteria is by opsonization and phagocytosis by macrophages. The IgG and IgM antibody
produced in response to the organism reacts with the capsular structure, stimulating the activation of
the classic pathway of the complement system. This pathway produces large amounts of C3b that
56
coat the organism, preparing it for phagocytosis. ADCC cells (choice A) are actually natural killer
(NK) cells that find virally infected cells and tumor cells that have been coated with antibody and react
with and destroy them. These cells do not destroy antibody-coated bacteria, only body cells that are
coated with antibody. Cytotoxic T lymphocytes (choice C) react only with cells that have antigen
epitopes presented in association with class I MHC molecules. An example would be a virus epitope
from a virally infected cell presented by class I molecules on the surface of the cell. LAK cells (choice
D) are NK cells that have been activated by IL-2. They are considered to be superactivated NK cells.
NK cells (choice E) are cells of the innate immune system that destroy virally infected cells or tumor
cells. This does not involve antibody and it does not involve extracellular pathogens.

143
Q
  1. A 33-year-old man with AIDS and a history of shingles develops a severe, multifocal
    encephalitis. Which of the following viruses is the most likely cause of his encephalitis?
    A. Cytomegalovirus
    B. Herpes simplex type I
    C. Herpes simplex type II
    D. Herpes zoster-varicella
    E. Measles virus
A

The correct answer is D. The specific clue to the cause of the severe encephalitis in this patient is
the history of shingles, caused by reactivation of the herpes zoster-varicella virus. In otherwise healthy
adults, the virus (which is usually introduced to the body as a childhood case of chickenpox) remains
dormant in a dorsal root ganglion, only to reactivate in later life as a painful dermatomal vesicular
eruption. In patients with AIDS, the virus can cause a severe, multifocal encephalitis that may be
resistant to acyclovir therapy. Cytomegalovirus (choice A), Herpes simplex type I (choice B) and
herpes simplex type II (choice C), can cause disseminated disease (including brain infection) in AIDS
patients, but is less likely in this patient, given the past history of shingles and the lack of response to
acyclovir. Measles virus (choice E) appears to be related to subacute sclerosing panencephalitis, but
this condition is not specifically increased in AIDS patients.

144
Q
  1. A patient with a lung lesion coughs up sputum that contains thin, acid-fast positive rods.
    Which of the following features would most likely be associated with these bacteria?
    A. Nutritional requirement for factors V and X
    B. Streptokinase
    C. Toxic shock syndrome toxin
    D. Visible under dark field illumination
    E. Waxy envelope
A

The correct answer is E. M. tuberculosis is a non-spore-forming thin aerobic bacteria. Once stained
the bacilli are not discolorized by acid alcohol; hence the term acid-fast. Mycobacteria are also
considered “acid fast” because they have an envelope that contains large amounts of lipids and even
true waxes (unlike envelopes of other types of bacteria) that prevents the acid-fast stain
(carbolfuchsin) from leaking out. The other characteristics listed in the answers are commonly tested
features of specific bacteria: Nutritional requirement for factors V and X (choice A) is a feature of
Haemophilus influenzae. Streptokinase (choice B) is produced by Streptococci. Toxic shock
syndrome toxin (choice C) is a feature of Staphylococcus aureus. Visibility under dark field
illumination (choice D) is a feature of the syphilis organism Treponema pallidum. Furthermore, T.
pallidum does not grow on artificial media and therefore cannot be cultured in a laboratory.

145
Q
  1. A 58-year-old alcoholic with multiple dental caries develops a pulmonary abscess and is
    treated with antibiotics. Several days later he develops nausea, vomiting, abdominal pain, and
    voluminous green diarrhea. Which of the following antibiotics is most likely responsible for
    this patient’s symptoms?
    A. Chloramphenicol
    B. Clindamycin
    C. Gentamicin
    D. Metronidazole
    E. Vancomycin
A

The correct answer is B. Any time you see the development of diarrhea in the same question stem
with “treated with antibiotics,” you should immediately think of pseudomembranous colitis. This
57
condition is caused by Clostridium difficile and typically occurs as a result of treatment with
clindamycin or ampicillin. You would confirm your suspicion by sending a stool culture to be tested for
the presence of the C. difficile toxin. The most test-worthy side effect of chloramphenicol (choice A) is
aplastic anemia, not diarrhea. In addition, you might have been able to eliminate this choice simply
because of the extremely low probability that this patient would receive this antibiotic in the United
States. The key side effects of gentamicin (choice C) include ototoxicity and nephrotoxicity.
Metronidazole (choice D) and vancomycin (choice E) do not cause pseudomembranous colitis; they
are used to treat it.

146
Q
  1. A 37-year-old man is admitted to the hospital with shortness of breath, cyanosis, and
    fever. Chest x-rays reveal consolidation of the right lower lobe with relative sparing of the
    remaining lobes. A clinical diagnosis of lobar pneumonia is made and supported by the results
    of sputum cultures. Which of the following is the genus of the bacterium most likely to be
    isolated from this patient’s sputum?
    A. Haemophilus
    B. Klebsiella
    C. Streptococcus, alpha-hemolytic
    D. Streptococcus, beta-hemolytic
    E. Streptococcus, gamma-hemolytic
A

The correct answer is C. The clinical and radiologic characteristics of this patient’s condition are
consistent with lobar pneumonia, a respiratory infection that, in its classic presentation, involves a
single pulmonary lobe. In 90-95% of cases, the etiologic agent is Streptococcus pneumoniae (AKA
pneumococcus), an alpha-hemolytic streptococcus present in the throat of 40-70% of healthy
individuals. This organism is best described as a gram-positive diplococcus. Penicillin is the drug of
choice for pneumococcal pneumonia, but sputum cultures are necessary to identify the infectious
agent and determine its antibiotic sensitivity.S. pneumoniae is a major cause of purulent meningitis in
the elderly. Other alpha-hemolytic streptococci (viridans streptococci) cause subacute endocarditis in
patients with previously altered cardiac valves. Bacteria belonging to the genera Haemophilus(choice
A) and Klebsiella (choice B) cause respiratory infections; however, in immunocompetent individuals,
these bacterial infections usually result in bronchopneumonia, which leads to multilobar, and often
bilateral, pulmonary involvement. Beta-hemolytic streptococci (choice D) cause countless infections
in humans. Recall that classification of beta-hemolytic streptococci is based on their surface antigens
known as Lancefield antigens. Human diseases caused by this group of bacteria include: -
Streptococcus pyogenes (group A): pharyngitis, scarlet fever, erysipelas, impetigo, rheumatic fever,
and glomerulonephritis. - Streptococcus agalactiae (group B): neonatal sepsis and urinary infections.
- Enterococcus faecalis (group D): endocarditis and urinary infections. Respiratory involvement is
rarely seen. Gamma-hemolytic streptococci (choice E) are streptococci that do not produce
hemolysins (nonhemolytic streptococci) and are not a significant cause of human disease.

147
Q
  1. Which of the following organisms would most likely be isolated from shunt tubing or
    indwelling catheters in hospitalized children who develop meningitis?
    A. Bacteroides fragilis
    B. Corynebacterium diphtheriae
    C. Escherichia coli
    D. Staphylococcus epidermidis
    E. Streptococcus pneumoniae
A

The correct answer is D. Staphylococcal meningitis is fairly rare, occurring mostly in patients with
indwelling ventricular-peritoneal shunts. Staphylococcus epidermidis, a coagulase-negative organism
that normally colonizes the skin, is the most common organism causing this disorder; Staphylococcus
aureus meningitis occasionally occurs. Bacteroides fragilis(choice A) is a common cause of
anaerobic infections, including sepsis and peritonitis, but it does not commonly cause meningitis.
Diphtheroids (including Corynebacterium diphtheriae, choice B) are sometimes isolated from
indwelling shunts, but in many cases they are simply contaminants, because some diphtheroids are
normal skin flora. Escherichia coli(choice C) is isolated from 30-50% of neonates with bacterial
meningitis, but is not typically associated with shunt infections. Streptococcus pneumoniae(choice E)
is the most common cause of bacterial meningitis in people older than 30 years of age; it is much less
common in children.

148
Q
  1. A 25-year-old woman presents with an erythematous rash that began on her face, then
    spread downward over her trunk. She has a fever and headache, bilateral pain of the front and
    back of her neck, and arthralgia. Which of the following diseases does she most likely have?
    A. Infectious mononucleosis
    B. Lyme disease
    C. Roseola
    D. Rubella
    E. Rubeola
A

The correct answer is D. Rubella, or German measles, is a disease caused by a Togavirus, which is
a small, enveloped, single-stranded, (+) linear RNA virus. Approximately 40% of patients are
asymptomatic or have mild symptoms. In symptomatic patients, the clinical presentation typically
consists of an erythematous rash beginning on the head that spreads downward to involve the trunk,
lasting for approximately 3 days. In addition to the rash, symptoms include fever, malaise, and
arthralgias coinciding with the eruption. Leukopenia also may be seen. Enteroviral rashes may mimic
rubella and rubeola. Infectious mononucleosis (choice A) is caused by the Epstein-Barr virus, a
herpesvirus. Classic findings include fever, exudative pharyngitis, generalized lymphadenopathy,
severe malaise (most common complaint), and hepatosplenomegaly. A rash is not a characteristic
feature unless the patient has been treated with ampicillin. This is a self-limited disease that is rarely
fatal unless splenic rupture occurs. Lyme disease (choice B) is caused by the spirochete Borrelia
burgdorferi. The disease is transmitted by the bite of the tick, Ixodes dammini. Reservoirs in nature
include deer and mice. The initial lesion is an annular rash with central clearing and a raised red
border (erythema chronicum migrans) at the bite site. The rash is warm, but not painful or itchy.
Patients also have fever, malaise, myalgias, arthralgias, headache, generalized lymphadenopathy,
and, occasionally, neurologic findings. Roseola (choice C) is caused by human herpesvirus 6. Other
names include exanthem subitum or sixth disease. Children have a febrile period of 3-5 days with
rapid defervescence followed by an erythematous maculopapular rash lasting 1-3 days. Rubeola
(choice E), or measles, is caused by a paramyxovirus. Patients present with an upper respiratory
prodrome and characteristic oral lesions (Koplik spots) that precede the rash. The nonpruritic
maculopapular rash begins on the face and spreads to the trunk and extremities, including palms and
soles. The incubation period is 10-14 days. Patients also have a posterior cervical lymphadenopathy.
The virus is not associated with risk to a fetus.

149
Q
  1. A 60-year-old alcoholic smoker abruptly develops high fever, shakes, a severe headache,
    muscle pain, and a dry, insignificant cough. Later he develops marked shortness of breath
    requiring assisted ventilation. Chest x-ray reveals left lower lobe shadowing that spreads until
    both lungs are extensively involved. Culture of bronchoalveolar lavage fluid demonstrates a
    coccobacillary pathogen. Which is the most likely causative organism?
    A. Legionella pneumophila
    B. Listeria monocytogenes
    C. Pseudomonas aeruginosa
    D. Staphylococcus aureus
    E. Streptococcus pneumoniae
A

The correct answer is A. The patient has a severe, potentially fatal, pneumonia with prominent
systemic symptoms. Culture revealing a coccobacillus is the specific clue that the organism is
Legionella pneumophila. The disease is respiratory Legionellosis, also known as Legionnaire disease.
Patients tend to be older (40-70 years of age) and may have risk factors including cigarette use,
alcoholism, diabetes, chronic illness, or immunosuppressive therapy.
The bacteria can also be seen in the sputum with direct fluorescent antibody staining. Listeria
monocytogenes(choice B) causes listeriosis and is not a notable cause of pneumonia. Pseudomonas
aeruginosa (choice C) is a gram-negative rod that is a cause of pneumonia in patients on ventilators
and those with cystic fibrosis.
Staphylococcus aureus(choice D) can cause pneumonia, but is a plump gram-positive cocci in
clumps. Streptococcus pneumoniae(choice E) can cause pneumonia, but is a gram-positive
diplococci.

150
Q
151. The electron transport system of Neisseria is located on which of the following
structures?
A. Cytoplasmic membrane
B. Mesosome
C. Mitochondria
D. Nuclear membrane
E. Polyribosome DNA aggregates
A

The correct answer is A. This is a good microbiology question that uses the principles of bacterial
physiology. Unlike eukaryotic cells in which the electron transport system is located on mitochondria,
the electron transport system of bacteria is located on the cytoplasmic (plasma) membrane.
Mesosomes (choice B) and polyribosome DNA aggregates (choice E) are also found in bacterial
cells and function in cell division and protein synthesis, respectively. Bacteria do not have
mitochondria (choice C) or membrane-bounded nuclei (choice D).

151
Q
  1. A 32-year-old man presents to the emergency room with a severe headache. Nuchal
    rigidity is found on physical examination. Lumbar puncture demonstrates cerebrospinal fluid
    with markedly increased lymphocytes. Other cell populations are not increased. If the fluid is
    clear in appearance, which of the following agents is the most likely cause of his symptoms?
    A. Escherichia coli
    B. Haemophilus influenzae
    C. Herpes virus
    D. Mycobacterium tuberculosis
    E. Treponema pallidum
A

The correct answer is C. The clinically suspected diagnosis is meningitis, which is confirmed by the
abnormal cerebrospinal fluid. The markedly increased lymphocytes suggests acute lymphocytic
meningitis, which is distinguished from acute pyogenic meningitis (increased neutrophils as well as
lymphocytes). Acute lymphocytic meningitis is usually viral in origin. Among the many viruses that
have been implicated, mumps, herpes, Epstein-Barr, echovirus, and Coxsackie virus are the most
common. One of the easiest ways to differentiate between a viral and bacterial meningitis is the
appearance of the cerebral spinal fluid. A viral meningitis will not generally change the color of the
CSF. However, a bacterial meningitis often produces a cloudy CFS. Escherichia coli(choice A) and
Haemophilus influenzae(choice B) cause acute pyogenic meningitis and would be expected to
produce a cloudy cerebral spinal fluid. Mycobacterium tuberculosis(choice D) and Treponema
pallidum(choice E) cause chronic meningitis and would be expected to produce a cloudy fluid.

152
Q
153. Which of the following diseases is associated with the "Dane Particle"?
A. Hepatitis A
B. Hepatitis B
C. Hepatitis C
D. Hepatitis D
E. Hepatitis E
A

The correct answer is choice B. A Dane particle is the name for the entire Hepatitis B virion (viral
particle). It is one of three entities that will test positive as Hepatitis B surface antigens. The other two
are pieces of viral coats known as spheres or filaments, which, although they are not complete viral
particles, they do contain the surface antigen (HBsAg). Other hepatitis B antigens include the core
antigen (HBcAg) and an “e-antigen” (HBeAg). The “e-antigen” is associated with increased viral
replication and infectivity of the patient.